Jump to content

bangbang1412's Content

There have been 135 items by bangbang1412 (Search limited from 08-06-2020)



Sort by                Order  

#737676 Thế nào là một lược đồ?

Posted by bangbang1412 on 12-03-2023 - 22:09 in Toán học lý thú

Cảm ơn @bangbang1412 rất nhiều về bài viết mà phải nói là quá công phu và trình bày rất đẹp mắt! Tuyệt vời!

Mới bổ sung khá nhiều anh ạ :icon6: do em ở nhà cuối tuần chán nên viết tý.




#737364 Phỏng vấn với Jean-Pierre Serre

Posted by bangbang1412 on 20-02-2023 - 23:49 in Kinh nghiệm học toán

Phỏng vấn với Jean-Pierre Serre

 

Serre.jpg

 

While other sciences search for the rules that God has chosen for the Universe, we mathematicians search for the rules that even God has to obey.

 

Trong khi các ngành khoa học khác tìm kiếm các quy luật mà Chúa đã chọn cho vũ trụ, chúng tôi những nhà toán học tìm kiếm các quy luật mà ngay cả Chúa cũng phải tuân theo.

 

Jean-Pierre Serre sinh năm 1926 tại Pháp. Ông từng theo học toán tại đại học sư phạm Paris. Vào năm 1954, ở tuổi 28, ông đã được giải Fields bởi Hiệp hội Toán học Quốc tế, chứng nhận cao nhất cho một thành tựu trong toán học. Hai năm sau ông được bổ nhiệm chức Giáo sư về Đại số và Hình học tại College de France, nơi mà ông là giáo sư trẻ nhất trong khoảng 15 năm. Ông thăm khoa toán Đại học Quốc Gia Singapore từ ngày 2 tới 15 tháng Hai năm 1985. Chuyến thăm của ông được tài trợ chương trình trao đổi học thuật Pháp-Sing. Khi ở Singapore, giáo sư Serre đã trình bày hai bài giảng về đường cong đại số trên trường hữu hạn và một bài giảng về hàm Ramanujan. Ông cũng góp một bài nói seminar hai tiếng về chứng minh của Falting cho giả thuyết Mordell, và một bài giảng hội đàm với tiêu đề "Biệt thức = $b^2-4ac$" về class numbers của các trường toàn phương ảo (imaginary quadratic fields). Vào ngày 14 tháng hai năm 1985, ông có một cuộc phỏng vấn trong đó ông thảo luận nhiều khía cạnh sự nghiệp toán học của mình và cách nhìn của ông về toán học. Những gì sau đây là ghi chép từ cuộc phỏng vấn đó, chỉnh sửa bởi C. T. Chong và Y. K. Leong, và đã được kiểm tra lại bởi J. P. Serre.

 

Q: Điều gì khiến ông chọn toán học làm sự nghiệp của mình?

 

A: Tôi nhớ rằng tôi bắt đầu thích toán vào khoảng năm 7 hoặc 8 tuổi. Hồi còn trung học tôi thường giải các bài toán của các lớp lớn hơn. Hồi ấy tôi ở một khu nhà trọ ở Nimes cùng với lũ trẻ lớn hơn và chúng thường bắt nạt tôi. Để làm hoà với chúng, tôi thường làm hộ bài tập về nhà môn toán cho chúng. Dù sao đó cũng là một bài luyện tập tốt.

 

Mẹ tôi là một dược sĩ (cũng như bố tôi), và bà yêu toán học. Khi bà ấy vẫn còn là một sinh viên dược tại đại học Montpellier, bà ấy đã đăng ký học một khoá học năm đầu tiên môn giải tích, chỉ để cho vui, và bà đã vượt qua bài kiểm tra. Và bà ấy đã giữ cẩn thận những quyển sách giải tích của mình (viết bởi Fabry và Vogt, nếu tôi nhớ không nhầm). Khi tôi 14 hoặc 15, tôi thường xem những quyển sách này và nghiên cứu chúng. Đó là cách tôi học về đạo hàm, tích phân, chuỗi và các thứ kiểu thế (tôi học theo một cung cách hình thức thuần tuý - phong cách của Euler với tôi mà nói thì tôi không thích, và không hiểu ngôn ngữ epsilon delta.) Vào thời điểm đó, tôi không biết rằng người ta có thể kiếm sống bằng cách trở thành một nhà toán học. Chỉ sau này tôi mới khám phá ra rằng người ta được trả tiền để làm toán! Đầu tiên tôi nghĩ tôi sẽ trở thành một giáo viên trung học, điều đó dường như là tự nhiên với tôi. Nhưng khi tôi 19, tôi đã đăng ký kì thi vào trường Đại học Sư phạm Paris và thành công. Khi tôi đã ở trong "Trường", tôi thấy rõ hơn rằng không phải giáo viên trung học là cái tôi muốn trở thành, mà tôi muốn trở thành một nhà toán học.

 

Q: Những lĩnh vực khác có từng thu hút ông không, như vật lý hoặc hoá học?

 

A: Vật lý thì không nhiều lắm, nhưng hoá học thì có đấy. Như tôi đã nói, bố mẹ tôi đều là dược sĩ, nên họ có cả đống hoá chất và ống nghiệm. Tôi nghịch chúng khi tôi mới 15 16 ngoài việc làm toán. Và tôi đọc những quyển sách hoá học của bố tôi (tôi vẫn giữ một trong số chúng, một quyển sách lôi cuốn, "Les Colloides" của Jacques Duclaux). Tuy nhiên, khi học hoá, tôi thất vọng vì cái khía cạnh toán học nửa vời của nó: có một chuỗi các hợp chất hữu cơ như $CH_4,C_2H_6,...$, tất cả đều trông na ná nhau. Tôi nghĩ, nếu mà phải có chuỗi thì thà tôi đi làm toán! Do đó tôi từ bỏ hoá học - nhưng không hoàn toàn: cuối cùng tôi cưới một nhà hoá học.

 

Q: Ông có từng bị ảnh hưởng bởi giáo viên nào trong việc làm toán không?

 

A: Tôi chỉ từng có đúng một giáo viên tốt. Đó là năm đầu tiên trung học của tôi (1943-1944) ở Nimes. Ông ấy có biệt danh "Le Barbu": để râu rất hiếm vào thời đó. Ông ấy luôn rất rõ ràng và chặt chẽ; ông ấy yêu cầu mọi công thức và chứng minh phải được trình bày gọn gàng. Và ông ấy đã dành cho tôi một bài luyện tập tận tâm cho kỳ thi toán học quốc gia có tên "Concours General", nơi mà tôi dành giải nhất.

 

Nói về Concours General, tôi cũng thử với tay mình sang kỳ thi đó bên vật lý vào cùng năm (1944). Vấn đề chúng tôi được hỏi hoàn toàn dựa vào một định luật vật lý mà tôi nên biết, nhưng tôi lại không biết nó. May mắn thay, chỉ có một công thức dường như có thể áp dụng cho định luật đó. Tôi đã giả sử nó đúng, và triển khai làm việc với một vấn đề-6-tiếng dựa trên giả sử đó. Tôi thậm chí còn nghĩ mình sẽ đạt giải. Không may mắn lắm, công thức của tôi sai, và tôi không đạt được gì - điều tôi xứng đáng!

 

Q: Cảm hứng có vai trò quan trọng như thế nào trong việc tìm ra các định lý?

 

A: Tôi không biết "cảm hứng" thực sự có nghĩa gì. Các định lý và các lý thuyết đến theo những cách buồn cười. Thỉnh thoảng, anh chỉ không hài lòng với các chứng minh đã có, và anh tìm những chứng minh tốt hơn để có thể áp dụng cho các tình huống khác. Một ví dụ điển hình với tôi là khi tôi làm việc với định lý Riemann-Roch (khoảng năm 1953), mà tôi xem nó như một công thức "Euler-Poincare" (tôi đã không biết rằng Kodaira-Spencer đã có cùng ý tưởng.) Mục tiêu đầu tiên của tôi là chứng minh nó cho các đường cong đại số - một trường hợp đã biết từ cả thế kỷ trước! Nhưng tôi muốn một chứng minh theo một phong cách đặc biệt, và khi tôi thành công trong việc tìm ra nó, tôi nhớ rằng tôi mất không quá một hoặc hai phút để đi từ đó lên trường hợp 2 chiều (điều được chứng minh bởi Kodaira). Sáu tháng sau, kết quả hoàn chỉnh được đưa ra bởi Hirzebruch, và công bố trong bài Habilitationsschrift nổi tiếng của ông ấy.

 

Thông thường, anh không giải quyết một vấn đề bằng cách tấn công trực diện nó. Thay vào đó anh có vài ý tưởng trong đầu mà anh cảm giác sẽ có ích, nhưng anh không thật sự biết chúng có ích cho việc gì. Do đó, anh tìm kiếm xung quanh, và cố gắng áp dụng chúng. Giống như việc có cả chùm chìa khoá, và cố gắng thử chúng vào những cái cửa.

 

Q: Ông có từng có trải nghiệm nào mà ông ấy một vấn đề là không thể giải quyết, nhưng sau khi bỏ nó sang một bên một thời gian thì một ý tưởng đột nhiên xuất hiện dẫn tới lời giải không?

 

A: Có, điều này vẫn thường diễn ra. Ví dụ, khi tôi làm việc với các nhóm đồng luân (khoảng 1950), tôi tự thuyết phục mình rằng, với một không gian $X$, nên tồn tại một không gian phân thớ $E$ với nền $X$ mà có thể co rút về một điểm; một không gian như thế có thể cho phép tôi (sử dụng các phương pháp của Leray) làm rất nhiều các tính toán về nhóm đồng luân và đối đồng điều của không gian Eilenberg-MacLane. Nhưng làm thế nào để tìm ra nó? Tốn vài tuần (một thời gian rất dài, vào cái tuổi tôi lúc đó...) để tôi nhận ra không gian các đường trên $X$ có đủ các tính chất cần thiết - chỉ khi tôi dám gọi nó là không gian phân thớ, và tôi đã làm vậy. Đây là điểm bắt đầu của phương pháp không gian cung trong tôpô đại số, rất nhiều kết quả sau đó nhanh chóng được chứng minh.

 

Q: Ông thường làm việc với chỉ một vấn đề hay nhiều vấn đề cùng lúc?

 

A: Hầu như chỉ một vấn đề từng thời điểm, nhưng không phải luôn luôn. Và tôi thường làm việc buổi đêm (giấc ngủ chập chờn), khi mà anh không phải viết thứ gì ra sẽ làm cho đầu óc có độ tập trung cao hơn, và thay đổi các chủ đề dễ dàng hơn.

 

Q: Trong vật lý, có rất nhiều khám phá là do tình cờ, ví dụ như tia X, bức xạ nền vũ trụ, vân vân. Điều đó có xảy ra với ông trong toán học?

 

A: Một sự tình cờ thật sự là rất hiếm. Nhưng thi thoảng anh vẫn ngạc nhiên vì một lập luận của anh cho một mục đích lại có thể giải quyết một câu hỏi trong một hướng nghiên cứu khác; tuy nhiên, người ta khó có thể gọi nó là "tình cờ".

 

Q: Đâu là những vấn đề cốt lõi trong hình học đại số và lý thuyết số?

 

A: Tôi không thể trả lời chính xác. Anh thấy đấy, vài nhà toán học có những chương trình rõ ràng và dài hơi. Ví dụ, Grothendieck có một chương trình như thế cho hình học đại số; bây giờ Langlands có một cái như vậy cho lý thuyết biểu diễn, trong mối quan hệ với dạng modular và số học. Tôi chưa từng có một chương trình như vậy, kể cả một cái cỡ nhỏ. Tôi chỉ làm việc với những thứ hấp dẫn tôi tại một thời điểm. (hiện tại, vấn đề hấp dẫn tôi nhất là đếm số điểm trên các đường cong đại số trên những trường hữu hạn. Nó là một kiểu toán ứng dụng: anh cố sử dụng bất kỳ công cụ nào trong hình học đại số và lý thuyết số mà anh biết... và anh không thành công lắm!)

 

jeanpierreserre.jpg

 

...comme Grothendieck nous l'a appris, les objets d'une catégorie ne jouent pas un grand rôle, ce sont les morphismes qui sont essentiels.

 

Like Grothendieck has taught us, objects of a category do not play a great role, it is the morphisms between them that are essential.

 

Q: Ông cho rằng đâu là những tiến bộ vượt bậc nhất trong hình học đại số và lý thuyết số năm năm trở lại đây?

 

A: Câu này dễ trả lời hơn. Chứng minh của Falting cho giả thuyết Mordell và giả thuyết Tate là điều đầu tiên tôi nghĩ đến. Tôi cũng xin nhắc đến công trình của Gross-Zagier về vấn đề số lớp (class number problem) cho các trường toàn phương (dựa trên một định lý trước đó của Goldfeld), và định lý Mazur-Wiles về lý thuyết Iwasawa, sử dụng đường cong modular. (Ứng dụng của đường cong modular và hàm modular vào lý thuyết số là cực kỳ thú vị: anh sử dụng $\mathrm{GL}_2$ để nghiên cứu $\mathrm{GL}_1$, có thể nói như vậy! Hiển nhiên là còn rất nhiều thứ sẽ tới theo hướng đó… thậm chí có thể là chứng minh giả thuyết Riemann vào một ngày nào đó!) 

 

Q: Vài nhà khoa học đã làm được một số công trình nền tảng trong một lĩnh vực và nhanh chóng chuyển sang lĩnh vực khác. Ông đã làm việc ba năm với tôpô, và sau đó làm thứ khác. Điều đó đã xảy ra như thế nào?

 

A: Nó là một con đường liền, không phải một thay đổi rời rạc. Năm 1952, sau luận án của tôi về các nhóm đồng luân, tôi đến Princeton nơi mà tôi có bài giảng về luận án của mình (và phần tiếp theo của nó “lý thuyết C”), và tham dự seminar nổi tiếng Artin-Tate về lý thuyết trường các lớp. 

 

Sau đó tôi trở về Paris, nơi mà seminar Cartan đang thảo luận về các hàm phức nhiều biến, và các đa tạp Stein. Hoá ra các kết quả của Cartan-Oka có thể được biểu diễn hiệu quả hơn (và được chứng minh theo cách đơn giản hơn) nhờ sử dụng đối đồng điều và bó. Điều đó khá phấn khích, và tôi đã làm việc một thời gian ngắn về chủ đề này, áp dụng lý thuyết Cartan vào các đa tạp Stein. Tuy nhiên, một mảng thú vị của lý thuyết hàm phức nhiều biến là nghiên cứu các đa tạp xạ ảnh (đối lập với các đa tạp affine - được xem là khá bệnh hoạn với một nhà hình học); bởi vậy tôi bắt đầu làm việc với các đa tạp xạ ảnh phức, sử dụng lý thuyết bó: đó là cách tôi đến với cái vòng các ý tưởng xung quanh Riemann-Roch vào năm 1953. Nhưng các đa tạp xạ ảnh thì đại số (định lý Chow), và nó hơi không tự nhiên nếu muốn nghiên cứu những đối tượng đại số này bằng các hàm giải tích, vốn có thể có rất nhiều kỳ dị. Rõ ràng, chỉ nên làm với các hàm hữu tỷ là đủ - và đúng là như vậy. Điều đó đưa tôi (vào năm 1954) đến với hình học đại số trừu tượng trên bất kỳ trường đóng đại số nào. Nhưng cần gì  tính đóng đại số? Các trường hữu hạn thú vị hơn, như giả thuyết Weil và những thứ tương tự. Và từ đó đến với những trường số, đó là một bước chuyển tiếp tự nhiên… Đây ít nhiều là con đường tôi đã theo đuổi. 

 

Một hướng nghiên cứu khác đến tự sự hợp tác (và tình bạn) của tôi với Armand Borel. Ông ấy nói về tôi về nhóm Lie, thứ mà ông ấy biết như không ai khác. Mối liên hệ giữa chúng với tôpô, hình học đại số, lý thuyết số,... rất lôi cuốn. Để tôi cho anh một ví dụ (mà tôi để ý vào khoảng năm 1968): 

 

Hãy xem xét nhóm con rời rạc hiển nhiên nhất của $\mathrm{SL}_2(\mathbf{R})$, tức là $G = \mathrm{SL}_2(\mathbf{Z})$. Anh có thể tính “đặc trưng Euler-Poincare” $X(G)$ của nó và nó bằng $-1/12$ (nó không phải là số nguyên, vì $G$ có xoắn). Giờ anh thấy $-1/12$ là giá trị $\zeta(-1)$ của hàm Riemann-zeta tại $s=-1$ (một kết quả biết từ thời Euler). Và đây không phải là sự trùng hợp! Nó mở rộng lên bất kỳ trường số $K$ hoàn toàn thực nào (totally real number field), và có thể sử dụng để nghiên cứu mẫu số của $\zeta_k(-1)$. (Các kết quả tốt hơn thu được bằng cách sử dụng dạng modular, như sau này được tìm ra.) Những câu hỏi như thế không phải lý thuyết nhóm, không phải tôpô, cũng chẳng phải lý thuyết số: chúng chỉ là toán học.

 

Q: Đâu là những triển vọng trong việc đạt được vài sự thống nhất các lĩnh vực toán học đa dạng?

 

A: Tôi có thể nói điều này đã đạt được rồi. Tôi đã đưa ra trong câu hỏi trước một ví dụ điển hình về nhóm Lie, lý thuyết số, vân vân, đến cùng nhau và không thể bị tách rời. Để tôi cho anh một ví dụ khác (và không khó để đưa ra thêm nhiều ví dụ như vậy): 

 

Có một định lý được chứng minh gần đây bởi S. Donaldson về các đa tạp khả vi compact 4 chiều. Nó nói rằng một dạng toàn phương (trên $H^2$) của một đa tạp như thế bị hạn chế nghiêm trọng: nếu nó xác định dương, nó phải là tổng các bình phương. Và mấu chốt của chứng minh là xây dựng một đa tạp phụ trợ (một “đồng biên”) như một tập nghiệm của các phương trình đạo hàm riêng (phi tuyến tính, hiển nhiên)! Đây là một ứng dụng hoàn toàn mới của giải tích vào tôpô vi phân. Và điều còn làm nó đáng chú ý hơn là, nếu tính khả vi bị bỏ đi, thì tình huống trở nên khá khác: bằng một định lý của M. Freedman, dạng $H^2$-toàn phương có thể là hầu như bất cứ thứ gì. 

 

Q: Làm thế nào để một người bắt kịp sự bùng nổ của kiến thức toán học? 


A: Anh không nhất thiết phải bắt kịp. Khi anh hứng thú với một câu hỏi cụ thể, anh sẽ thấy rất ít thứ đã được làm có liên quan gì đến anh, và nếu nó có liên quan gì đến anh, thì anh sẽ học nó nhanh hơn rất nhiều, vì anh có sẵn một ứng dụng trong đầu. Một thói quen tốt là thường kiểm tra Math. Reviews (đặc biệt là những ấn bản về lý thuyết số, lý thuyết nhóm, vân vân). Và anh cũng học rất nhiều từ các bạn của mình: sẽ dễ dàng khi xem một chứng minh được giải thích trên bảng đen hơn là khi đọc nó. 


Một vấn đề nghiêm trọng hơn là vấn đề với các “bài toán lớn”, những bài toán rất có ít nhưng chứng minh lại quá dài để kiểm tra (trừ khi anh dành ra một phần lớn thời gian đời mình…). Một ví dụ điển hình là định lý Feit-Thompson: các nhóm cấp lẻ thì giải được. (Chevalley có lần đã muốn lấy nó làm chủ đề cho một seminar với ý định tìm hiểu hoàn chỉnh chứng minh của nó. Sau hai năm, ông ấy bỏ cuộc.) Người ta nên làm gì với các bài toán như thế nếu họ buộc phải dùng tới nó? Chấp nhận chúng bằng niềm tin? Có thể. Cơ mà nó không phải là cái gì thoải mái lắm.


Bản thân tôi khá bứt rứt với vài chủ đề, chủ yếu là tôpô vi phân, nơi người ta vẽ một bức hình phức tạp (2 chiều) và bắt anh chấp nhận nó như chứng minh cho một cái gì đó trong 5 chiều hoặc hơn nữa. Chỉ có các chuyên gia có thể “thấy” một chứng minh như thế là đúng hay sai - nếu anh gọi đó là một chứng minh. 


Q: Ông nghĩ rằng máy tính sẽ ảnh hưởng gì đến sự phát triển của toán học?


A: Máy tính vốn đã làm rất tốt phần của nó trong vài nhánh toán học. Lấy ví dụ trong lý thuyết số thì chúng được dùng theo rất nhiều cách. Đầu tiên, để đề xuất các giả thuyết hoặc câu hỏi. Nhưng cũng để kiểm tra các định lý tổng quát trên các ví dụ tính toán - vốn giúp rất nhiều trong việc tìm ra các lỗi có thể có. 


Máy tính cũng rất có ích khi phải tìm kiếm diện rộng (ví dụ, khi anh có $10^6$ hoặc $10^7$ trường hợp). Một ví dụ khét tiếng là định lý bốn màu: Tuy nhiên có một vấn đề ở đây, hầu như tương tự như định lý Feit-Thompson: một chứng minh như thế không thể kiểm tra bằng tay; anh cần một cái máy tính (và một chương trình rất phức tạp). Điều đó cũng không thực sự thoải mái. 


Q: Làm thế nào để chúng ta khuyến khích những người trẻ theo đuổi toán học, đặc biệt là trong trường học?


A: Tôi có một lý thuyết thế này, điều đầu tiên người ta nên làm là không khuyến khích mọi người làm toán, chẳng có lý do gì để có nhiều nhà toán học. Nhưng nếu sau đó, họ vẫn khăng khăng đòi làm toán, thì anh nên khuyến khích họ, và giúp họ.


Với các học sinh trung học, điểm mấu chốt là phải làm cho các em hiểu rằng toán học tồn tại, rằng nó không chết (học sinh có xu hướng tin rằng chỉ có vật lý, hoặc sinh học mới có các bài toán mở). Khuyết điểm trong cách dạy toán truyền thống là giáo viên không bao giờ đề cập đến các bài toán như thế. Xót xa thay cho điều đó. Có rất nhiều bài toán như vậy, ví dụ trong lý thuyết số, mà các bạn trẻ vị thành niên có thể hiểu: của Fermat dĩ nhiên, hoặc Goldbach chẳng hạn, hoặc sự tồn tại của vô hạn số nguyên tố dạng $n^2+1$. Và người ta nên thoải mái phát biểu chúng mà không cần chứng minh (ví dụ định lý Dirichlet về các số nguyên tố trên các cấp số cộng). 


Q: Liệu ông có nghĩ rằng sự phát triển của toán học trong vòng ba mươi năm vừa rồi nhanh hơn năm mươi năm trước đó không?

 

A: Tôi không chắc liệu điều đó đúng không. Hoàn cảnh khác nhau mà. Trong những năm 50 60, điểm nhấn thường là về các phương pháp tổng quát: distributions, đối đồng điều, và kiểu thế. Những phương pháp như thế đã rất thành công, nhưng ngày nay người ta làm việc với những câu hỏi rất cụ thể (ví dụ, những câu hỏi rất cũ như phân loại đường cong đại số trong không gian xạ ảnh 3 chiều!). Họ áp dụng những công cụ đã có trước đó; điều này khá tốt. (Và họ cũng tạo ra những công cụ mới: microlocal analysis, supervarieties, intersection cohomology,...). (xin phép không dịch ba thuật ngữ này, ai biết có thể bổ sung)


Q: Trong sự bùng nổ của toán học, ông có nghĩ rằng một nghiên cứu sinh mới có thể tiếp thu một lượng lớn toán học trong bốn, năm năm hoặc sáu năm và ngay sau đó bắt đầu làm các công trình lớn không?


A: Tại sao không? Với một vấn đề anh thường không cần biết nhiều đến thế - và bên cạnh đó, những ý tưởng rất đơn giản đã làm được rồi. 


Vài lý thuyết được đơn giản hoá. Vài cái khác thì tuột khỏi tầm nhìn. Ví dụ, vào năm 1949, tôi nhớ rằng tôi đã trầm cảm vì mọi vấn đề trên Annals of Mathematics đều chứa một bài báo khác về tôpô mà khó đọc hơn bài báo trước đó. Nhưng không ai đọc những bài báo đó nữa; chúng đã bị lãng quên (và đáng bị như vậy: tôi không nghĩ rằng chúng có gì sâu sắc…). Quên là một hoạt động rất lành mạnh.


Nói gì thì nói, vài chủ đề cần luyện tập nhiều hơn một số chủ đề khác, bởi vì những kỹ thuật nặng nề được sử dụng. Hình học đại số là một ca như vậy; và lý thuyết biểu diễn cũng thế.


Dù sao thì một người không nên bảo là “tôi sẽ đi làm hình học đại số”. Với hầu hết mọi người, tốt hơn là nên bám theo các seminar, đọc, tự hỏi những câu hỏi; và học lượng kiến thức cần cho những câu hỏi đó. 


Q: Nói cách khác, một người nên nhắm tới một bài toán trước rồi sau đó học bất cứ công cụ nào cần thiết cho nó?


A: Kiểu vậy đấy. Nhưng vì tôi biết rằng tôi không có lời khuyên tốt cho chính bản thân, tôi sẽ không đưa ra lời khuyên cho những người khác. Tôi không có một kỹ thuật nội tại cho việc nghiên cứu.


Q: Ông có nhắc đến các bài báo đã bị quên lãng. Ông nghĩ có bao nhiêu phần trăm các bài báo được công bố sẽ còn tiếp tục tồn tại?


A: Một phần trăm khác không, tôi tin là vậy. Sau cùng, chúng ta vẫn đọc những bài báo thú vị của Hurwitz, hoặc Eisenstein, hay thậm chí Gauss.


Q: Ông có bao giờ nghĩ rằng mình sẽ hứng thú với lịch sử toán học?

 

A: Tôi vốn đã hứng thú rồi. Nhưng nó không thật sự dễ dàng; tôi không có khả năng ngôn ngữ trong việc học tiếng Latin hay Hy Lạp chẳng hạn. Và tôi thấy rằng việc viết một bài báo về lịch sử toán học còn tốn thời gian hơn viết một bài báo về chính toán. Dù sao thì lịch sử cũng rất thú vị; nó gom mọi thứ vào một góc nhìn ngay ngắn.


Q: Ông có tin người ta sẽ giải quyết được bài toán phân loại nhóm đơn? 


A: Ít nhiều - nhưng nhiều hơn ít. Tôi sẽ rất thích thú nếu một nhóm sporadic mới được tìm ra, nhưng tôi e rằng điều đó sẽ không xảy ra.


Quan trọng hơn, định lý phân loại là một cái gì đó lộng lẫy. Anh có thể kiểm tra rất nhiều tính chất chỉ bằng việc kiểm qua một loạt các nhóm (ví dụ điển hình: phân loại các nhóm $n$-truyền dẫn, với $n$ lớn hoặc hoặc bằng $4$).

 

Q: Ông nghĩ rằng mọi thứ sẽ thay đổi thế nào sau bài toán phân loại nhóm đơn?


A: Anh đang ám chỉ rằng một số nhà lý thuyết nhóm hữu hạn sẽ bị mất tinh thần bởi bài toán phân loại; họ nói rằng (và tôi được nói rằng) “chẳng có gì mà làm sau đó nữa”. Tôi nghĩ điều này thật nực cười. Chắc chắn rằng vẫn còn nhiều thứ để làm! Đầu tiên là đơn giản hoá chứng minh (cái mà Gorenstein gọi là “tái tầm nhìn luận” (revisionism)). Nhưng cũng còn phải tìm ứng dụng trong các phần khác của toán học; ví dụ, đã có rất nhiều khám phá gây tò mò liên hệ nhóm quỷ Griess-Fischer với các dạng modular (cái được gọi là “Moonshine”). 


Hỏi thế không khác gì hỏi rằng liệu chứng minh của Falting cho giả thuyết Mordell có làm lý thuyết điểm hữu tỷ trên đường cong chết đi không. Không! Nó chỉ đơn thuần là điểm bắt đầu. Rất nhiều câu hỏi vẫn bỏ ngỏ.


(nhưng cũng cần phải nói rằng có những lý thuyết có thể bị giết. Một ví dụ nổi tiếng là vấn đề thứ mười lăm của Hilbert: mọi nhóm tôpô Euclid địa phương là một nhóm Lie. Khi tôi vẫn còn là một nhà tôpô trẻ, đó là vấn đề mà tôi rất muốn giải quyết - nhưng tôi đã chẳng đi đến đâu. Gleason, và Montgomery-Zippin là những người giải quyết nó, và lời giải của họ tất cả không là gì ngoài việc giết chết vấn đề. Còn gì để khám phá trong hướng này? Tôi có thể nghĩ tới một câu hỏi: liệu nhóm các số nguyên p-adic có tác động effective lên một đa tạp? Câu hỏi này dường như khá khó - nhưng ngay cả thế lời giải của nó sẽ còn không có chút ứng dụng nào theo tôi thấy.) 


Q: Nhưng người ta có thể nói rằng hầu hết các vấn đề trong toán học đều như vậy, tức là vấn đề tự chúng có thể khó và thách thức nhưng sau khi có lời giải thì chúng trở nên vô dụng. Thực tế có rất ít vấn đề như giả thuyết Riemann khi mà thậm chí trước cả lời giải của nó, người ta đã biết tới rất nhiều hệ quả. 


A: Đúng, giả thuyết Riemann là một trường hợp rất tốt: nó suy ra rất nhiều điều (bao gồm các bất đẳng thức thuần dữ liệu số, ví dụ về biệt thức của các trường số). Nhưng vẫn còn các ví dụ khác: định lý giải kỳ dị của Hironaka chẳng hạn; hay bài toán phân loại nhóm đơn mà chúng ta vừa bàn tới.


Đôi khi phương pháp sử dụng trong chứng minh là thứ có nhiều ứng dụng: tôi tự tin mà nói rằng điều này đúng với Falting. Và đôi khi, đúng là vấn đề tự nó không có ứng dụng; chúng là một dạng bài kiểm tra có những lý thuyết đang tồn tại; chúng ép chúng ta đi xa hơn nữa.


Q: Ông có còn quay lại với các vấn đề trong tôpô không?


A: Không. Tôi đã không dõi theo các kỹ thuật gần đây, và tôi không biết những tính toán cuối cùng của nhóm đồng luân của mặt cầu $\pi_{k+n}(S^n)$ (tôi đoán người ta đã chạm tới mức $k=40$ hoặc $50$. Tôi từng biết chúng tới khoảng $k=10$.)


Nhưng tôi vẫn sử dụng các ý tưởng từ tôpô theo nghĩa rộng, ví dụ đối đồng điều, cản trở, lớp đặc trưng Stiefel-Whitney, vân vân.


Q: Bourbaki đã ảnh hưởng gì lên toán học?


A: Một ảnh hưởng tốt. Tôi biết rằng trách mắng Bourbaki vì mọi thứ là một thứ mốt thời thượng (ví dụ “New Math”), nhưng điều đó không công bằng. Người ta đã lạm dụng những quyển sách của Bourbaki; chúng vốn không được dùng cho giảng dạy đại học, thậm chí còn ít hơn trong việc giảng dạy trung học. 


Q: Liệu có nên có một dấu hiệu cảnh báo?


A: Một dấu hiệu cảnh báo thực tế đã được đưa ra bởi Bourbaki: đó là seminar Bourbaki. Seminar đó không hình thức như những gì trong sách: nó bao gồm tất cả các thể loại toán học, thậm chí một chút vật lý. Nếu anh gộp cả seminar và sách thì anh sẽ có một góc nhìn cân bằng hơn rất nhiều.


Q: Ông có thấy ảnh hưởng của Bourbaki lên toán học đang đi xuống?


A: Ảnh hưởng là khác so với nó đã từng. Bốn mươi năm trước, Bourbaki có một nhiệm vụ để làm; đó là chứng minh rằng một thứ toán học hệ thống và trình bày tốt là khả thi. Và bây giờ nhiệm vụ hoàn thành rồi, Bourbaki chiến thắng. Như một hệ quả, những quyển sách của anh ta chỉ còn dùng cho sở thích kỹ thuật; câu hỏi chỉ là liệu chúng có trình bày tốt về chủ đề người ta quan tâm hay không. Đôi khi là có (quyển về “root systems” đã trở thành tham khảo chính quy cho ngành) ( :D  hồi thi môn lý thuyết nhóm Lie mình đã phải tham khảo ở đây); đôi khi là không (tôi sẽ không đưa ra ví dụ: nó là vấn đề thị hiếu).


Q: Nói về thị hiếu, phong cách nào (cho sách, báo) mà ông thích nhất?


A: Sự chính xác kết hợp với tính không hình thức! Cái đó là lý tưởng, như cho các bài giảng. Anh có thể tìm thấy sự hoà quyện niềm vui ở các tác giả như Atiyah hay Milnor, và một số ít khác. Nhưng rất khó để đạt tới mức này. Ví dụ, tôi thấy rất nhiều nhà toán học Pháp (bao gồm cả tôi) làm mọi thứ hơi hình thức, và một số nhà toán học Nga lại quá thiếu chính xác…


Một điểm nữa tôi muốn đề cập các bài báo nên bao gồm nhiều hơn những lưu ý, câu hỏi mở, vân vân. Rất thường xuyên, có những thứ thú vị hơn những định lý thực sự được chúng minh. Than ôi, hầu hết mọi người sợ thừa nhận rằng họ không biết câu trả lời cho một câu hỏi, và như một hệ quả họ tránh luôn việc nói tới câu hỏi, ngay cả khi nó là một câu hỏi rất tự nhiên. Thương xót làm sao! Như tôi vẫn hay làm, tôi thích nói “tôi không biết”.

 

Dịch bởi: Phạm Khoa Bằng, Đại học Rennes 1, Pháp.

Nguồn: xem tại đây.


 




#744467 Phỏng vấn với Jean-Pierre Serre

Posted by bangbang1412 on 01-04-2024 - 21:09 in Kinh nghiệm học toán

Bài này vừa lên Pi số mới nhất

 

Screenshot 2024-04-01 at 16-01-30 Pi3_diendantoanhoc.pdf.png




#737358 Lý thuyết phạm trù vô cực mang lại tầm nhìn “từ trên xuống” cho toán học

Posted by bangbang1412 on 20-02-2023 - 21:53 in Toán học lý thú

Định bụng cày hết background trước, nhưng theo như trích đoạn ở trên thì có vẻ là học category theory trước sẽ thuận lợi hơn để học nhiều mảng khác nhau của Toán? Vậy thì chính xác lúc nào thì nên học category theory nhỉ? Ví dụ, nên học category trước hay algebraic topology trước? Nếu đi từ undergraduate lên chẳng hạn, thì cần học những gì trước category theory?

Lý thuyết phạm trù (category theory) là một ngôn ngữ mang tính hình thức ở mức bắt đầu (ở các trình độ cao hơn nó mới phát triển thành ngành nghiên cứu), do đó sẽ không hợp lý lắm nếu chỉ mở sách ra và đọc các định nghĩa. Ngay bản thân tên gọi theory (vẫn ở mức bắt đầu) cũng không đúng lắm, nó không thực sự là lý thuyết mà có thể nói là một cách gom các cấu trúc toán học khác nhau vào cùng một ngôn ngữ hình thức (qua các biểu đồ chẳng hạn). Với lý do đó anh nên bắt đầu với tôpô đại số (algebraic topology), vì nó là một trong các gốc gác đầu tiên cho lý thuyết phạm trù. Thực chất lý thuyết phạm trù "không cần" học, người ta tiếp thu nó rất tự nhiên qua việc biết rất nhiều ví dụ khác nhau. Cách tốt nhất để học nó là biết càng nhiều ví dụ càng tốt và nhìn được nhiều điểm khác nhau. Ví dụ như đồng cấu nhóm, đồng cấu vành, đồng cấu trường (group homomorphism, ring homomorphism, field homomorphism) thì cùng là các cấu xạ (morphism) trong các phạm trù nhóm, phạm trù vành, phạm trù trường, mỗi khi ta thêm cấu trúc vào vật (object) trong phạm trù thì ta thêm điều kiện vào cấu xạ.

 

Nếu chỉ ngồi học định nghĩa không của lý thuyết phạm trù, thì không nắm được bản chất của gì cả, còn gọi là abstract nonsense. Từ level undergraduate thì chỉ cần biết những thứ như nhóm, vành, trường, module, đại số (những thứ như hình học cũng tốt) và sau đó đọc thử đại số đồng điều là một cách để thực sự "sờ nắn" lý thuyết phạm trù.




#737380 Lý thuyết phạm trù vô cực mang lại tầm nhìn “từ trên xuống” cho toán học

Posted by bangbang1412 on 21-02-2023 - 21:11 in Toán học lý thú

Ví dụ, một đính lý trong một mảng này có thể được "dịch" sang một mảng khác thông qua category, và như vậy ta có thể thu được một định lý mới (hoặc một định lý đã có sẵn, nhưng xem như cách chứng minh là mới, thông qua category theory và kết quả của mảng kia). Đã có trường hợp nào mà category theory giúp phát hiện ra định lý mới như vậy chưa nhỉ? Anh nghĩ chắc là phải có chứ?

Nếu học category theory, thì anh trông đợi là sẽ được học những ví dụ hay làm những bài tập tương tự như vậy. Còn nếu chỉ có định nghĩa, thì học category theory để làm gì? 

 

Đến đây làm nhớ tới một câu mà Dieudonné nói với Grothendieck, đại loại "il ne faut pas généraliser pour le plaisir de généraliser". Cụm từ "abstract nonsense" chắc cũng xuất phát từ việc cho rằng category theory chỉ để trừu tượng hoá lên mọi thứ chứ chẳng dùng được làm gì. Nhưng ngày nay thì đã rất rõ ràng rằng điều đó không đúng (và lưu ý rằng "abstract nonsense" ngày nay được mọi người dùng với nghĩa tích cực chứ không phải tiêu cực).

 

Anh đang hiểu theo cách em đã cảnh báo ở trên, em nói rằng chữ theory trong category theory không có nghĩa là một theory như các ngành cụ thể, mà ở sơ khởi nó chỉ thuần tuý là một cách diễn đạt nhiều khái niệm trong toán học dưới một ngôn ngữ hình thức và cho gọn, cho đẹp hơn. Một khi người ta học đủ họ sẽ tự nhận ra sự tương đồng giữa các cách viết định nghĩa, lập luận trong các ngành, và cần thiết để có một cách trình bày gọn gẽ hơn, đó là cái lý thuyết phạm trù làm. Nhưng Không có việc chuyển dịch một định lý từ cái này sang cái kia, em có thể nói cụm từ abstract nonsense cá nhân em không dùng nó với nghĩa tiêu cực nhưng cũng không tích cực. Nếu ai cũng chỉ học category theory xong vẽ ra mấy cái biểu đồ thì như anh Nxb bảo thì ai cũng làm toán được hết. Bản chất vẫn là nội tại của ngành mình làm.

 

Nói thế không có nghĩa là có thể coi thường cái cách lý thuyết phạm trù làm người ta viết mọi thứ chuẩn chỉ hơn, như Brian Conrad nói before functoriality, people still live in caves. Lý thuyết phạm trù, ngoài việc giúp trình bày còn mang lại cái philosophies. Ví dụ điển hình là quan điển functor of points (hàm tử điểm) của Grothendieck hay gọi là relative point of view, hay gì cũng được, xuất phát từ bổ đề Yoneda. Bổ đề Yoneda nói rằng một vật trong một phạm trù được xác định nếu ta biết tất cả các cấu xạ đi vào vật đó (thế nên Serre mới viết ...comme Grothendieck nous l'a appris, les objets d'une catégorie ne jouent pas un grand rôle, ce sont les morphismes qui sont essentiels.). Hoặc một cái khác là tính phổ dụng (universal property) và một thứ quan trọng nữa lý thuyết phạm trù dạy cho ta, đó là xây dựng cái gì thì cũng phải có tính hàm tử (functoriality). Em sẽ để ra vài ví dụ:

  • Đầu tiên, và kinh điển nhất, là đối ngẫu của không gian vector $V$ trên $\mathbf{R}$ (bất cứ trường nào cũng được): đối ngẫu một lần $V^{\vee}$ thì đẳng cấu với $V$ (vì có cùng số chiều). Nhưng bất biến về số chiều thì quá thô, đẳng cấu này dựa vào việc chọn một cơ sở của $V$, có rất nhiều cơ sở, làm sao mà biết cái nào chuẩn nhất? Tuy nhiên đối ngẫu hai lần $V^{\vee \vee}$ thì tự nhiên, tồn tại một phép xây dựng $V \longmapsto V^{\vee \vee}$ tự nhiên theo nghĩa cứ có một đồng cấu không gian vector $V \longrightarrow W$ thì có một đồng cấu $V^{\vee \vee} \longrightarrow W^{\vee \vee}$. Trong trường hợp đối ngẫu một lần, phép hợp thành cũng cho ta $W^{\vee} \longrightarrow V^{\vee}$, nhưng do phụ thuộc vào cơ sở, nó không tương thích với phép xây dựng $V \longmapsto V^{\vee}$.
  • Cho $G$ là một nhóm, làm thế nào để tạo ra một nhóm giao hoán (commutative group, hoặc abelian group) từ $G$? Có một cách, lấy tâm của $$Z(G) = \left \{g \in G \mid gx = xg \forall x \in G \right \}$$ Tuy nhiên cách làm này có quá nhiều bất lợi, tâm của một nhóm có thể rất bé (ví dụ tâm của nhóm ma trận của là bội của ma trận đơn vị), và quan trọng hơn, nếu có một đồng cấu nhóm $G \longrightarrow H$, ta không có một đồng cấu nhóm giao hoán $Z(G) \longrightarrow Z(H)$. May mắn, ta có một cách khác, là xét abel-hoá $G^{ab}$ của $G$, tức là nhóm thương $G/[G,G]$ trong đó $[G,G]$ là nhóm con các giao hoán tử, i.e. sinh bởi các phần tử $aba^{-1}b^{-1}$. Xây dựng này thì có tính hàm tử nên nó tốt. Nó còn tốt hơn nữa do nó là đảo ngược quá trình ta đưa một nhóm abel thành một nhóm. Khi anh có một nhóm abel $G$, anh có thể xem nó như một nhóm (mà quên mất tính giao hoán), quá trình này gọi là quên (forgetful functor). Khi anh xem xét một đồng cấu $G \longrightarrow H$ với $H$ abel, $G$ bất kỳ, thực chất anh đã quên mất tính abel của $H$ (do đồng cấu của nhóm abel cũng chỉ là đồng cấu nhóm). Làm thế nào để không mất thông tin? Đó là với bất kỳ đồng cấu nào thế kia, nó đều tách thành hợp thành $G \longrightarrow G^{ab} = G/[G,G] \longrightarrow H$, tức là tách qua nhóm abel hoá của $G$, và thực chất ta đang làm với các nhóm abel. Rồi bấy giờ chúng ta hãy thử chứng minh $(G \times H)^{ab} \cong G^{ab} \times H^{ab}$. Đây là một bài tập đơn giản, nhưng không khai sáng lắm nếu ta viết ra cụ thể một đẳng cấu. Tính phổ dụng cho ta một chứng minh độc đáo hơn.
  • Nếu chưa thoả mãn với ví dụ này, hãy xét tiếp với tích tensor $V \otimes_{\mathbf{R}} W$ của hai không gian vector $V,W$ trên $\mathbf{R}$ chẳng hạn. Tích tensor trong đại số tuyến tính dù được xây dựng tường minh nhưng tính phổ dụng mới là cái độc đáo của nó. Nó nói rằng phép xây dựng tensor là cách ta "đơn" tuyến tính hoá một cái gì đó song tuyến tính. Rất nhiều chứng minh ví dụ như kiểu $V \otimes W \cong W \otimes V$ mà viết cụ thể ra thì rất mệt, tính phổ dụng cho tư duy gọn gàng hơn, và chứng minh khai sáng hơn. Tóm lại ta "không" nhớ về vật, mà ta nhớ về cấu xạ giữa vật. Ta cũng "không" nhớ về xây dựng, mà ta nhớ tính phổ dụng (tức là mục đích của nó là gì). Theo nghĩa này, lý thuyết phạm trù là một cách thay đổi tư duy: lý thuyết phạm trù là cách thể hiện mối tương quan giữa các đối tượng và là chọn ra một xây dựng tự nhiên nhất.
  • Đây là một "phản ví dụ" cho cái mà anh muốn hỏi: liệu có thể chứng minh một định lý ở lĩnh vực này và áp dụng category theory để translate sang lĩnh vực khác không? Câu trả lời là , nhưng không đơn giản như thế. Nguyên lý GAGA (Géométrie algébrique et géométrie analytique) của J. P. Serre nói rằng một một tương ứng $1-1$ giữa các đa tạp xạ ảnh đại số xạ ảnh phức (projective complex algebraic variety) và các không gian giải tích compact (compact analytic space). Nói khác đi, một bài toán về đa tạp đại số xạ ảnh trên trường $\mathbf{C}$ có thể nghiên cứu bằng công cụ giải tích phức và ngược lại. Nói tổng quát hơn, hình học đại số trên $\mathbf{C}$ dù làm bằng đại số hay giải tích thì cũng như nhau! Nhưng đây không phải là lý thuyết phạm trù, nếu anh đọc chứng minh của nguyên lý này, hoàn toàn không có gì là phạm trù mà chỉ là đại số giao hoán và giải tích phức. Phạm trù chỉ là cách viết $X$ (đa tạp đại số) biến thành $X^{an}$ (không gian giải tích) thì là một tương đương phạm trù (equivalence of categories).
  • Một cái tương đương khác như vậy gọi là tương ứng Dold-Kan, nó nói rằng phạm trù các nhóm abel đơn hình (simplicial abelian groups) và phạm trù các phức (complexes) là tương đương nhau. Tức là dù anh làm với mô hình đại số (phức) hay mô hình tổ hợp (simplicial objects) thì cũng như nhau. Nhưng đọc chứng minh thì hoàn toàn không có gì gọi là thuần tuý phạm trù, toàn là dãy phổ (spectral sequences) với đại số đồng điều (homological algebra). Ở đây còn có một cầu nối rất sâu sắc giữa hai kiểu mô hình này, đó là chúng cùng là các phạm trù mô hình (model categories) riêng, sinh bởi đối phân thớ (proper + cofibrantly generated model categories). Ở đây mới bắt đầu có thể nói phạm trù (hoặc infinity category mà anh Nxb làm) thực sự là một ngành nghiên cứu. 
  • Một ví dụ khác, cao cấp hơn nữa cho những ai quan tâm về việc phạm trù có thể giúp định hình tư duy như thế nào và tại sao lại nên có một phạm trù tốt. Tức là rất nhiều xây dựng trong các ngành là "cái bóng" (shadow) của một xây dựng nào đó "bên trên": đối đồng điều là cái bóng của motives hoặc lý thuyết về các derivator trong bản thảo 2000 trang của Grothendieck (the theory of derivators). Cái này không phải là em nói về lý thuyết derivator như một chuyên gia, mà như tư cách một người học, tức là có trải nghiệm cụ thể với lý thuyết. Lý thuyết về derivator có thể coi là cái "motive" thật sự (in some sense, of course!) của hình thức luận sáu hàm tử (six functors formalism). Mục đích ban đầu của Grothendieck khi đề xuất nó là do xây dựng trụ (cone) của phạm trù tam giác (triangulated categories) không có tính hàm tử (nhiều người vì lý do này tin rằng phạm trù tam giác vẫn chưa phải là xây dựng tốt, cũng như người ta tin đối đồng điều étale thì vẫn chưa phải là đối đồng điều "đúng" cho đa tạp đại số; xin nói thêm hơi dài chỗ này một chút: cái Grothendieck muốn cho lý thuyết motive là tìm được phạm trù motives trộn - aka category of mixed motives mà từ đó các đối đồng điều đều là cái bóng của các vật trong phạm trù này. Cái đúng theo philosophy của Grothendieck là phạm trù đúng, đúng tức là tốt, chứ không phải một đối tượng cụ thể tốt (một phạm trù tốt mà vật xấu thì còn tốt hơn một phạm trù xấu mà vật tốt - có thể nói vậy). Ví dụ một quá trình $$D^b(X_{\overline{k}},\mathbf{Q}_l) \longmapsto H^{\bullet}_{et}(X_{\overline{k}};\mathbf{Q}_l) \longmapsto \zeta_X(T)$$ là đi từ categorical-level invariant thu được set-level invariant và lấy toàn tử Frobenius thu được element-level invariant), tức là từ phạm trù, được bất biến đại số, và được hàm. Derivators là chỗ mà ta làm việc "rất hình thức" nhưng các xây dựng thì lại tổng quát (mà không hề nonsense) tới mức có thể đặc biệt hoá (chiếu xuống) các xây dựng cụ thể tới một cách bất ngờ. Ví dụ các định lý đổi cơ sở (Bech-Chevalley), dãy địa phương của một cặp bù trừ lược đồ đòng mở,... đều có thể viết thành ngôn ngữ phạm trù hết. Khi em học về các xây dựng của lý thuyết phạm trù mô hình (nhất là tính tam giác hoá của phạm trù đồng luân của một phạm trù định điểm) thì rất bối rối vì dù các chứng minh cũng không quá khó (lấy động lực từ tôpô cả, nhưng mà vẫn khó, phần nữa phong cách Pháp họ còn không viết nhiều ví dụ) nhưng vẫn rất loằng ngoằng. Lý thuyết derivator là một cách định hình hoá lại cái mindset của em. Cái cone construction thực chất ban đầu nằm trong phạm trù gốc, nhưng thực chất lý thuyết derivator nói nó nên nằm ở "một phạm trù cao hơn". Tìm kiếm các phạm trù ngày nay không còn là công việc xa lạ với những người làm hình học đại số.

Tặng anh và anh Nxb cái meme em làm:

chỉ mục.jpg

 

Cuối cùng là tự pr, hồi mới học tô-pô đại số em có viết cái note này để tổng hợp các ví dụ. Như anh có thể thấy, sau mỗi khái niệm là một loạt các ví dụ. :D Nên em nghĩ là nếu anh muốn học thì có hai cách: lao vào tô-pô đại số hoặc như anh Nxb bảo, kiếm cái gì đó ứng dụng vì giờ người ta cũng mang nó đi nhiều nơi mà. Ngay cả mấy ông triết chủng cũng còn học phạm trù cơ mà. 




#735490 Toán học như văn hóa và tri thức

Posted by bangbang1412 on 27-10-2022 - 22:28 in Toán học lý thú

Toán học như văn hóa và tri thức - Mathematics as Culture and Knowledge

 

Toán học là một hoạt động tri thức, được cho là một trong những hoạt động tinh tế nhất từng được tạo ra bởi văn minh nhân loại. Hermann Hesse phác họa chân dung những hoạt động của các nhà toán học một cách ẩn dụ trong Glass Bead Game. Có lẽ đó là nỗ lực văn học tốt nhất để bắt dù chỉ một cái nhìn thoáng qua những hoạt động nội tại trong xã hội toán học. Người ta không phê phán một tác phẩm hư cấu bằng sự thiếu chính xác của nó, nhưng sẽ thực sự khó để nói cái gì đó có nghĩa về việc thế nào là làm toán.

Có khá nhiều các nhà toán học thừa hưởng quan điểm kiểu Plato về toán học. Điều này có nghĩa là họ có niềm tin rằng các đối tượng và xây dựng toán học có một kiểu tồn tại nào đó trong "thế giới của những ý tưởng", tồn tại độc lập với trí óc con người. Như trong trường hợp của thiên đường thần thoại, những người khởi xướng niềm tin đó tỏ ra khá mập mờ về vị trí và tính nhất quán của thế giới Plato ngoại lai này. Một lý do thường được viện ra để củng cố góc nhìn Plato là sự hiệu quả của toán học trong việc mô hình hóa thế giới vật lý. Không nghi ngờ gì những định luật Kepler cuối cùng cũng có thể được quan sát và thông hiểu bởi bất kì trí thông minh công nghệ nào sống trên một hình tinh bao quanh bởi lực hấp dẫn để quay quanh một ngôi sao (nhưng liệu một khám phá như vậy có tuân theo tiến trình mà chúng ta biết, hành tinh có xoay quanh hai ngôi sao không?). Tuy nhiên người ta khó có thể viện ra một trường hợp mạnh mẽ như thế để mà củng cố ý tưởng về cái đẹp trong các nhánh toán học khác trừu tượng hơn rất nhiều.

Nếu không ai có thể nghi ngờ rằng bất kỳ trí thông minh ngoài trái đất nào được tiến hóa đủ sẽ hiểu được khái niệm về số nguyên tố, thì sẽ có bằng chứng kém thuyết phục hơn nhiều rằng chúng sẽ có những khái niệm giống chúng ta về các phạm trù dẫn suất (derived categories) hoặc shtukas (chú thích: Drinfeld mô-đun suy rộng). Những năm gần đây chúng ta đã phải dùng đến những loại toán học tinh vi hơn và hơn nữa, chúng được đưa vào vì sự phát triển ngày càng phức tạp của vật lý năng lượng cao. Mặc cho kiểu viện dẫn này, tôi vẫn cực kì hoài nghi về giả thuyết của chủ nghĩa Plato.

Bộ não chúng ta đã phát triển qua hàng triệu năm tiến hóa có chọn lọc. Năng lực chế tạo toán học có một lợi ích tiến hóa rõ ràng vì nó là chìa khóa cho một nền văn minh khoa học và công nghệ. Địa vị nổi bật mà loài vượn này đã chiếm được, trong so sánh với các loài động vật khác trên trái đất, hiển nhiên là bằng chứng về lợi ích tiến hóa của khả năng não bộ phục vụ cho các hoạt động khoa học.

Các kiểu não bộ khác mà là sản phẩm của một quá trình tiến hóa hoàn toàn khác biệt trong một môi trường hoàn toàn khác biệt cũng có thể đạt được cùng một kết quả trong tiến bộ công nghệ trong khi sáng tạo ra một kiểu toán học có khác biệt đáng kể với thứ toán học mà chúng ta biết. Không hoàn toàn khác, chắc chắn (các số nguyện tố), nhưng là một sự khác biệt đối xứng to lớn. Sự tồn tại của trí thông minh ngoài trái đất hoàn toàn mang tính giả thuyết. Sagan và Shklovskii đã suy đoán rất hay về nó trong những năm 70 và tôi sẽ để tất cả ở đó, chủ nghĩa Plato và những thứ đó.

Nếu toán học (ít nhất là một phần lớn toán học) chẳng phải một dấu hiệu của thiên đường chủ nghĩa Plato mà chỉ là một đơn thuần là sản phẩm của não bộ và quá trình tiến hóa thì nó cũng chẳng mất đi tý vẻ đẹp nào. Nó còn trở nên thú vị hơn vì nó là một phần của văn hóa con người, và nó đi cùng và chịu ảnh hưởng của sự phát triển của toàn bộ những gì còn lại của văn minh.

Toán học mà chúng ta biết ngày nay là kết quả của một hành trình phát triển văn hóa dài và quanh co. Tuy nhiên, nó còn lâu mới là một tòa lâu đài bất động. Sự liên tục của nó, sự tiến hóa mau chóng có thể nhìn thấy dễ dàng bằng cách nhìn vào một số thống kê quan trọng. MathSciNet, nguồn review chính của các công trình toán học, liệt kê ra tổng cộng 2,245,194 công trình, và tăng thêm 60,000 mỗi năm (và những gì liệt kê bởi MathSciNet chỉ là một tuyển chọn trên tổng số những công trình toán học).

Bước quan trọng cho bất cứ ai hứng thú trong việc làm toán là ý thức về sự to lớn trong địa hạt này. Một rủi ro chính, theo ý tôi, trong toán học và bất kì lĩnh vực nào của tri thức con người, là trở nên ngây thơ. Người ta không tự nhận mình là nhà toán học. Trở thành một nhà toán học đòi hỏi ít nhất mười năm tu tập chuyên sâu và học hành cẩn thận. Cái đó mới chỉ là để tích lũy một lượng tối thiểu kiến thức và kĩ năng cần thiết để hiểu làm toán là như thế nào. Để bắt đầu thực sự làm cái gì đó trong toán học đòi hỏi một vài bước sau đó nữa.

Một thứ cực kì khó để tiếp thu, và là một dấu hiệu tốt để trở thành một nhà toán học trưởng thành chuyên nghiệp là khả năng đánh hơi ra cái gì thú vị. Có rất nhiều thứ trong toán học mà người ra có thể làm chỉ để làm, Marcel Duchamp đã đặt tên cho một tác phẩm điêu khắc đầy khiêu khích của ông ta "phân loại lược theo số lượng răng".

Thứ toán học thực sự thú vị không phải là một bài tập phân loại lược. Cái thường làm một kết quả toán học bất ngờ và thú vị nằm trong khả năng khám phá ra những kết nối không ngờ tới: một cách liên hệ kết quả và xây dựng mà ban đầu tỏ ra chẳng liên quan, nhận ra sự tương tự trong cấu trúc thông qua những hiện tượng khác biệt rõ ràng.

Ngây thơ trong toán học (với những ngoại lệ hiếm hoi) có một tác động đơn thuần là cắm đầu vào một góc tù mù của một trò chơi vô ích. Kiến thức là những gì cung cấp những ngọn hải đăng và hải đồ quan trọng cho phép các nhà toán học đang hoạt động định hướng đường đi của họ một cách an toàn trong khi băng qua vùng biển động.

Có những huyền thoại lãng mạn được lan truyền rộng rãi kiểu như những thiên tài cô đơn chẳng đọc điếc gì mà vẫn xổ ra được những định lý đẹp đẽ. Những huyền thoại này phần lớn dựa trên các giai thoại bịa đặt. Thực tế, một thời gian dài đọc và hấp thu tri thức toán học của quá khứ và hiện tại là tối hậu trong việc tạo ra thứ toán học thú vị trong tương lai. Cô lập chỉ đơn giản là cạn kiệt khả năng sáng tạo.

Ngoài sự hiệu quả của nó như một chất xúc tác cho sáng chế, việc truyền tải kiến thức thông qua chữ viết là thứ tạo nên con người chúng ta. Nó là chìa khóa cho tiến bộ của văn minh. Chúng ta đọc và học bởi vì chúng ta tìm thấy niềm vui khi làm thế, vì chúng ta là những tồn tại người quan tâm tới tồn tại không chỉ như mảnh vụn cô lập mà là một phần của nhân loại như một thể thống nhất. Như trong thơ nổi tiếng của John Donne, "không có người nào là một hòn đảo riêng, chỉ mình nó với nó; mọi con người là một mẩu của lục địa, một phần của cái chính yếu."

Toán học là thú vị với tự cách một mức độ cực rất cao giữa những thành tựu của nhân loại, bởi vì nó có tính phổ quát có thể cho chúng ta cách bắc cầu và vượt qua những khác biệt không đáng kể về địa lý và lịch sử đã chia rẽ loài người. Nó là ngôn ngữ chung mà bộ não chúng ta đã tạo ra, thứ ngôn ngữ chèo lái tiến bộ khoa học và công nghệ và đồng thời là một nỗ lực nghệ thuật có tính triết lý sâu sắc.

 

Screenshot 2022-10-27 at 22-20-29 The Unravelers Mathematical Snapshots - Jean-Francois Dars - The Unravelers_ Mathematical Snapshots-AK Peters (2008).pdf.png

 

Thật sự, có một khía cạnh đặc biệt của toán học làm nó tách biệt với những lĩnh vực tri thức khác của con người. Nó hoạt động đồng thời dưới tư cách của một khoa học chính xác và cũng dưới tư cách một nghệ thuật. Trí tưởng tượng bay bổng, hình ảnh thơ mộng và trực quan cùng những cân nhắc thẩm mỹ thúc đẩy sự phát triển của toán học và sống kề cạnh với những quy luật nghiêm ngặt nhất của khoa học.

Thật đáng thương khi các nhà khoa học thần kinh cố gắng hiểu làm thế nào não bộ phát triển toán học nói chung, họ thường tỏ ra nhầm lẫn toán học với "cảm giác số" (tạm dịch từ number sense). Cái thứ hai là một khoa tri thức rất khác biệt, vốn hoàn toàn tách rời khỏi toán học (có hằng tá ví dụ về những nhà toán học nổi tiếng mà chẳng tý cảm giác số nào). Toán học có nghĩa là tạo ra các cấu trúc và nói riêng, những con số tỏ ra là một cấu trúc thú vị, nhưng nhưng điều đó là khá xa khi kết nối với toán học nói chung.

Cố gắng hiểu toán học được tạo ra trong não bộ như thế nào sẽ là một cách tuyệt vời để khám phá ra nhiều hơn nữa những chức năng của não bộ tự nó, vì nó cung cấp một phổ các cách thức vận hành của sự sáng tạo và tưởng tượng cũng như sự vận dụng hình ảnh và kí hiệu, với một sự chú tâm được xác định rõ ràng và chính xác.

Câu trả lời cuối cùng, nếu ai đó cần, cho câu hỏi là tại sao chúng ta làm toán, là do chúng ta tìm thấy niềm vui khi làm vậy. Nó là một phụ phẩm của tiến hóa bằng chọn lọc tự nhiên mà chúng ta chiết xuất ra sự vui thú từ việc làm những thứ có lợi cho sự sinh tồn của bộ gene chúng ta. Toán học có lợi cho giống loài chúng ta bởi vì những ứng dụng nó mang đến cho khoa học và công nghệ, nhưng đó không phải lý do chúng ta làm toán. Chúng ta không nghĩ về sự quan trọng của nó trong ứng dụng thực tiễn khi chúng ta thích thú sáng tạo những thứ toán học mới, cũng như chúng ta không nghĩ về tầm quan trọng của việc trộn lẫn DNA khi làm tình.

 


                                                                                                                                                  Tác giả: Matilde Marcolli, Max-Planck-Institut für Mathematik, Bonn.

                                                                                                                                                                                Dịch: Phạm Khoa Bằng, Université de Rennes 1.




#735491 Toán học như văn hóa và tri thức

Posted by bangbang1412 on 27-10-2022 - 22:57 in Toán học lý thú

Beyond Numbers

 

Rất thường xuyên các nhà toán học của chúng ta nhìn đồng nghiệp của mình trong các lĩnh vực khác với một thái độ khinh bỉ - gã này có thể tìm thấy cái kiểu niềm vui biến thái nào trong cái lĩnh vực không có động lực và nhàm chán của hắn? Tôi đã cố học cách tìm ra những vẻ đẹp trong rất nhiều lĩnh vực, nhưng vẫn còn đó rất nhiều ngành mà sự hấp dẫn đối với tôi vẫn là cái gì đó hoàn toàn bí ẩn.

 

Lý thuyết của tôi là con người thường xuyên phóng chiếu cái yếu đuối kiếp người của họ lên hoạt động toán học của họ.

Có những ví dụ bề nổi hiển nhiên: ví dụ, ý tưởng phân loại một kiểu đối tượng nào đó là hiện thân của bản năng thu thập, tìm kiếm giá trị lớn nhất là một dạng khác của sự tham lam, khả năng tính toán/khả năng quyết định đúng sai đến từ ham muốn kiểm soát toàn bộ.

 

Screenshot 2022-10-27 at 22-51-38 The Unravelers Mathematical Snapshots - Jean-Francois Dars - The Unravelers_ Mathematical Snapshots-AK Peters (2008).pdf.png

Đam mê với sự lặp lại thì tương tự như thôi miên của nhịp điệu âm nhạc. Dĩ nhiên, kết quả phân loại của những kiểu đối tượng có thể rất hữu ích trong việc phân tích những cấu trúc phức tạp hơn, hoặc có thể nó chỉ để được ghi nhớ trong những trường hợp đơn giản.

Kiếm thức về cực đại đúng hoặc chặn trên của một đại lượng phụ thuộc vào các tham số cho ta một ý tưởng về phạm vi những giá trị khả dĩ. Một lý thuyết tính toán trên thực tế có thể thực dụng với những thí nghiệm máy tính.

Tuy nhiên, đối với tôi thì động lực chủ yếu phải là hiểu được cái cỗ máy chạy ẩn trong những ví dụ nổi bật cụ thể, mà từ đó chúng ta mới có thể xây dựng lý thuyết/hình thức luận.

Nếu một người cố gắng đi xa hơn tới chỗ "loại bỏ tính người" trong toán học, thì một bước tự nhiên tiếp theo là động tới các số thực (phát sinh từ các thuộc tính cơ bản của thế giới vật chất) chỉ như một trường không đóng đại số phức tạp nữa. Theo nghĩa nào đó nó đúng; số phức đẹp hơn rất nhiều.

Nhưng theo một nghĩa khác, các số thực mới thực sự là nền tảng vì chúng là hiện thân của ý tưởng chặn, của sự kiểm soát các cấu trúc đại số trừu tượng. Theo một cách sâu xa nào đó, tất cả chúng ta là các nhà hình học.



Tác giả: Maxim Kontsevich. IHES.

Dịch: Phạm Khoa Bằng, Université de Rennes 1.




#734876 K-lý thuyết Milnor

Posted by bangbang1412 on 09-09-2022 - 14:32 in Toán học hiện đại

Liên hệ lý K-lý thuyết với đối đồng điều motivic

 

Cho $k$ là một trường, trong phần này chúng ta sẽ chứng minh rằng $K^M_n(k)$ đẳng cấu tự nhiên với $H^{n,n}(\mathrm{Spec}(k),\mathbb{Z})$. Cuối cùng ta sẽ giới thiệu liên hệ của K-lý thuyết Milnor với đối đồng điều Galois cũng như phiên bản lược đồ của nó là giả thuyết Milnor-Bloch-Kato chứng minh bởi Voevodsky với đóng góp rất lớn của Markus Rost. Không như giả thuyết Milnor-Bloch-Kato, đẳng cấu $K^M_n(k) \simeq H^{n,n}(\mathrm{Spec}(k),\mathbb{Z})$ không quá khó để chứng minh, nó dựa vào việc cả hai lý thuyết này đều có ánh xạ chuẩn với tính chất giống hệt nhau. Việc còn lại của ta là xây dựng đủ một số chu trình đại số sơ cấp để phục vụ tính toán.

 

Định lý. Giả sử $L/k$ là một mở rộng hữu hạn của trường, khi đó phép đẩy xuôi riêng (proper push-forward) của các chu trình đại số cảm sinh cho ta một ánh xạ chuẩn $N_{L/k}:H^{*,*}(\mathrm{Spec}(L),\mathbb{Z}) \longrightarrow H^{*,*}(\mathrm{Spec}(k),\mathbb{Z})$ thoả mãn các tính chất sau:

  • $N_{L/k}:H^{0,0} \longrightarrow H^{0,0}$ là phép nhân cho $[L:k]$.
  • $N_{L/k}:H^{1,1} = L^{\times} \longrightarrow H^{1,1} = k^{\times}$ là chuẩn thông thường của mở rộng trường.
  • Ta có các công thức chiếu $N_{L/k}(y_L \cdot x) = y \cdot N_{L/k}(x)$ và $N_{L/k}(x \cdot y_L) = N_{L/k}(x) \cdot y$.
  • Nếu $F/L/k$ là một tháp trường sao cho $F/k$ là Galois, khi đó $$N_{L/k}(x)_F = [L:k]_{insep}\sum_{j:L \hookrightarrow F}j^*(x) \in H^{*,*}(\mathrm{Spec}(F),\mathbb{Z}).$$
  • Nếu $F/L/k$ là một tháp trường thì $N_{F/k} = N_{F/L} \circ N_{L/k}.$

Bây giờ theo bổ đề, ta cần tìm một ánh xạ $f$ từ $\mathbb{Z}_{tr}(\mathbb{G}_m^{\wedge n})(\mathrm{Spec}(k))$ vào $K^M_n(k)$ mà hợp thành với hai toán tử biên triệt tiêu. Một ánh xạ như vậy cảm sinh một đồng cấu duy nhất từ $H^{n,n}(k,\mathbb{Z})$ vào $K^M_n(k)$.

 

Ta thấy $\mathbb{Z}_{tr}(\mathbb{G}_m^{\wedge n})(\mathrm{Spec}(k))$ là nhóm abel tự do sinh bởi các điểm đóng của $\mathbb{G}_{m,k}^n$ modulo cho nhóm con sinh bởi các điểm dạng $(x_1,...,1,...,x_n)$ ($1$ có thể ở bất kì vị trí nào). Điều này là do một tương ứng hữu hạn từ $\mathrm{Spec}(k)$ tới một lược đồ chỉ là một điểm đóng, modulo thì do định nghĩa của tích wedge. Nếu $x$ là một điểm đóng của $\mathbb{G}_{m,k}^n = (\mathbb{A}_k^1 - \left \{ 0 \right \})^n$ với trường thặng dư $L$ thì $x$ định nghĩa một điểm $(x_1,...,x_n) \in (L^{\times})^n$. Hơn nữa $L/k$ là mở rộng hữu hạn nên ta có thể dùng đồng cấu chuẩn để xác định một phần tử $N_{L/k}(\left \{x_1,...,x_n \right \}) \in K^M_n(k)$. Ta định nghĩa

$$f(x) = N_{L/k}(\left \{x_1,...,x_n \right \}).$$

Nhưng ta lưu ý rằng $\left \{x_1,...,1,...,x_n \right \} = 0$ trong $K^M_*(L)$ nên $f$ phải tách qua $\mathbb{Z}_{tr}(\mathbb{G}_m^{\wedge n}(\mathrm{Spec}(k))$, cho ta một ánh xạ $f: \mathbb{Z}_{tr}(\mathbb{G}_m^{\wedge n}(\mathrm{Spec}(k)) \longrightarrow K^M_n(k)$. Ánh xạ $f$ hợp thành với $\partial_0 - \partial_1$ triệt tiêu theo bổ đề và do đó cảm sinh một đồng cấu $\theta: H^{n,n}(k,\mathbb{Z}) \longrightarrow K^M_n(k)$.

 

Nếu $x$ là một $k$-điểm của $(\mathbb{A}_k^1 - \left \{0 \right \})^n$ thì toạ độ của nó $(x_1,...,x_n)$ cho ta một điểm trong $(k^{\times})^n$, ta viết $[x_1: \cdots : x_n]$ để chỉ lớp của $x$ trong $H^{n,n}(k,\mathbb{Z})$. Hiển nhiên là $\theta$ toàn cấu do $\theta([x_1:\cdots :x_n]) = \left \{x_1,...,x_n \right \}$ với $x_1,...,x_n \in k^{\times}$.

 

Bổ đề. Với mọi $a_1,...,a_n \in k$ ta có $[a_1:\cdots:a_n]=[a_1]\cdots[a_n].$

 

Giờ ta sẽ xây dựng đồng cấu ngược của $\theta$. Kí hiệu $T(k^{\times})$ bởi đại số tensor. Ta định nghĩa một đồng cấu

$$T(k^{\times}) \longrightarrow \bigoplus_n H^{n,n}(k,\mathbb{Z}),  \ \ \ a_1 \otimes \cdots \otimes a_n \longmapsto [a_1]\cdots [a_n].$$

Phần còn lại của bài viết này sẽ dùng để chứng minh đồng cấu này tách qua $K^M_n(k)$, tức là $[a:1-a]=0$ trong đối đồng điều motivic.

 

Bổ đề. Nếu tồn tại $n>0$ sao cho $[x:1-x]=0$ với mọi mở rộng hữu hạn của $k$ và $x \neq 0,1$ thuộc $k$ thì $[x:1-x]=0$ trong $H^{2,2}(k,\mathbb{Z})$.

 

Chứng minh. Viết $n=m.p$ với $p$ nguyên tố, ta sẽ chứng minh $m[x:1-x]=0$. Đặt $y = \sqrt[p]{x}$ và $L=k(y)$. Ta có

$$ 0 = mp[y:1-y] = m[x:1-y] \ \ \ \text{và} \ \ \ 1 - x = N_{L/k}(1-y).$$

Do đó

$$0 = N_{L/k}(m[x:1-x]) = m [x:N_{L/k}(1-y)] = m[x:1-x].$$

Ta cứ làm như vậy cho tới khi thu được $[x:1-x]=0$.

 

Mệnh đề. $[a:1-a]=0$ trong $H^{2,2}(k,\mathbb{Z})$.

 

Chứng minh. Xét $Z$ là một tương ứng hữu hạn từ $\mathbb{A}_k^1$ (tham số hoá bởi $t$) đến $\mathbb{A}_k^1 - \left \{0 \right \}$ (tham số hoá bởi $x$) xác định bởi phương trình

$$x^3 - t(a^3+1)x^2 + t(a^3+1)x - a^3 = 0.$$

Gọi $\omega$ là một nghiệm của $x^2+x+1=0$ (trong một mở rộng nào đó), thế thì $\omega^3=1$, ta đặt $L=k(\omega)$. Thớ trên điểm $t=0$ có ba điểm $a,\omega a,\omega^2 a$ và thớ trên điểm $t=1$ bao gồm $a^3$ và hai căn bậc $6$ của $1$ do

$$x^3 - (a^3+1)x^2 + (a^3+1)x - a^3 = x^2(x-a^3)-x(x-a^3)+(x-a^3) = (x-a^3)(x^2-x+1)=0.$$

Dùng ánh xạ $x \longmapsto (x,1-x)$ để nhúng $\mathbb{A}^1_k - \left \{0,1 \right \}$ vào $X^2$ thì $Z$ cảm sinh một tương ứng $Z'$ từ $\mathbb{A}^1$ vào $X$. Khi đó trong $H^{2,2}(L,\mathbb{Z})$ ta có

$$\partial_0(Z') = [a:1-a] + [\omega a:1 - \omega a] + [\omega^2 a : 1 - \omega^2 a] = [a : 1- a^3] + [\omega : (1 - \omega a)(1-\omega^2 a)^2].$$

bằng với

$$\partial_1(Z') = [a^3:1-a^3] + [-\omega : 1 + \omega] + [-\omega^2 : 1 + \omega^2].$$

Nhân tất cả với $3$ thì các hạng tử có dạng $[\omega:b]$ triệt tiêu do $\omega^3 = 1$ và $[-1:1+\omega]+[-1:1+\omega^2] = 0$ do $(1+\omega)(1+\omega^2)=1$. Do đó trừ vế với vế ta thu được $2[a^3:1-a^3]=0$ trên $L$. Áp dụng chuẩn vào cho ta $4[a^3:1-a^3]=0$ trên $k$. Đẩy lên $k(\sqrt[3]{a})$ và lấy chuẩn lần nữa cho ta $12[a:1-a]=0$ trên $k$. Áp dụng bổ đề trước với $n=12$ cho ta đpcm.




#734879 K-lý thuyết Milnor

Posted by bangbang1412 on 09-09-2022 - 15:26 in Toán học hiện đại

Kết nối với đối đồng điều Galois và đối đồng điều étale

 

Cho $F$ là một trường mà $\mathrm{Char}(F) \neq 2$, $F_{sep}$ là một bao tách được của $k$, $G = \mathrm{Gal}(F_{sep}/F)$ là nhóm Galois tuyệt đối. Khi đó ta có một dãy khớp

$$1 \longrightarrow \left \{\pm 1 \right \} \longrightarrow F_{sep}^{\times} \overset{2}{\longrightarrow} F_{sep}^{\times} \longrightarrow 1.$$

Ta có thể thấy $G$ tác động lên dãy khớp này, lấy dãy khớp của đối đồng Galois cho ta một dãy khớp

$$H^0(G,F_{sep}^{\times}) \overset{2}{\longrightarrow} H^0(G,F_{sep}^{\times}) \longrightarrow H^1(G,\mathbb{Z}/2) \longrightarrow H^1(G,F_{sep}^{\times}).$$

Do đó ta có thể đồng nhất dãy trên với dãy

$$F^{\times} \overset{2}{\longrightarrow} F^{\times} \overset{\delta}{\longrightarrow} H^1(G,\mathbb{Z}/2) \longrightarrow 0,$$

trong đó $0$ ở tận cùng là ta áp dụng định lý Hilbert 90, $H^1(G,F_{sep}^{\times})=0$. Điều này cho ta một đẳng cấu $h_1: H^1(G,\mathbb{Z}/2) \simeq F^{\times}/(F^{\times})^2$. Ta định nghĩa K-lý thuyết Milnor modulo $2$ bởi $k_*F = K_*^M(F)/2K_*^M(F)$. Đẳng cấu $h_1$ có thể mô tả cụ thể: bằng cách lạm dụng kí hiệu, ta vẫn kí hiệu $1$-kí hiệu $\left \{a \right \}$ modulo $2$ bởi $\left \{a \right\}$. Gọi $b$ là một căn bậc hai của $a$, i.e., $b^2=a$. Đồng cấu $G = \mathrm{Gal}(F_{sep}/F) \longrightarrow \mathbb{Z}/2$ định nghĩa bởi $\gamma \longmapsto \epsilon_{\gamma}$ sao cho $(-1)^{\epsilon} = \gamma(b)/b$ là một $1$-đối chu trình. Lớp đối đồng điều của nó chính là $h^1(\left \{a \right \})$. K-lý thuyết Milnor modulo $2$ được sinh bởi các kí hiệu $\left \{a \right \}$ thoả mãn các quan hệ:

  • $\left \{ab \right \} = \left \{a \right \} + \left \{b \right \}$;
  • $\left \{a,1-a \right \} = 0 \ \forall \ a \neq 0,1$;
  • $2\left \{a \right \} = 0$.

Định lý (Bass-Tate). Đẳng cấu $h_1: k_1(F) \longrightarrow H^1(G,\mathbb{Z}/2)$ mở rộng duy nhất thành một đồng cấu vành $h_F: k_*F \longrightarrow H^*(G,\mathbb{Z}/2)$.

 

Chứng minh. Hiển nhiên đẳng cấu $h_1$ cho ta một đồng cấu $k_1(F)^{\otimes} \longrightarrow H^n(G,\mathbb{Z}/2)$ trong đó vế phải ta đã lấy tích cup. Để kiểm tra rằng đồng cấu $k_1(F)^{\otimes} \longrightarrow H^n(G,\mathbb{Z}/2)$ tách qua $k_*F$ ta chỉ cần kiểm tra ba quan hệ sinh của $k_*F$. Quan hệ đầu và cuối là tầm thường, quan hệ giữa tương đương với $(a) \cdot (1-a)=0$ trong $H^2(G,\mathbb{Z}/2)$. Nếu $a \in (F^{\times})^2$ thì điều này hiển nhiên. Nếu không lấy $b^2=a$ và đặt $L=K(b)$. Khi đó $N_{L/F}(1-b)=(1-b)(1+b)=1-a$. Do đó theo công thức chiếu,

$$(a) \cdot (1-a) = (a) \cdot N_{L/F}(1-b) = N_{L/F}((a)_L \cdot (1-b)) = N_{L/F}((b^2) \cdot (1-b)) = 0.$$




#734827 K-lý thuyết Milnor

Posted by bangbang1412 on 06-09-2022 - 15:29 in Toán học hiện đại

Luật thuận nghịch Weil và ánh xạ chuẩn của K-lý thuyết Milnor

 

Trong chứng minh của bổ đề x, ta thấy với mỗi cặp $\pi,\pi'$, ta thu được một đồng cấu $\phi_{\pi}:K^M_{n-1}(k[t]/(\pi) \longrightarrow K^M_n(k(t))$ trong đó $\phi_{\pi} \circ \partial_{\pi'}$ là $\mathrm{id}$ nếu $\pi = \pi'$ và $0$ nếu $\pi \neq \pi'$. Ta gọi nó là đồng cấu đối thặng dư.

 

Ta đồng nhất mỗi điểm đóng $P$ của đường thẳng xạ ảnh $\mathbb{P}_k^1$ với một đa thức bất khả quy, do đó ta có một ánh xạ thặng dư $\partial_P$. Tại điểm $\infty$ ta cũng có đồng cấu thặng dư mà ta kí hiệu bởi $\partial_{\infty}$. Với mỗi điểm đóng $P$, ta kí hiệu định giá tương ứng bởi $v_P$ và trường thặng dư bởi $\kappa(P)$.

 

Định nghĩa. Với $\pi \in k[t]$ bất khả quy, ta định nghĩa đồng cấu chuẩn (norm map) $N_{\pi}$ bởi công thức $N_{\pi} = -\partial_{\infty} \circ \phi_{\pi}$, tức là đối của hợp thành của đồng cấu đối thặng dư và đồng cấu thặng dư. Tại điểm $\infty$, ta định nghĩa $N_{\infty} = \mathrm{id}$ trên $K^M_n(k)$.

 

Định lý (luật thuận nghịch Weil). Với mọi $\alpha \in K^M_n(k(t))$, ta có

$$\sum_{\pi \in \mathbb{P}^1_0 }(N_{\pi} \circ \partial_{\pi})(\alpha) = 0,$$

trong đó $\mathbb{P}^1_0$ là tập các điểm đóng của $\mathbb{P}^1_k$, có nghĩa là $\pi$ hoặc là đa thức bất khả quy hoặc là điểm vô hạn.

 

Chứng minh. Với $P \neq \infty$ ta có

$$\partial_P\left(\alpha - \sum_{P \neq \infty}(\psi_P \circ \partial_P)(\alpha) \right) = \partial_P(\alpha)-\partial_P(\alpha)=0.$$

Do đó theo dãy khớp Tate $\alpha - \sum_{P \neq \infty}(\psi_P \circ \partial_P)(\alpha) \in K^M_n(k)$. Áp dụng $\partial_{\infty}$ vào hai vế cho ta đpcm.

 

Trong trường hợp $n=2$ luật thuận nghịch Weil có dạng sao

$$\sum_{P \in \mathbb{P}^1_0} (N_{\kappa(P)/k} \circ \partial_P)(\alpha)=0.$$

Ta áp dụng với $\alpha = \left \{f,g \right \}$ trong đó $f,g \in k(t)$. Theo ví dụ x thì ánh xạ thặng dư $\partial_P: K^M_2(k(t)) \longrightarrow K^M(\kappa(P))$ thoả mãn $\partial(\left \{f,g \right \}) = (-1)^{v_P(f)v_P(g)}\overline{f^{v_P(g)}g^{v_P(f)}}$, ở đây $\overline{-}$ kí hiệu ảnh của phần tử trong $\kappa(P)$. Vấn đề còn lại là làm sao để tính $N_{\kappa(P)/k}$ tại bậc $1$.

 

Mệnh đề. Với $n=0$ thì $N_P:K_0^M(\kappa(P)) \longrightarrow K_0^M(k)$ cho bởi phép nhân với $[\kappa(P):k]$ (bậc của mở rộng trường) còn khi $n=1$ thì $N_P:\kappa(P)^{\times} \longrightarrow k^{\times}$ chính là đồng cấu chuẩn của mở rộng trường (theo nghĩa thông thường).

 

Để chứng minh mệnh đề này ta cần bổ đề sau.

 

Bổ đề. Cho $K/k$ là một mở rộng trường và $P$ là một điểm đóng của $\mathbb{P}^1_k$, khi đó biểu đồ

bieudo1.jpg

 

giao hoán, trong đó $Q \longmapsto P$ có nghĩa là điểm đóng $Q$ của $\mathbb{P}^1_K$ nằm trên (lying over) $P$, và $e_Q$ là chỉ số rẽ nhánh của định giá $v_Q$ mở rộng $v_P$ từ $K(t)$, $i$ là phép nhúng tự nhiên.

 

Chứng minh. Đây là hệ quả của lưu ý x và định nghĩa của chuẩn. Người đọc tự hoàn thành chứng minh.

 

Chứng minh mệnh đề. Áp dụng bổ đề trên với $K = k^{alg}$ là bao đóng đại số của $k$, khi đó mỗi điểm $Q$ đều có bậc $1$ trên $K$ nên $N_Q$ đều là ánh xạ đồng nhất. Hơn nữa các đồng cấu dọc đều là các phép nhúng với $n=0,1$. Khi đó trường hợp $n=0$ là hệ quả của công thức $\sum e_Q = [\kappa(P):k]$ và trường hợp $n=1$ là hệ quả của định nghĩa chuẩn của mở rộng trường $N_{\kappa(P)/k}(\alpha)$ là tích các phần tử liên hợp của $\alpha$ (tính cả bội).

 

Định lý (Kato). Cho $L/k$ là một mở rộng trường hữu hạn, khi đó với mỗi $n \geq 0$ tồn tại một ánh xạ chuẩn $N_{K/k}: K^M_n(L) \longrightarrow K^M_n(k)$ thoả mãn các tính chất sau:

  • Khi $n = 0$ thì $N_{L/k}$ là phép nhân với $[L:k]$.
  • Khi $n = 1$ thì $N_{L/k}$ là ánh xạ chuẩn thông thường $L^{\times} \longrightarrow k^{\times}$.
  • Với $\alpha \in K^M_n(k)$ và $\beta \in K^M_m(L)$ ta có công thức chiếu $N_{L/k}(\left \{\alpha_L,\beta \right \}) = \left \{\alpha,N_{L/k}(\beta)\right \}$.
  • Với $F/L/k$ là một tháp trường, ta có $N_{F/k} = N_{F/L} \circ N_{L/k}$.

Ta không chứng minh định lý này (nó rất khó!) mà chỉ nêu một vài lưu ý ở đây:

  • Khi $L=k(a)$ là một mở rộng đơn thì đa thức tối tiểu của $a$ định nghĩa một điểm đóng $P \in \mathbb{P}^1_k$ sao cho $L \simeq \kappa(P)$, ta định nghĩa $N_{L/k}$ bởi $N_{\kappa(P)/k}$ như đã làm trước định lý Kato.
  • Khi $L=k(a_1,...,a_r)$ ta tách nó thành một dãy hợp thành các mở rộng đơn và định nghĩa $N_{L/k}$ như là hợp của các ánh xạ chuẩn trên các mở rộng đơn. Công sức lớn của Kato là chứng minh cách định nghĩa này không phụ thuộc vào hệ sinh $a_1,...,a_r$.

Định lý (luật thuận nghịch Weil tổng quát dạng hình học). Cho $C$ là một đường cong xạ ảnh trơn trên $k$ với trường hàm $k(C)$. Khi đó với mỗi điểm đóng $P$ của $C$ ta có ánh xạ thặng dư $\partial_P: K^M_n(k(C)) \longrightarrow K^M_{n-1}(\kappa(P))$. Khi đó với mọi $\alpha \in K^M_n(k(C))$ ta có

$$\sum_{P \in C_0} (N_{\kappa(P)/k} \circ \partial_P)(\alpha) = 0,$$ trong đó $C_0$ là tập các điểm đóng của $C$.

 

Ta không chứng minh luận thuật nghịch Weil mà đưa ra một hệ quả thú vị của nó. Lưu ý rằng luật thuận nghịch Weil còn có thể phát biểu như:

 

Định lý (luật thuận nghịch Weil dạng đại số). Cho $L$ là một trường hàm trên $k$ (một mở rộng hữu hạn của $k(t)$). Khi đó với mọi $x \in K^M_{n+1}(L)$ ta có

$$\sum_v N_{k(v)/k} \circ \partial_v)(x) = 0,$$

trong đó tổng chạy trên tất cả các định giá rời rạc của $L$.

 

Hệ quả. Cho $p: Z \longrightarrow \mathbb{A}_k^1$ là một toàn cấu hữu hạn (surjective + finite) và giả sử $Z$ là nguyên (integral). Lấy $f_1,...,f_n \in \mathcal{O}^{\times}(Z)$ cùng các số nguyên $n_i^0,n_j^1$ sao cho

$$p^{-1}(0) = \coprod n_i^0 z_i^0 \ \ \ \text{và} \ \ \ p^{-1}(1) = \coprod n_i^1 z_i^1$$

trong đó $n_i^{\epsilon}$ là chỉ số rẽ nhánh của các điểm $z_i^{\epsilon}=\mathrm{Spec}(E_i^{\epsilon})$ ($\epsilon \in \left \{0,1\right \}$). Định nghĩa

$$\varphi_0 = \sum n_i^0 N_{E_i^0/k}(\left \{f_1,...,f_n \right \}_{E_i^0}) \ \ \ \text{và} \ \ \ \varphi_1 = \sum n_i^1 N_{E_i^1/k}(\left \{f_1,...,f_n \right \}_{E_i^1}).$$

Khi đó $\varphi_0 = \varphi_1 \in K^M_n(k)$.

 

Chứng minh. Kí hiệu $L$ là trường hàm của $Z$, khi đó $L/k$ là mở rộng hữu hạn, gọi $t$ là tham số cho $\mathbb{A}^1_k$. Ta xét kí hiệu $x = \left \{\frac{t}{t-1},f_1,...,f_n \right \}$. Tại các chốn vô cùng, $\frac{t}{t-1}$ bằng $1$ và do đó ánh xạ thặng dư triệt tiêu, $\partial_v(x)=0$. Tương tự, $\partial_v(x)=0$ bằng không tại mọi chốn trừ các chốn tại $0$ và $1$. Nếu $v_i$ nằm trên $t=0$ thì

$$\partial_{v_i}(x) = \partial_{v_i}(\left \{t,f_1,...,f_n \right \} - \left \{t-1,f_1,...,f_n \right \}) = v_i(t)\left \{f_1,...,f_n \right \} = n_i^0\left \{\overline{f_1},...,\overline{f_n} \right \},$$

xem như một phần tử của $K^M_n(E^0_i)$. Tương tự $\partial_{v_i}(x) = -n_i^0\left \{\overline{f_1},...,\overline{f_n} \right \}$ nếu $v_i$ nằm trên $t=1$. Cuối cùng theo luật thuận nghịch Weil thì $\sum N\partial_{v_i}(x) = \varphi_0 - \varphi_1$ triệt tiêu trong $K^M_n(k)$.




#734875 K-lý thuyết Milnor

Posted by bangbang1412 on 09-09-2022 - 14:26 in Toán học hiện đại

Đối đồng điều motivic

 

Trong phần này ta sẽ định nghĩ ngắn gọn đối đồng điều motivic và chứng minh rằng K-lý thuyết Milnor bậc $n$ có thể xem như middle $(n,n)$-đối đồng điều motivic. Formulation của đối đồng điều motivic mà ta sử dụng ở đây sẽ là hypercohomology trên Zariski site. Cố định một trường $k$. Giả sử $X,Y \in \mathbf{Sch}/k$ là hai $k$-lược đồ hữu hạn sinh (finitely generated $k$-schemes), ta kí hiệu $Z(X)$ bởi nhóm abel tự do sinh bởi các chu trình đại số của $X$.

 

Định nghĩa. Một tương ứng hữu hạn sơ cấp (elementary finite correspondence) từ $X$ tới $Y$ là một biểu diễn lược đồ nguyên, đóng $W \subset X \times_k Y$ sao cho $pr_1: W \longrightarrow X$ là hữu hạn và $pr_1(W)$ là một thành phần bất khả quy của $X$. Một tương ứng hữu hạn là một tổng hình thức $\sum n_i W_i$ với $n_i \in \mathbb{Z}$ và $W_i$ là cac tương ứng hữu hạn sơ cấp.

 

Ta kí hiệu $\mathrm{Cor}(X,Y)$ bởi tập các tương ứng hữu hạn từ $X$ tới $Y$. Giả sử $Z \in \mathbf{Sch}/k$. Lấy $\alpha \in \mathrm{Cor}(X,Y), \beta \in \mathrm{Cor}(Y,Z)$, khi đó ta có thể định nghĩa một phép hợp thành

$$\beta \circ \alpha = \pi_{XZ *}(\pi_{XY}^*(\alpha) \cdot \pi_{YZ}^*(\beta)),$$

trong đó $\pi_{XY},\pi_{YZ},\pi_{ZX}$ là phép chiếu từ $X \times Y \times Z$ xuống các thành phân tương ứng, các phép $(-)^*$ và $(-)_*$ là kéo lùi và đẩy xuôi của chu trình đại số.

 

Bằng cách này ta có một phạm trù gọi là phạm trù các tương ứng hữu hạn trên $k$, kí hiệu bởi $\mathbf{Cor}(k)$ với vật là các $k$-lược đồ hữu hạn sinh và cấu xạ là các tương ứng hữu hạn. Kí hiệu $\mathbf{Sm}/k$ là phạm trù các $k$-lược đồ trơn. Khi đó ta có một hàm tử

$$\mathbf{Sm}/k \longrightarrow \mathbf{Cor}(k), \ \ X \longmapsto X, \ \ f: X \longrightarrow Y \longmapsto \Gamma_f = \text{đồ thị của} \ f,$$

trong đó đồ thị của $f$ xem như một tương ứng hữu hạn từ $X$ tới $Y$; ta có thể bẻ nó thành các thành phần bất khả quy nếu cần thiết. Phạm trù $\mathbf{Cor}(k)$ là một phạm trù tensor, cộng tính.

 

Định nghĩa. Một PST hay một tiền bó đo chu trình-đồ thị (tạm dịch presheaf with transfers) là một hàm tử cộng tính $F:\mathbf{Cor}(k)^{op} \longrightarrow \mathbf{Ab}$ ($\mathbf{Ab}$: phạm trù các nhóm giao hoán). Một PST được gọi là một bó đo chu trình-đồ thị nếu $F_{X_{Nis}}$ là một bó với mỗi $X \in \mathbf{Sm}/k$, ở đây $X_{Nis}$ là Nisnevich site trên $X$.

 

(mình chọn cách dịch như vậy vì một tiền bó $F: \mathbf{Cor}(k)^{op} \longrightarrow \mathbf{Ab}$ thực chất là một tiền bó đi cùng với các ánh xạ $F(Y) \longrightarrow F(X)$ lưu trữ phần thông tin của các cấu xạ $X \longrightarrow Y$ trong $|mathbf{Cor}(k)$ không cảm sinh bởi đồ thị của một cấu xạ lược đồ.)

 

Ví dụ.

  • Nhóm nhân tính $\mathcal{O}^*$ luôn là một PST.
  • Cho $X \in \mathbf{Sm}/k$, khi đó hàm tử $\mathbb{Z}_{tr}(X)(U) = \mathrm{Hom}_{\mathbf{Cor}(k)}(U,X)$ là một PST (chính xác hơn, nó là PST khả diễn với vật biểu diễn là $X$. Kí hiệu $\mathbb{Z}_{tr}(\mathrm{Spec}(k))$ bởi $\mathbb{Z}$. Thực chất $\mathbb{Z}_{tr}(X)$ còn là một bó chu trình-đồ thị; nó thậm chí là bó đo chu trình-đồ thị trên cả étale site và Zariski site chứ không chỉ Nisnevich site.
  • Cho $(X,x)$ là một $k$-lược đồ định điểm $(x: \mathrm{Spec}(k) \longrightarrow X)$ khi đó có một phép chẻ $\mathbb{Z}_{tr}(X) = \mathbb{Z} \oplus \mathbb{Z}_{tr}(X,x)$. Nói riêng ta áp dụng cho $(X,x) = (\mathbb{G}_m,1)$ với $\mathbb{G}_m = \mathbb{A}^1 - \left \{0 \right \}$ để thu được một PST $\mathbb{Z}_{tr}(\mathbb{G}_m,1)$.
  • Cho $(X_1,x_1),...,(X_n,x_n)$ là các $k$-lược đồ định điểm, khi đó ta có thể định nghĩa tích smash $$\mathbb{Z}_{tr}(X_1 \wedge \cdots \wedge X_n) = \mathrm{Coker}\left(\mathbb{Z}_{tr}(X_1 \times \cdots \times \hat{X_i} \times \cdots \times X_n) \overset{\mathrm{id} \times \cdots \times x_i \times \cdots \times \mathrm{id}}{\longrightarrow} \mathbb{Z}_{tr}(X_1 \times \cdots \times X_n) \right).$$ Hệ quả là ta có tích smash $\mathbb{Z}_{tr}(\mathbb{G}_m^{\wedge n})$ với mọi $n \geq 0$.
  • Xét các $n$-đơn hình đại số cho bởi $\Delta^n = \mathrm{Spec}\left(k[x_0,...,x_n]/(x_0+\cdots+x_n-1)\right)$. Cho $F$ là một tiền bó đo chu trình-đồ thị, khi đó ta có các tiền bó đo chu trình-đồ thị $F^{\Delta^n}$ cho bởi $F^{\Delta^n}(U) = F(U \times \Delta^n)$. Chúng được bó với nhau bởi một phức $$C_*F: ... \longrightarrow F^{\Delta^n} \longrightarrow ... \longrightarrow F^{\Delta^2} \longrightarrow F^{\Delta^1} \longrightarrow F \longrightarrow 0.$$

Định nghĩa. Với mọi $n\geq 0$, ta định nghĩa phức $\mathbb{Z}(p)$ bởi công thức $\mathbb{Z}(p) = C_*\mathbb{Z}_{tr}(\mathbb{G}^{\wedge p}_m)[-p]$. Khi $A$ là một nhóm giao hoán, ta định nghĩa $A(p) = \mathbb{Z}(p) \otimes A$. Đối đồng điều motivic bậc $(p,q)$ được định nghĩa bởi hypercohomology trên Zariski site

$$H^{p,q}(X,A) = \mathbb{H}^p_{Zar}(X, A(q)),$$

trong đó $X \in \mathbf{Sm}/k$.

 

Lưu ý. Ở đây đã có một điểm ẩn mà ta lưu ý, đó là đối đồng điều lấy trên Zariski site thì phức $\mathbb{Z}(p)$ phải là phức của các bó chứ không phải tiền bó. Điều đó được khẳng định trong mệnh đề sau.

 

Bổ đề. Với mọi $k$-lược đồ $Y$, $\mathbb{Z}_{tr}(Y)$ là một bó trên tô-pô Zariski và do đó $C_*\mathbb{Z}_{tr}(Y)$ là một phức các bó; nói riêng, hypercohomology là có nghĩa.

 

Chứng minh. Giả sử $U$ được phủ bởi $U_1,U_2$, ta cần chứng minh có một dãy khớp

$$0 \longrightarrow \mathrm{Cor}(U,Y) \overset{diag}{\longrightarrow} \mathrm{Cor}(U_1,Y) \oplus \mathrm{Cor}(U_2,Y) \overset{(+,-)}{\longrightarrow} \mathrm{Cor}(U_1 \cap U_2,Y).$$

Không giảm tổng quát ta giả sử $U$ liên thông, do đó $U$ là bất khả quy (do $U$ trơn, xem tại đây) và do đó cũng nguyên do regular local rings thì reduced. Mọi tương ứng hữu hạn từ $U$ sang $Y$ thì áp đảo (dominant) $U$ và do đó hoàn toàn xác định bởi thớ trên điển generic của $U$ (thớ trên điểm generic trù mật). Do đó $\mathrm{Cor}(U,Y)$ nhúng vào từng $\mathrm{Cor}(U_i,Y)$.

 

Để thấy dãy trên khớp ở vị trí còn lại, ta lấy $Z_1 =  \sum_{i \in I} m_i Z_{1i} \subset Z(U_i \times Y)$ và $Z_2 = \sum_{j \in J} n_j Z_{2j} \in Z(U_2 \times Y)$ sao cho $Z_1=Z_2$ trên $(U_1 \cap U_2) \times Y$. Theo lập luận trên, tương ứng hữu hạn xác định hoàn toàn bởi thớ trên điểm generic của $U,U_1,U_2$, do đó tồn tại một phép song ánh giữa $I$ và $J$ sao cho nếu $i \in I$ tương ứng với $j \in J$ thì $m_i = n_j$ và $Z_{1i} = Z_{2j}$ trên $(U_1 \cap U_2) \times Y$. Không giảm tổng quát ta giả sử $Z_1,Z_2$ là các tương ứng hữu hạn sơ cấp. Khi đó $Z = Z_1 \cup Z_2$ là một tương ứng hữu hạn từ $U$ tới $Y$ mà hạn chế xuống $U_i \times Y$ là $Z_i$, đpcm.

 

 

Bổ đề. Với mọi trường $F$ và các số nguyên $p,q \geq 0$, ta có $H^{p,q}(\mathrm{Spec}(F),\mathbb{Z}) = H_{q-p}(C_*\mathbb{Z}_{tr}(\mathbb{G}_m^{\wedge q})(\mathrm{Spec}(F))$. Nói riêng

$$H^{n,n}(\mathbb{Spec}(F),\mathbb{Z}) = H_0(C_*\mathbb{Z}_{tr}(\mathbb{G}_m^{\wedge n})(\mathrm{Spec}(F)) = \mathrm{Coker}\left(\mathbb{Z}_{tr}(\mathbb{G}_m^{\wedge n})(\mathbb{A}^1) \overset{\partial_0 - \partial_1}{\longrightarrow} \mathbb{Z}_{tr}(\mathbb{G}_m^{\wedge n})(\mathrm{Spec}(F)) \right).$$

 

Sau đây ta tìm hiểu một số bậc thấp của đối đồng điều motivic.

 

Mệnh đề. Tồn tại một tựa đẳng cấu (quasi-isomorphism) $\mathbb{Z}(1) \simeq \mathcal{O}^*[-1]$.

 

Phác thảo chứng minh. Xét hàm tử $\mathscr{M}(\mathbb{P}^1;0,\infty): \mathbf{Sm}_k \longrightarrow \mathbf{Ab}$ gửi mỗi $k$-lược đồ trơn $X$ với nhóm của các hàm hữu tỷ trên $X \times \mathbb{P}^1$ mà chính quy (regular) trên một lân cận của $X \times \left \{0, \infty \right \}$ và bằng $1$ trên $X \times \left \{1,\infty \right \}$. Khi đó với mọi $f \in \mathscr{M}(\mathbb{P}^1;0,\infty)(X)$ thì ước Weil tương ứng của $f$ là một tương ứng hữu hạn $\mathbf{Cor}(X,\mathbb{A}^1 - \left \{0 \right \})$. Ngoài ra ta có một dãy khớp

$$0 \longrightarrow \mathscr{M}(\mathbb{P}^1;0,\infty) \longrightarrow \mathbb{Z}_{tr}(\mathbb{A}^1 -\left \{0 \right \})(X) \longrightarrow \mathbb{Z} \oplus \mathcal{O}^*(X) \longrightarrow 0.$$

Ngoài ra $\mathscr{M}(\mathbb{P}^1;0,\infty)$ là một PST, và $C_*(\mathscr{M})(X)$ là acyclic với mọi $X \in \mathbf{Sm}/k$, cho ta đpcm.

 

Hệ quả. Với $X \in \mathbf{Sm}/k$, ta có

$$H^{p,q}(X,\mathbb{Z}) = \begin{cases}
        0 & q \leq 1 \ \text{and} \ (p,q) \neq (0,0),(1,1),(2,1) \\
        \mathbb{Z}(X) & (p,q) = (0,0) \\
        \mathcal{O}^*(X) & (p,q)=(1,1) \\
        \mathrm{Pic}(X) & (p,q)=(2,1).
    \end{cases}$$

 

Cho $l$ là một số nguyên tố sao cho $1/l \in k$. Tensor $\mathbb{Z}/l$ với đẳng cấu $\mathbb{Z}(1) \simeq \mathcal{O}^*[-1]$ cho ta

 

Hệ quả. Nếu $1/l \in k, X \in \mathbf{Sm}/k$, khi đó $H^{p,1}(X,\mathbb{Z}/l)=0$ với $p \neq 0,1,2$ và

\begin{align*}
    H^{0,1}(X,\mathbb{Z}/l) & =\mu_l(X) \\
    H^{1,1}(X,\mathbb{Z}/l) & = H^1_{\text{ét}}(X,\mu_l) \\
    H^{2,1}(X,\mathbb{Z}/l) & = \mathrm{Pic}(X)/l\mathrm{Pic}(X).
\end{align*}




#734826 K-lý thuyết Milnor

Posted by bangbang1412 on 06-09-2022 - 15:08 in Toán học hiện đại

Ánh xạ thặng dư của K-lý thuyết Milnor và bổ đề Bass-Tate

 

Giả sử $k$ được trang bị một định giá rời rạc $v: k^{\times} \longrightarrow \mathbb{Z}$. Ta có thể xem $v$ như một ánh xạ $K_1^M(k) \longrightarrow K_0^M(\overline{k})$ trong đó $\overline{k}$ là trường thặng dư. Gọi $\mathcal{O} = \left \{x \in k^{\times} \mid v(x) \geq 0 \right \} \cup \left \{0 \right \}$ là vành định giá tương ứng, $\mathfrak{m} = \left \{x \in k^{\times} \mid v(x) = 0 \right \} \cup \left \{0 \right \}$ là ideal cực đại và $\mathfrak{m}^{\times} = \mathfrak{m} \setminus \left \{0 \right \}$, $\pi \in \mathcal{O}$ là một uniformizer, i.e., $v(\pi)=1$. Với mỗi $u \in \mathfrak{m}$, kí hiệu $\overline{u}$ là ảnh của $u$ qua ánh xạ thương $\mathcal{O} \longrightarrow \mathcal{O}/\mathfrak{m} = \overline{k}$.

 

Mục đích trong phần này của ta là chứng minh tồn tại duy nhất một ánh xạ mở rộng $v$ lên $K_n^M(k) \longrightarrow K_{n-1}^M(\overline{k})$.

 

Mệnh đề. Với mọi $n \geq 1$, tồn tại duy nhất một đồng cấu, gọi là ánh xạ thặng dư $\partial_v: K_n^M(k) \longrightarrow K^M_{n-1}(\overline{k})$ thoả mãn

$$\partial_v(\left \{a,u_2,...,u_n \right \}) = v(a)\partial_v(\left\{\overline{u_2},...,\overline{u_n} \right \})$$ với mọi $a \in k^{\times}$, $u_2,...,u_n \in \mathfrak{m}^{\times}$.

 

Chứng minh.

 

Tính duy nhất. Nếu $\partial_v$ tồn tại, thế thì với mọi uniformizer $\pi$ ta có

$$\partial_v(\left \{\pi,u_2,...,u_n \right \}) = \partial_v(\left\{\overline{u_2},...,\overline{u_n} \right \}), \ \ \ \partial_v(\left \{u_1,...,u_n \right \}) = 0$$

với mọi $u_1,...,u_n \in \mathfrak{m}$. Nhưng mọi $a \in k^{\times} = (\mathrm{Frac}(\mathcal{O}))^{\times}$ có thể viết dưới dạng $a = \pi^{v(a)}u$ với $u \in \mathfrak{m}^{\times}$. Hơn nữa $\left \{\pi \right \}^2 = \left \{\pi \right \} \left \{-1 \right\}$ nên ta có thể thấy $K^M_*(k)$ được sinh bởi các phần tử thuộc một trong hai dạng $\left \{\pi,u_2,...,u_n \right \}$ hoặc $\left\{u_2,...,u_n \right \}$. Từ đó ta thấy nếu tồn tại thì nó xác định duy nhất.

 

Tính tồn tại. Xét vành $L(\overline{k}) = K^M_n(\overline{k})[\zeta]$ trong đó $\zeta^2 = \left \{-1 \right \}\zeta$ và $\zeta \alpha = -\alpha \zeta$ với mọi $\alpha \in K^M_1(\overline{k})$. Ta đặt $\zeta$ tại bậc $1$, và do đó $L$ được phân bậc như sau

$$L_n(\overline{k}) = K_n^M(\overline{k}) \oplus \zeta K_{n-1}(\overline{k}).$$

Xét ánh xạ $d_{\pi}:K_1^M(k) \longrightarrow L_1(\overline{k})$ xác định bởi $\left \{\pi^i u \right \} \longmapsto \left \{\overline{u} \right \} + \zeta i$. Hiển nhiên $d_{\pi}$ cảm sinh một đồng cấu $d_{\pi}^{\otimes 2}: (K_1^M(k))^{\otimes} \longrightarrow L_2(\overline{k})$. Ta sẽ chứng minh rằng

$$d_{\pi}^{\otimes 2}(\left \{a \right\} \otimes \left \{1 - a \right \}) = 0,$$

với mọi $a \in k \setminus \left \{0, 1 \right \}$. Thật vậy mọi phần tử $a \in k^{\times}$ đều có dạng $\pi^{v(a)} u$ với $u$ là đơn vị, i.e., $u \in \mathfrak{m}^{\times}$. Nếu $v(a) > 0$ thì $1 - a = 1 - \pi^{v(a)}u$ là đơn vị với thặng dư $\overline{1-a} = \overline{1-\pi^{v(a)}u} = \overline{1}$. Khi đó $d_{\pi}(\left \{1-a \right \}) = \left \{\overline{1} \right \} = 0 \in K_1^M(\overline{k})$. Trong trường hợp $v(a) < 0$, ta có $1 - a = \pi^{v(a)}(\pi^{-v(a)}-u)$ trong đó $\pi^{-v(a)}-u$ là một đơn vị và có lớp thặng dư là $\overline{\pi^{-v(a)}-u} = -\overline{u}$. Do đó

$$d^{\otimes 2}_{\pi}(\left \{a \right \} \otimes \left \{1-a \right \}) = (\left \{\overline{u} \right \} + \zeta v(a))(\left \{-\overline{u} \right \} + \zeta v(a)) = (v(a)-v(a)^2)\zeta \left \{-1 \right \} = \zeta \left \{(-1)^{v(a)-v(a)^2} \right \} = 0.$$ Trường hợp cuối cùng khi $v(a)=0$ mà $v(1-a) \neq 0$ thì ta có thể sử dụng hai trường hợp trước. Nếu $v(a)=v(1-a)=0$, tức là cả $a,1-a$ đều là đơn vị thì

$$d^{\otimes 2}_{\pi}(\left \{a \right \} \otimes \left \{1-a \right \}) = \left \{\overline{u} \right \} \left \{\overline{1-u} \right \} = \left \{\overline{u},1 - \overline{u} \right \} = 0 \in K_2^M(\overline{k}).$$

Như vậy $d^{\otimes n}_{\pi}$ tách qua $K^M_n(k)$ và ta hợp thành ánh xạ tách này với phép chiếu chính tắc $L_n(\overline{k}) \longrightarrow K^M_{n-1}(\overline{k})$ để thu được $\partial_v$. Lưu ý rằng dù ta cố định một uniformizer, ánh xạ cuối cùng không phụ thuộc cách chọn ban đầu.

 

Lưu ý. Ánh xạ thặng dư của mở rộng trường có mô tả cụ thể như sau. Cho $L/k$ là một mở rộng của các trường định giá rời rạc với các định giá tương ứng $v_L$ và $v_k$ sao cho $(v_L)_{\mid k} = v_k$. Kí hiệu các uniformizer tương ứng là $\pi_L$ và $\pi_k$ và $e$ bởi chỉ số rẽ nhánh, i.e., $\pi_k = u\pi_L^e$ ($u$: đơn vị). Khi đó $\partial_L(\alpha_L) = e \partial_k(\alpha)$ với mọi $\alpha \in K^M_n(k)$.

 

Giờ với trường $k$ bất kì, lấy $\pi \in k[t]$ là một đa thức bất khả quy, khi đó ta có một định giá $(\pi)$-adic, là một định giá rời rạc $v_{\pi}: k(t) \longrightarrow k[t]/(\pi)$. Nó gửi mỗi $a(t)/b(t)$ (với $a,b \in k[t]$) tới $\mathrm{ord}_{\pi}(a) - \mathrm{ord}_{\pi}(b)$ trong đó $\mathrm{ord}_{\pi}(a) = i$ nếu $a(t)=\pi(t)^i r(t)$ với $\pi \nmid r$ trong $k[t]$. Ta lưu ý rằng mọi trị tuyệt đối trên $k(t)$ mà tầm thường trên $k$ thì tương đương với một trị tuyệt đối $(\pi)$-adic hoặc một trị tuyệt đối ở vô hạn (định nghĩa bởi hiệu bậc của tử và mẫu). Do đó ta có một ánh xạ thặng dư $\partial_{\pi}: K^M_n(k(t)) \longrightarrow K^M_{n-1}(k[t]/(\pi))$.

 

Mệnh đề (Tate). Họ các ánh xạ thặng dư $\partial_{\pi}$ cho ta một dãy khớp chẻ

$$0 \longrightarrow K^M_n(k) \longrightarrow K_n^M(k(t)) \longrightarrow \bigoplus K^M_n k[t]/(\pi) \longrightarrow 0.$$

với mọi $n \geq 0$.

 

Chứng minh. Cố định $n$, xét $L_d \subset K_n(k(t))$ là nhóm con sinh bởi $\left \{f_1 \right \}\cdots\left \{f_n \right \}$ với $f_1,...,f_n \in F[t]$ là các đa thức bậc $\leq d$. Do đó $L_0 \subset L_1 \subset L_2 \subset ...$ và hợp thành $\bigcup_{i \geq 0} L_i = K_n(k(t))$. Khi $\mathrm{deg}(\pi)$ ta có một đồng cấu thặng dư $\partial_{\pi}:K_n(k(t)) \longrightarrow K_{n-1}(k)$, và do đó $L_0$ là một hạng tử trực tiếp của $K_n(k(t))$, hơn nữa $L_0$ đẳng cấu tự nhiên với $K_n(k)$. 

 

Để tính toán hạng tử trực tiếp còn lại, ta cần hai bổ đề sau. Với $\pi$ bất khả quy, cố định, ta kí hiệu $\overline{g} \in k[t]/(\pi)$ là lớp thương của một đa thức $g \in k[t]$. Không giảm tổng quát, ở đây ta luôn có thể giả sử $\mathrm{deg}(g) < \mathrm{deg}(\pi) = d$.

 

Bổ đề. Tồn tại duy nhất một đồng cấu

$$\psi_{\pi}: K_{n-1}k[t]/(\pi) \longrightarrow L_d/L_{d-1},$$

sao cho

$$\psi_{\pi}(\left \{\overline{g_2} \right \}\cdots\left \{\overline{g_n} \right \}) = \left \{\pi \right \}\left \{g_2 \right \}\cdots \left \{g_n \right \} \mathrm{mod} \ L_{d-1}.$$

Chứng minh. Trước tiên xét đồng cấu

$$K_1(k[t]/(\pi)) \times \cdots \times K_1(k[t]/(\pi)) \longrightarrow L_d/L_{d-1}, \ \ \ \left \{\overline{g_2} \right\} \cdots \left \{\overline{g_n} \right \}\longmapsto \left \{\pi \right\}\left \{g_2\right \}\cdots \left \{g_n \right \} \ \mathrm{mod} \ L_{d-1}.$$ Ta sẽ chứng minh đồng cấu này tách qua $K_nk[t]/(\pi)$. Trước tiên ta cần chứng minh nó đa tuyến tính. Ví dụ theo biến $\overline{g_2}$. Giả sử rằng $g_2 \equiv ab \ \mathrm{mod} \ (\pi)$, khi đó $g_2 = \pi c + ab$, trong đó $\mathrm{deg}(g),\mathrm{deg}(a),\mathrm{deg}(b) < d$. Trường hợp $c=0$ là hiển nhiên. Nếu $c\neq 0$, ta có $1 = \pi c/g_2 + ab/g_2$. Ta lấy $\left \{ - \right\}$ của đẳng thức này, sau đó nhân hai vế với $\left \{g_3\right \}\cdots \left \{g_n \right \}$ sau đó modulo $L_{d-1}$ để thu được tính đa tuyến tính.

 

Để chứng minh nó tách qua $K_nk[t]/(\pi)$, ta chỉ cần chứng minh ánh xạ vừa định nghĩa bằng không bất cứ khi nào có $\overline{g_i} + \overline{g_j} = \overline{1}$. Nhưng điều này có nghĩa là $1 + \pi h = g_i + g_j$. Nhưng ta giả sử $\mathrm{deg}(g_i),\mathrm{deg}(g_j) < \mathrm{deg}(\pi)=d$ nên $h=0$, hay $g_i+g_j=1$, suy ra đpcm.

 

Bổ đề. Họ các ánh xạ thặng dư $\partial_{\pi}$ với $\mathrm{deg}(\pi) =d$ bất khả quy cảm sinh một đẳng cấu giữa $L_d/L_{d-1}$ và $\bigoplus K_{n-1}k[t]/(\pi)$.

 

Chứng minh. Trước tiên với mỗi $\pi$, $\partial_{\pi}$ cảm sinh một đồng cấu $L_d/L_{d-1} \longrightarrow K_{n-1}k[t]/(\pi)$. Hơn nữa mỗi hợp thành

$$K_{n-1}k[t]/(\pi) \overset{\psi_{\pi}}{\longrightarrow} L_d/L_{d-1} \longrightarrow K_{n-1}k[t]/(\pi')$$

là ánh xạ đồng nhất hoặc ánh xạ không, phụ thuộc vào việc $\pi=\pi'$ hay $\pi \neq \pi'$, lần lượt. Như vậy để kết thúc chứng minh ta chỉ cần chứng minh $L_d/L_{d-1}$ được sinh bởi ảnh của các $\psi_{\pi}$. Lấy mỗi phần tử sinh của $L_d$ và viết dưới dạng tích $\left \{f_1\right \}\cdots \left \{f_s \right \}\left\{g_{s+1} \right \}\cdots \left \{g_n \right \}$ trong đó $\mathrm{deg}(f_i)=d$ và $\mathrm{deg}(g_j)<d$. Nếu $s\geq 2$ ta có thể viết $f_2 = -af_1+g$ với $\mathrm{deg}(g)<d$. Nếu $g \neq 0$ ta có $af_1/g + f_2/g=1$, do đó

$$(\left \{a \right \} + \left \{f_1 \right \} - \left \{g \right \})(\left \{f_2 \right \} - \left \{g \right \}) = 0.$$

Hệ quả là $\left \{f_1 \right \}\left \{f_2 \right \}=\left \{f_1\right\}\left \{g \right \} + \left \{g \right\}\left \{f_2\right \} - \left \{a \right\}\left \{f_2\right \} + \left \{a \right \}\left \{g \right \} - \left \{g \right \}^2$. Mỗi tích trong tổng vế phải chứa nhiều nhất một đa thức bậc $d$ (với $g=0$ tương tự). Như vậy bằng quy nạp ta có thể chứng minh sau khi modulo $L_{d-1}$, mọi phần tử của $L_d$ có dạng $\left \{f \right \}\left \{g_2 \right \}\cdots \left \{g_n \right \}$ với $\mathrm{deg}(f)=d$ và $\mathrm{deg}(g_i)<d$. Nếu $f$ bất khả quy thì $f=a\pi$ với $a$ đơn vị, khi đó $\left \{f \right \}\left \{g_2 \right \}\cdots \left \{g_n \right \}$ hiển nhiên nằm trong ảnh của $\psi_{\pi}$. Nhưng nếu $f$ khả quy thì $\left \{f \right \}\left \{g_2 \right \}\cdots \left \{g_n \right \}$ đồng dư $0$ modulo $L_{d-1}$ (do $f$ tách thành hai đa thức, mỗi đa thức bậc $\leq d-1$, nằm trong $L_{d-1}$). Điều này kết thúc chứng minh bổ đề.

 

Tiếp tục chứng minh mệnh đề. Quy nạp theo $d$ ta chứng minh được họ $\partial_{\pi}$ cảm sinh đẳng cấu $L_d/L_0$ với $\bigoplus K_nk[t]/(\pi)$ trong đó $\pi$ chạy trên các đa thức bất khả quy bậc $\leq d$. Đẩy tới giới hạn trực tiếp cho ta điều phải chứng minh.

 

Cuối cùng ta phát biểu và chứng minh bổ đề Bass-Tate cũng như đưa ra một hệ quả quan trọng của nó mà ta sẽ dùng trong chứng minh về sau khi liên hệ với đối đồng điều motivic.

 

Mệnh đề (Bass-Tate). Cho $K=k(a)$ là một mở rộng đơn sinh. Gọi $d$ là bậc của đa thức tối tiểu của $a$. Khi đó $K^M_*(K)$ xem như một $K^M_*(k)$-module sinh bởi các phần tử có dạng

$$\left \{\pi_1(a),\pi_2(a),...,\pi_m(a) \right \}$$

trong đó $\pi_i$ là các đa thức monic bất khả quy trong $k[t]$ sao cho $\mathrm{deg}(\pi_1)<\cdots <\mathrm{deg}(\pi_m) \leq d-1$.

 

Bổ đề.

 

Hệ quả. Giả sử $K/k$ là một mở rộng hữu hạn. Giả sử một trong hai giả thiết sau thoả mãn:

  • $K/k$ là một mở rộng bậc hai.
  • $[K:k]=p$ là một số nguyên tố và $k$ không có mở rộng hữu hạn không tầm thường có bậc nguyên tố cùng nhau với $p$.

Khi đó $K^M_n(K)$ xem như một $K^M_n(k)$-module sinh bởi $K^M_1(K)=K^{\times}$. Nói cách khác, ánh xạ tự nhiên $K^M_{n-1}(k) \otimes K^{\times} \longrightarrow K^M_n(K)$ là toàn cấu.




#734748 K-lý thuyết Milnor

Posted by bangbang1412 on 01-09-2022 - 22:50 in Toán học hiện đại

K-lý thuyết Milnor

 

Trong phần này mình sẽ định nghĩa K-lý thuyết Milnor và trình bày một số tính toán sơ cấp của nó. Cố định một trường $k$, kí hiệu $k^{\times}=k \setminus 0$.

 

Định nghĩa. $K$-lý thuyết Milnor $K^M_*(k)$ của $k$ là thương của đại số tensor (trên $\mathbb{Z}$)

$$\mathbb{Z} \oplus k^{\times} \oplus (k^{\times} \otimes k^{\times}) \oplus \cdots ...$$

cho ideal hai phía sinh bởi $\left \{a \otimes (1-a) \mid a \in k \setminus \left \{0,1 \right \} \right \}$. Như vậy $K$-lý thuyết Milnor là một đại số phân bậc trong đó

$$K^M_n(k) = (k^{\times} \otimes \cdots \otimes k^{\times})/\left \{a \otimes (1-a) \mid a \in k \setminus \left \{0,1 \right \} \right \},$$

trong đó tích tensor được lấy $n$ lần với $n \geq 2$.

 

Ta kí hiệu một lớp tương đương $[a_1 \otimes a_2 \otimes \cdots \otimes a_n]$ bởi $\left \{a_1,...,a_n \right \}$ và gọi nó là một $n$-kí hiệu. Bởi vì nhóm $k^{\times}$ viết theo lối nhân nên ta có:

  • $\left \{a_1,...,a_n \right \}=0$ nếu $a_i=1$ nào đó.
  • $\left \{a_1,...,aa_i,...,a_n \right \} = \left \{a_1,...,a,...,a_n \right \} + \left \{a_1,...,a_i,...,a_n \right \}$.
  • $\left \{a_1,...,a,1-a,...,a_n \right \}=0$ với $a \neq 1$.
  • $\left \{a_1,...,a_n \right \}\left \{b_1,...,b_m \right\} = \left \{a_1,...,a_n,b_1,...,b_m \right \}$.

Bổ đề. Với mọi $a \in k^{\times}$, ta có $\left \{a,-a \right \} =0$.

 

Chứng minh. Bổ đề hiển nhiên đúng nếu $a=1$, khi $a \neq 1$ thì $-a = \frac{1-a}{1-a^{-1}}$, do đó

$$\left \{a,-a \right \} = \left \{a,(1-a)(1-a^{-1})^{-1} \right \} = \left \{a,1-a \right \} - \left \{a,1-a^{-1} \right \} = \left \{a^{-1},1-a^{-1} \right \} = 0.$$

 

Bổ đề. Với mọi $a,b \in k^{\times}$, ta có $\left \{a,b\right \} = -\left \{b,a \right \}$. Hệ quả là trong $K_*^M(k)$ thì $\beta \alpha = (-1)^{\left |\alpha \right| \left |\beta \right|} \alpha \beta$.

 

Chứng minh. Ta có

$$0 = \left \{ab,-ab \right \} = \left \{a,-a\right \} + \left \{a,b \right \} + \left \{b,a \right \} + \left \{b,-b \right \} = \left \{a,b \right \} + \left \{b,a \right \}.$$

 

Bổ đề. Cho $a_1,...,a_n \in k^{\times}$, khi đó $\left \{a_1,...,a_n \right \}=0$ nếu $a_i+a_j$ bằng $0$ hoặc $1$ với một cặp chỉ số $1 \leq i \neq j \leq n$ nào đó.

 

Chứng minh. Hiển nhiên theo bổ đề trước do ta có thể di chuyển $a_i$ về cạnh $a_j$.

 

Bổ đề. Cho $a_1,...,a_n \in k^{\times}$ sao cho $a_1 + \cdots + a_n$ bằng $0$ hoặc $1$, khi đó $\left \{a_1,...,a_n \right \}=0$.

 

Chứng minh. Ta chứng minh bằng quy nạp trên $n$. Với $n=2$ thì khẳng định là hiển nhiên. Giả sử khẳng định đúng tới $n-1$ (với $n \geq 3$) và $a_1+\cdots a_n$ bằng $0$ hoặc $1$. Nếu $a_1+a_2=0$ thì khẳng định hiển nhiên theo bổ đề trước. Nếu $a_1 + a_2 \neq 0$ thì

$$1 = \frac{a_1}{a_1+a_2} + \frac{a_2}{a_1+a_2}.$$

Do đó

$$0 = (\left \{a_1 \right \} - \left \{a_1+a_2 \right \})(\left \{a_2 \right \} - \left \{a_1+a_2 \right \}).$$

Hay

$$\left \{a_1,a_2 \right \}- \left \{a_1,a_1+a_2\right\} + \left \{a_2,a_1+a_2\right \}+ \left \{a_1+a_2,a_1+a_2 \right \}=0.$$

Cuối cùng ta nhân hai vế với $\left \{a_3,...,a_n \right \}$. Khi đó ba số hạng cuối là bội của $\left \{a_1+a_2,a_3,...,a_n \right \}$, bằng $0$ theo giả thiết quy nạp. Do đó số hạng đầu tiên $\left \{a_1,a_2,...,a_n \right \}$ cũng triệt tiêu.

 

Bổ đề. Với $a \in k^{\times}$, khi đó $\left \{a \right \}^2 = \left \{a,-1 \right \} = \left \{-1,a \right \}$.

 

Chứng minh. Ta có $\left \{a \right \}^2 = \left \{a,a\right \} = \left \{a,(-1)(-a) \right \} = \left \{a,-1\right \}+\left \{a,-a\right \} = \left \{a,-1\right \}=\left \{-1,a\right \}$.

 

Cuối cùng ta lưu ý rằng $K$-lý thuyết Milnor hiển nhiên có tính hàm tử.

 

Bổ đề + Định nghĩa. Cho $F/k$ là một mở rộng trường, khi đó ánh xạ gửi mỗi $n$-kí hiệu $\alpha=\left \{a_1,...,a_n \right\} \in K^M_n(k)$ tới chính nó xem như một phần tử trong $K^M_n(F)$ xác định tốt, ta gọi nó là ánh xạ hạn chế, kí hiệu $\alpha_F$.




#734738 K-lý thuyết Milnor

Posted by bangbang1412 on 01-09-2022 - 14:33 in Toán học hiện đại

Trong topic này mình muốn giới thiệu về K-lý thuyết Milnor (Milnor's K-theory) và kết nối nó với một số lý thuyết đối đồng điều như đối đồng điều Galois, nhóm Bloch-Chow, đối đồng điều motivic. Về mặt lịch sử, ban đầu K-lý thuyết đại số (algebraic K-theory) chỉ định nghĩa được cho $K_0,K_1,K_2$ (Grothendieck định nghĩa $K_0$) và các tính toán trên các nhóm này đã rất phức tạp rồi, về sau K-lý thuyết đại số chỉ được định nghĩa và nghiên cứu một cách có hệ thống từ sau Quillen khi ông đưa lý thuyết đồng luân vào các context khác của toán học. Trước đó một định lý của Matsumoto cho ta mô tả $K_2$ cụ thể dưới dạng phần tử sinh và quan hệ, Milnor dựa trên định nghĩa này đưa ra một định nghĩa ad-hoc cho một K-lý thuyết khác, gọi là K-lý thuyết Milnor, nó chứa một phần thông tin của K-lý thuyết đại số (theo nghĩa Quillen + cổ điển) theo nghĩa sau khi tensor với $\mathbb{Q}$ nó được nhúng vào $K$-lý thuyết đại số.

 

Để thuận tiện cho người đọc, mình sẽ định nghĩa lại một số nhóm cổ điển $K_0,K_1,K_2$ và một số tính chất cơ bản (không chứng minh).

 

Nhóm K_0

 

Cố định một vành $R$ (giao hoán có đơn vị). Nhắc lại rằng một module xạ ảnh là một hạng tử trực tiếp của một module tự do nào đó.

 

Định nghĩa. Nhóm $K_0(R)$ được định nghĩa bởi công thức sau

$$K_0(R) = \bigoplus \mathbb{Z}[P]/\sim,$$

trong đó tổng trực tiếp lấy trên lớp đẳng cấu các $R$-module xạ ảnh hữu hạn sinh, quan hệ $\sim$ được cho bởi $[P] + [Q] = [P \oplus Q]$. Ta cũng có thể trang bị cho $K_0(R)$ một cấu trúc vành bởi tích tensor $[P][Q] = [P \otimes Q]$, điều này có được do tích tensor của hai module xạ ảnh hữu hạn sinh cũng là một module xạ ảnh hữu hạn sinh. Như vậy thực chất $K_0(R)$ là một vành.

 

Lưu ý rằng xây dựng $K_0$ có tính hàm tử, tức là nếu $f: R \longrightarrow R'$ là một đồng cấu vành thì ta có một đồng cấu vành tự nhiên $f_*:K_0(R) \longrightarrow K_0(R')$ cho bởi phép đổi cơ sở $[P] \longmapsto [R' \otimes_R P]$. Như vậy nói chung mọi vành $R$ ta có một đồng cấu $K_0(\mathbb{Z}) \longrightarrow K_0(R)$ do $\mathbb{Z}$ là vật đầu trong phạm trù vành giao hoán.

Ví dụ.

  • Khi $R=k$ là một trường thì mọi module hữu hạn sinh là một không gian vector hữu hạn chiều, xác định chính xác tới một đẳng cấu bằng số chiều. Như vậy ánh xạ $K_0(k) \longrightarrow \mathbb{R}, V \longmapsto \dim_k(V)$ là một đẳng cấu.
  • Khi $R$ là một vành địa phương thì định lý của Kaplansky nói rằng mọi module xạ ảnh hữu hạn sinh trên $R$ là tự do, chứng minh $K_0(R) \simeq \mathbb{Z}$.

Giờ giả sử $R$ được nhúng vào một trường $k$ (luôn làm được ví dụ khi $R$ nguyên, $k=\mathrm{Frac}(R)$ trường các thương của $R$) thì ta có một phân tích

$$K_0(R) \simeq \mathbb{Z} \oplus \mathrm{Ker}(K_0(R) \longrightarrow \mathbb{Z})$$

do $K_0(R) \longrightarrow K_0(k)$ có một chẻ chính là đồng cấu $K_0(\mathbb{Z}) \longrightarrow K_0(R)$. Hạng tử $\mathrm{Ker}(K_0(R) \longrightarrow \mathbb{Z})$ được kí hiệu bởi $\widetilde{K_0}(R)$ và gọi là nhóm $K_0$ rút gọn của $R$.

 

Một lớp vành khác mà ta có thể tính nhóm $K_0$ là các miền Dedekind (miền Noether, đóng nguyên, chiều Krull một).

 

Mệnh đề. Cho $R$ là một miền Dedekind, khi đó $K_0(R) \simeq \mathbb{Z} \oplus \widetilde{K_0}(R)$ trong đó $\widetilde{K_0}(R)$ đẳng cấu với nhóm lớp ideal của $R$. Hơn nữa, tích hai phần tử bất kì trong nhóm rút gọn bằng không.
 

Nhóm Whitehead $K_1$

 

Cố định vành giao hoán có đơn vị $R$. Kí hiệu $GL(n,R)$ bởi nhóm tuyến tính tổng quát cỡ $n$ trên $R$. Nhóm $GL(n,R)$ được nhúng vào nhóm $GL(n+1,R)$ bởi

$$A \longmapsto \begin{pmatrix}
A & 0 \\
 0 &1
\end{pmatrix}$$

Định nghĩa nhóm tuyến tính tổng quát $GL(R)$ là giới hạn (hay hợp thành) trực tiếp của dãy $(GL(n,R))_{n \geq 0}$. Nhóm $GL(R)$ có một tính chất rất đặc biệt, đó là nhóm con $E(R)$ sinh bởi các ma trận cơ bản (elementary matrices) chính là nhóm giao hoán tử của $GL(R)$, do đó là một nhóm con chuẩn tắc.

 

Định nghĩa. Nhóm Whitehead $K_1(R)$ được định nghĩa là abel hoá $GL(R)^{ab} = GL(R)/E(R)$ của nhóm tuyến tính vô hạn.

 

Lưu ý rằng nhóm tuyến tính và phép abel hoá đều có tính hàm tử nên $K_1(-)$ có tính hàm tử.

 

Nhóm Steinberg và hàm tử $K_2$

 

Cố định một vành giao hoán có đơn vị $R$. Kí hiệu $GL(n,R)$ bởi nhóm tuyến tính tổng quát cỡ $n$ trên $R$. Với $1 \leq i,j \leq n, \lambda \in R$ ta có các ma trận sơ cấp $E^{\lambda}_{i,j}=\mathbb{1}+A^{\lambda}_{i,j}$  trong đó $A^{\lambda}_{i,j}$ có tất cả vị trí bằng $0$ ngoại trừ vị trí $(i,j)$ là $\lambda$. Có thể dễ chứng minh các đẳng thức dưới đây

$$E^{\lambda}_{i,j}E^{\mu}_{i,j} = E^{\lambda+\mu}_{i,j}, \ \ [E_{i,j}^{\lambda},E^{\mu}_{k,l}] = \begin{cases} 1 & j \neq k, i \neq l, \\ E^{\lambda \mu}_{i,l} & j = k, i\neq l, \\ E^{-\mu\lambda}_{k,j} & j\neq k, i = l. \end{cases}$$

Trong đó $[a,b]=aba^{-1}b^{-1}$ là giao hoán tử.

 

Định nghĩa. Với $n \geq 3$, nhóm Steinberg $St(n,R)$ được định nghĩa là nhóm tự do trên các kí hiệu $X^{\lambda}_{i,j}$ với $\lambda \in R, 1 \leq i,j \leq n$ chia thương cho quan hệ

$$X^{\lambda}_{i,j}X^{\mu}_{i,j} = X^{\lambda+\mu}_{i,j}, \ \ [X_{i,j}^{\lambda},X^{\mu}_{k,l}] = \begin{cases} 1 & j \neq k, i \neq l, \\ X^{\lambda \mu}_{i,l} & j = k, i\neq l, \\ X^{-\mu\lambda}_{k,j} & j\neq k, i = l. \end{cases}$$

Nhắc lại từ phép nhúng $GL(n,R) \longrightarrow GL(n+1,R)$ ta có phép nhúng tương ứng $St(n,R) \longrightarrow St(n+1,R)$ và do đó lấy giới hạn cho ta nhóm Steinberg vô hạn và một đồng cấu $St(R) \longrightarrow GL(R)$ thoả mãn ảnh của đồng cấu này chính là nhóm $E(R)$ các giao hoán tử của $GL(R)$.

 

Định nghĩa. Nhóm $K_2(R)$ được định nghĩa là $\mathrm{Ker}(St(R) \longrightarrow GL(R))$. Như vậy dễ thấy $K_2(-)$ có tính hàm tử.

 

Một định lý không tầm thường nói rằng là hạt nhân của $St(R)$. Như vậy $K_2(R)$ là nhóm abel và ta có một dãy khớp

$$1 \longrightarrow K_2(R) \longrightarrow St(R) \longrightarrow GL(R) \longrightarrow K_1(R) \longrightarrow 1.$$

 

(còn tiếp)




#732075 Vấn đề Hilbert thứ 21

Posted by bangbang1412 on 17-12-2021 - 06:40 in Toán học hiện đại

Vấn đề Hilbert thứ 21 dự đoán sự tồn tại của một hệ phương trình vi phân với nhóm monodromy cho trước. Bài viết này giới thiệu sơ lược về vấn đề Hilbert thứ 21 và các khái niệm liên quan. Setting của chúng ta trong bài toán này là lấy một đường cong $X$ xạ ảnh, không suy biến, liên thông trên $\mathbb{C}$. Gọi $U$ là một tập mở trong $X$ sao cho phần bù của $U$ là một số hữu hạn các điểm đóng. Ký hiệu $U^{an}$ là đa tạp phức ứng với $U$ trong nguyên lý GAGA. Trước tiên chúng ta xem xét một định nghĩa hợp lý của phương trình vi phân.

 

Ví dụ $X = \mathbb{P}^1$ và $U \subset \mathbb{P}^1 - \left \{\infty \right \}$, khi đó một hệ phương trình vi phân được mô tả bởi

$$\frac{d}{dz}\mathbf{f} = P(z) \cdot \mathbf{f},$$

trong đó $P$ là một ma trận các hàm hệ số. Hệ này bao gồm lớp các phương trình vi phân tuyến tính cấp $n$

$$f^{(n)}= p_{n-1}f^{(n-1)}+\cdots +  p_1f' + p_0f,$$

bằng cách lấy $P$ là ma trận

$$  P(z)=  \begin{pmatrix}
0 & 1 & ... & 0 &0 \\
0 & . & 1 & 0 & 0 \\
\vdots &  & \ddots & \ddots & \vdots\\
0 & 0 &  & 0 & 1\\
p_0 & p_1 & \cdots &  p_{n-2} & p_{n-1}
\end{pmatrix}$$

Trong trường hợp đường cong có giống cao hơn, do không có một hệ tọa độ toàn cục nên ta sẽ định nghĩa một hệ phương trình vi phân trên $U$ là một cặp $(M,\nabla)$ trong đó $M$ là một bó nhất quán tự do địa phương $M$ và một liên thông $\nabla: M \to M \otimes \Omega^{1}_{U/\mathbb{C}}$ thỏa mãn luật Leibniz giống như trường hợp trên đa tạp. Khi đó một phương trình vi phân sẽ được định nghĩa là hạt nhân của liên thông $\nabla$. Để giải thích cho định nghĩa này ta xét một ví dụ giải tích

 

Ví dụ. Ký hiệu $\pi: \mathbb{C}^2 \times \mathbb{C} \to \mathbb{C}$ là phân thớ tầm thường hạng $2$ trên $\mathbb{C}$. Khi đó mọi liên thông trên $\pi$ có dạng

$$\nabla = d+ \begin{pmatrix}
 -f_{11}(z)& -f_{12}(z)\\
 -f_{21}(z)& -f_{22}(z)
\end{pmatrix}dz$$

trong đó $d$ là đạo hàm ngoài và $f_{ij}$ là các hàm chỉnh hình. Một lát cắt $a \in \Gamma(s)$ có thể xem như một cặp $(a_1(z),a_2(z))$ gồm hai hàm chỉnh hình. Khi đó

$$ \nabla(a) = \nabla \begin{pmatrix}
a_1(z)\\
a_2(z)
\end{pmatrix} = \begin{pmatrix}
a_1'(z) -f_{11}(z)a_1(z) - f_{12}(z)a_2(z)\\
a'_2(z) - f_{21}(z)a_1(z) - f_{22}(z)a_2(z)
\end{pmatrix}dz.$$

Do đó ta thấy hạt nhân của $\nabla$ cho ta một hệ phương trình cấp $2$.

 

Cố định một điểm $z_0 \in U$, khi đó mầm $S$ của các nghiệm chỉnh hình địa phương gần $z_0$ là một không gian vector có chiều là $\mathrm{rank}(M)$ theo định lý giải được địa phương của hệ phương trình vi phân bậc nhất. Cho $\gamma$ là một đường cong đóng tại $z_0$ trong $U^{an}$, khi đó thác triển giải tích dọc theo $\gamma$ cho định nghĩa cho ta một tự đẳng cấu của $S$. Như vậy ta có một tác động $\pi_1(U^{an},z_0)$ lên $S$, ta gọi đây là biểu diễn monodromy của phương trình vi phân $(M,\nabla)$.

 

Ví dụ. Xét phương trình vi phân

$$ z\frac{df}{dz} = \alpha f, \ \alpha \in \mathbb{C}$$

trên mặt phẳng thủng $\mathbb{C} - \left \{0 \right \}$. Cố định một điểm và chọn một branch cut xuất phát từ điểm này, khi đó nghiệm toàn cục có dạng $z^{\alpha}=\mathrm{exp}(\alpha\log(z))$. Nếu ta xét thác triển giải tích của lớp đồng luân của đường cong $\gamma$ quay ngược chiều kim đồng hồ một góc $2\pi$ thì nghiệm sẽ trở thành $e^{2\pi \mathbf{i} \alpha}z^{\alpha}$. Ta thấy $\pi_1(\mathbb{C} - \left \{0 \right \}) \cong \mathbb{Z}$ với phần tử sinh $[\gamma]$, khi đó biểu diễn monodromy tương ứng là $\gamma \mapsto e^{2\pi \mathbf{i}\alpha} \in \mathbb{C}^{\times} = \mathrm{GL}(1,\mathbb{C})$.

 

Để có thể phát biểu hoàn toàn bài toán Hilbert, ta cần khái niệm điểm kỳ dị chính quy, nó giống như một điều kiện đầu của bài toán Cauchy để phương trình có nghiệm duy nhất. Ban đầu, khái niệm kỳ dị chính quy đến từ ODE, sau đó Fuchs định nghĩa nó bằng một số đánh giá giải tích nhưng cuối cùng ông chứng minh rằng khái niệm này là thuần túy đại số, ta sẽ dùng cách định nghĩa này ở đây.

 

Xét một phương trình vi phân thường biến $z \in \mathbb{C}$

$$ f^{(n)} =p_{n-1}f^{(n-1)}+\cdots +  p_1f' + p_0f,$$

trong đó $p_i$ là các hàm phân hình, Ta nói $a \in \mathbb{C}$ là một điểm kỳ dị chính quy nếu $p_{n-i}$ có một cực bậc không quá $i$ tại $a$. Trong trường hợp đó phương trình của chúng ta có thể chuyển về dạng

$$D^{(n)}f = b_{n-1}D^{(n-1)}f + ... + b_0 f,$$

trong đó $D = (z-a)\frac{d}{dz}$ là uniformizer tại $a$. Phương trình ban đầu có kỳ dị chính quy tại $a$ khi và chỉ khi $b_i$ đều là hàm chỉnh hình.

 

Ví dụ. Nếu $p,q$ là hai hàm chỉnh hình, khi đó phương trình $f^{"}(z) =\frac{p(z)}{z}f'(z)+\frac{q(z)}{z^2}f(z)$ tương đương với $D^2f = (p+1)Df + qf$ và do đó nó có điểm kỳ dị chính quy tại $z=0$.

 

Lưu ý. Một khi biết phương trình có điểm kỳ dị chính quy tại $a$, phương pháp Frobenius có thể được sử dụng để giải ra $n$ nghiệm địa phương dưới dạng chuỗi.

 

Bài toán Hilbert thứ 21. Cho $X$ là một đường cong không suy biến trên $\mathbb{C}$, $U$ là một tập mở với topo Zariski sao cho phần bù của $U$ là một số hữu hạn các điểm đóng. Hỏi rằng có phải mọi biểu diễn hữu hạn chiều của $\pi_1(U^{an})$ đều là một biểu diễn monodromy của một hệ phương trình vi phân có kỳ dị chính quy mọi nơi không?

 

Nếu ta bỏ điều kiện điểm kỳ dị chính quy, biểu diễn của ta có thể không đến từ một hệ phương trình duy nhất.

 

Ví dụ. Với $U = \mathbb{A}^1, U^{an} = \mathbb{C}$ và $\pi_1(U^{an}) = 0 \to \mathbb{C}^{\times}$ là biểu diễn tầm thường. Khi đó với mọi $P \in \mathbb{C}[z]$ thì phương trình

$$\frac{df}{dz} = P(z)f$$

có nghiệm

$$f(z) = \mathrm{exp}\left(\int_0^z f(t)dt \right)$$

là một nghiệm toàn cục, và do đó không có monodromy. Nhưng xem như các phương trình vi phân trên đa tạp đại số $\mathbb{A}^1$, các phương trình này đôi một không đẳng cấu. Hơn nữa nếu ta dùng uniformizer $D=(z-a)\frac{d}{dz}$ thì phương trình trở thành $Df = (z-a)P(z)f$, nếu ta yêu cầu nó có kỳ dị chính quy tại $a$ thì $(z-a)P(z)$ phải có cực bậc dương tại $a$, điều này chỉ xảy ra nếu $P \equiv 0$.

 

Để kiểm tra một hệ có kỳ dị chính quy không, Katz đã chứng minh nó tương đương với việc tìm các cyclic vector, tức là các vector "biểu hiện" như một nghiệm địa phương của phương trình vi phân. Bài toán tìm cyclic đã được giải quyết gần như trọn vẹn.




#732107 Vấn đề Hilbert thứ 21

Posted by bangbang1412 on 17-12-2021 - 19:35 in Toán học hiện đại

Khái niệm tổng quát về phương trình vi phân. Cho $X$ là một đường cong đại số không suy biến, liên thông trên $\mathbb{C}$. Một phương trình vi phân trên $X$ là một cặp $(M,\nabla)$ bao gồm một bó nhất quát tự do địa phương $M$ trên $X$ và một liên thông integrable

$$\nabla: M \to M \otimes \Omega^1_{X/\mathbb{C}},$$

tức là một ánh xạ cộng tính thỏa mãn luật $\nabla(fm)=m.df + f\nabla(m)$ với mọi $f \in \mathcal{O}_X, m \in M$; integrable theo nghĩa nếu ta tác động $D \in \mathrm{Der}(X/\mathbb{C})$ (các đạo hàm $X \to X$)  lên $M$ bằng hợp thành:

$$\nabla(D): M \overset{\nabla}{\longrightarrow} M \otimes \Omega^1_{X/\mathbb{C}} \overset{\mathrm{id} \otimes \text{contraction}}{\longrightarrow} M$$

thì ta yêu cầu rằng $\nabla([D_1,D_2]) = [\nabla(D_1),\nabla(D_2)]$. Tương tự, trên đa tạp phức ta định nghĩa phương trình vi phân là một liên thông integrable trên bó tự do nhất quán giải tích địa phương. Ta sẽ thấy về sau tại sau lại điều kiện integrable.

 

Trường hợp giải tích. Lời giải của bài toán Hilbert thứ 21 trong trường hợp giải tích được giải quyết trọn vẹn và câu trả lời là , tức là: cho $V$ là một đa tạp phức liên thông bất kỳ, khi đó mọi biểu diễn hữu hạn chiều của $\pi_1(V)$ đều đến từ một biểu diễn monodromy của một phương trình vi phân trên $V$. Điều kiên integrable để ta có thể áp dụng định lý Frobenius về tính tốn tại nghiệm địa phương của hệ phương trình vi phân.

 

Giải kỳ dị. Ở đây ta sử dụng một phiên bản giải kỳ dị của Hironaka

 

Cho $U$ là một đa tạp tựa xạ ảnh (quasi-projective) trên $\mathbb{C}$. Khi đó tồn tại một đa tạp trơn $X/\mathbb{C}$ sao cho $U \subset X$ là một tập mở trù mật và phần bù $D = X - U$ là hợp thành của các ước trơn $D_1,...,D_r$ (đa tạp con đóng đối chiều một) giao nhau tranversely. Ta gọi $X$ là một compact hóa của $U$. Hai compact hóa bất kỳ đều bị áp đảo bởi một compact hóa thứ ba.

 

Ký hiệu $\mathrm{Der}_D(X/\mathbb{C})$ là bó con của $\mathrm{Der}(X/\mathbb{C})$ gồm các đạo hàm bảo toàn bó ideal của $D_1,...,D_r$. Tại một điểm mà $D_1,...,D_r$ giao nhau ta có thể chọn một hệ tọa độ địa phương sao cho $D_i = \left \{x_i =0\right \}$. Khi đó $\mathrm{Der}_D(X/\mathbb{C})$ là module tự do trên các ký hiệu $x_1\frac{\partial}{\partial x_1},...,x_r\frac{\partial}{\partial x_r},\frac{\partial}{\partial x_{r+1}},...,\frac{\partial}{\partial x_n}$. Trường hợp một chiều, tức là các stalk là DVR thì điều kiện bảo toàn ideal nói rằng $\text{đạo hàm}(\mathfrak{m}) \subset \mathfrak{m}$; nói cách khác, nó phải triệt tiêu tại $x_i$ với bậc $\geq 1$ và do đó nằm trong module sinh bởi $x\frac{\partial}{\partial x}$.

 

Đối ngẫu tuyến tính cua $\mathrm{Der}_D(X/\mathbb{C})$ được ký hiệu là $\Omega^1_{X/\mathbb{C}}(\log(D))$ tự do địa phương bởi các kỳ hiệu $\frac{dx_1}{x_1},...,\frac{dx_r}{x_r},dx_{r+1},...,dx_n$, nó bao gồm các một dạng với các cực bậc một của $D$.

 

Phát biểu lại tiêu chuẩn tìm điểm kỳ dị chính quy. Bằng hai bó $\mathrm{Der}_D(X/\mathbb{C})$ và $\Omega^1_{X/\mathbb{C}}(\log(D))$ ta sẽ viết lại điều kiện điểm kỳ dị chính dị quy: một phương trình vi phân $(M,\nabla)$ trên một đường cong $U$ (mở trong $X$) có điểm kỳ dị chính quy trên $D = X - U$ tương đương với tồn tại một bó $\overline{M}$ trên $X$ mở rộng $M$ sao cho tác động của $\mathrm{Der}_D(X/\mathbb{C})$ trên $U$ mở rộng lên một tác động của $\mathrm{Der}_D(X/\mathbb{C})$ trên $\overline{M}$. Nói cách khác

$$\nabla: M \to M \otimes \Omega^1_{U/\mathbb{C}} \overset{\text{mở rộng}}{\longrightarrow} \overline{\nabla}: \overline{M} \to \overline{M} \otimes \Omega^1_{X/\mathbb{C}}(\log(D)).$$

 

Đưa bài toán về dạng giải tích. Cho $U$ là một đa tạp trơn, liên thông, tựa xạ ảnh trên $\mathbb{C}$, ta biết rằng trường hợp giải tích đã được xử lý nên một biểu diễn hữu hạn chiều của $U^{an}$ cảm sinh từ một biểu diễn monodromy của duy nhất một phương trình vi phân $(M^{an},\nabla^{an})$. Trước tiên ta thừa nhận bổ đề sau

 

Bổ đề. Cho $(M^{an},\nabla^{an})$ là một phương trình vi phân giải tích trên $U^{an}$ và $U \hookrightarrow X$ là một compact hóa của $U$. Khi đó tồn tại một bó giải tích nhất quán tự do địa phương $\overline{M^{an}}$ trên $X^{an}$ mở rộng $M^{an}$, sao cho $\nabla^{an}$ thác triển lên $\overline{\nabla^{an}}:\overline{M^{an}} \to \overline{M^{an}} \otimes \left(\Omega^1_{X/\mathbb{C}}(\log(D))\right)^{an}.$

 

Giả sử đã chứng minh được bổ đề, khi đó ta áp dụng nguyên lý GAGA cho $(\overline{M^{an}},\overline{\nabla^{an}})$ để suy ra tồn tại $\overline{M}$ trên $X$ và $\overline{\nabla}:\overline{M} \to \overline{M} \otimes \Omega^1_{X/\mathbb{C}}(\log(D))$ sao cho $(\overline{M^{an}},\overline{\nabla^{an}}) = (\overline{M},\overline{\nabla})^{an}$. Khi này ta có thể định nghĩa $(M,\nabla)$ là hạn chế của $(\overline{M},\overline{\nabla})$ lên $U$ thì hiển nhiên $(M,\nabla)$ có kỳ dị chính quy và $(M,\nabla)^{an} = (M^{an},\nabla^{an})$ đồng thời $(M,\nabla)$ cho ta biểu diễn của $\pi_1(U^{an})$.

 

Chứng minh bổ đề. Tại một điểm "$0$" mà $D_1,...,D_r$ giao nhau, ta có thể chọn một hệ tọa độ $x_1,..,x_n$ sao cho $D_i = \left \{x_i = 0\right \}$ với mọi $1 \leq i \leq r$. Trên một hệ tọa độ dạng đĩa đa diện $V$ quanh điểm giao có dạng $\left | x_i \right | < \epsilon$ với mọi $i = 1,...,n$ ($\epsilon$ đủ nhỏ) thì đa tạp $V \cap U^{an}$ có dạng

$$V \cap U^{an} = \prod_{i=1}^r \left \{ 0 < \left|x_i \right |< \epsilon \right \} \times \prod_{i=r+1}^n \left \{\left|x_i \right| < \epsilon \right \}.$$

Hạn chế của $(M^{an},\nabla^{an})$ xuống $V \cap U^{an}$ là một phương trình vi phân trên $V \cap U^{an}$ nên theo trường hợp giải tích, nó ứng với một biểu diễn $\rho:\pi_1(V \cap U^{an}) \to \mathrm{GL}(L)$ trong đó $L$ là một không gian vector hữu hạn chiều. Tuy nhiên cách chọn hệ tọa độ cho thấy

$$\pi_1(V \cap U^{an}) = \mathbb{Z}\gamma_1 \times \cdots \times \mathbb{Z}\gamma_r,$$

trong đó $\gamma_i$ là các đường cong đóng quay ngược chiều kim đồng hồ quanh $D_i$. Do đó biểu diễn $\rho$ xác định hoàn toàn nếu ta biết $\rho(\gamma_1),..., \rho(\gamma_r)$. Bằng cách sử dụng dạng chuẩn Jordan, ta thấy rằng tồn tại duy nhất các tự đẳng cấu $B_i$ của $L$ sao cho

  • $\mathrm{exp}(2\pi \mathbf{i} B_j) = \rho(\gamma_j)$ với mọi $j = 1,2,...,r$.
  • Các giá trị riêng của $B_j$ có phần thực nằm trong giải $-1 < \mathrm{Re} \leq 0$.
  • $B_j$ giao hoán từng đôi một.

Ta sẽ định nghĩa mở rộng lên $V$ của $(M^{an},V^{an})$ như sau

$$\overline{M^{an}} = L \otimes_{\mathbb{C}} \mathcal{O}_V,$$

và ta cũng định nghĩa mở rộng của tác động như sau

$$\begin{align*} \overline{\nabla^{an}}: \overline{M^{an}} = L\otimes \mathcal{O}_V & \to \overline{M^{an}} \otimes_{\mathcal{O}_V} \Omega^1_V(\log(D)) = L \otimes \Omega^1_{V/\mathbb{C}}(\log(D)) \\ l \otimes f & \mapsto f\left(-\sum_{i=1}^r B_i l \otimes \frac{dx_i}{x_i} \right) + l \otimes df \end{align*}$$

Để kiểm tra rằng liệu $(\overline{M^{an}},\overline{\nabla^{an}})$ thực sự mở rộng $(M^{an},\nabla^{an})$ ta cần kiểm tra liệu monodromy của nó có chính xác không. Như vậy ta cần tìm ma trận $A$ trong biểu diễn địa phương $d - A$ của $\overline{\nabla^{an}}$, dễ thấy ma trận này chính là $\sum_{i=1}^r \log(x_i)B_i$ ($\log(x_i)$ định nghĩa tốt trên $V$ do $V$ là tích các đĩa một chiều mở, do đó đơn liên) và do đó nghiệm cơ bản là $\mathrm{exp}\left(\sum_{i=1}^r \log(x_i)B_i \right)$ (ta sẽ ký hiệu là $\prod_{i=1}^r x_i^{B_i}$). Nếu ta xét thác triển giải tích dọc theo các đường cong $\gamma_i$ thì nghiệm sẽ trở thành $\mathrm{exp}\left(\sum_{i=1}^r \log(x_i)B_i + 2\pi \mathbf{i}B_i \right) = \rho(\gamma_i)\prod_{i=1}^r x_i^{B_i}$, và do đó chính là monodromy tương ứng.

 

Để kiểm tra định nghĩa của ta tốt, ta cần phải chứng định nghĩa địa phương này có thể dán thành toàn cục. Ta sẽ sử dụng tiêu chuẩn địa phương sau (không chứng minh):

 

Trong mọi cơ sở của $\overline{M^{an}}$ xem như một $\mathcal{O}_V$ module thì nghiệm cơ bản luôn có dạng $H(x)\prod_{i=1}^r x_i^{B_i}$ trong đó $H(x) \in \mathrm{GL}(n,\mathcal{O}_V)$ và $B_i$ là các ma trận đôi một giao hoán, có các giá trị riêng với phần thực nằm trong đoạn $(-1,0]$.

 

Giờ ta quay lại một hệ tọa độ $x_1,...,x_n$ dạng đa đĩa trên $V$, nghiệm cơ bản của hệ có dạng $\prod_{i=1}^r x_i^{B_i}$. Giả sử trên $V' \subset V$ thì $D_i$ có thể viết dưới dạng $D_i = \left \{y_i \right \}$ với $i=1,...,s$ ($s < r$), ta cần chứng minh nghiệm cơ bản vừa trên có thể biểu diễn dưới dạng $H(x)\prod_{i=1}^s y_i^{B_i}$. Với $i=1,...,r$, do $x_i,y_i$ cùng định nghĩa $D_i$ trên $V'$ nên tồn tại các hàm khả nghịch $u_i$ mà $x_i = u_i y_i$. Các ước $D_{s+1},...,D_r$ tự thân chúng khả nghịch trong lân cận này. Từ đây ta suy ra rằng có thể viết

$$u_i = \mathrm{exp}(z_i) \ \forall \ i = 1,...,s \ \text{và} \ x_i = \mathrm{exp}(z_i) \ \forall \ i = s+1,...,r.$$

Cuối cùng ta có

$$\prod_{i=1}^r x_i^{B_i} = \prod_{i=1}^s (u_i y_i)^{B_i} \prod_{i=s+1}^r x_i^{B_i} = \mathrm{exp}\left(\sum_{i=1}^r B_i z_i \right)\prod_{i=1}^s y_i^{B_i},$$

trong đó hiển nhiên $\mathrm{exp}\left(\sum_{i=1}^r B_i z_i \right) \in \mathrm{GL}(n,\mathcal{O}_{V'})$. Kết thúc chứng minh.




#733234 Motivic integration: an introduction

Posted by bangbang1412 on 13-04-2022 - 18:17 in Toán học hiện đại

Grothendieck ring of varieties

 

Definition 9. Let $S$ be a ring. A motivic measure $\lambda$ from the category $\mathrm{Var}_k$ with values in $S$, assigns to any $X$ in $\mathrm{Var}_k$ an element $\lambda(X)$ of $S$ such that:

  • $\lambda([\mathrm{Spec}(k)]) = 1$.
  • $\lambda([X]) = \lambda([Y]) + \lambda([X \setminus Y])$ for $Y$ closed in $X$.
  • $\lambda([X][Y]) = \lambda([X])\lambda([Y])$ for $X, Y \in \mathrm{ob}(\mathrm{Var}_k)$.

 

Remark. Any motivic measure $\lambda$ naturally extend to take its values on constructible subsets of algebraic varieties. Indeed a constructible subset $W$ maybe written as a finite disjoint union of locally closed subvarieties $Z_i$ and hence we can define $\lambda(W)$ to be $\sum \lambda(Z_i)$. By the very axioms, this is independent of the choice of the decomposition into locally closed subvarieties.

 

Example 10. Let $k$ be a finite field and $K/k$ a finite extension, then the additive invariant $[X] \longmapsto \left | X(K) \right|$ is a motivic measure.

 

Example 11. Let $l$ be a prime number distinct from the characteristic of $k$. The assignment $$[X] \longmapsto \sum_{i=0}^{2\dim(X)}(-1)^i H_{c}^n(X,\mathbb{Q}_l)$$ where $H_c^i$ denote the $i^{th}$ $l$-adic cohomology with compact support, defines a motivic measure. This also works effectively for every other classic cohomology theories, e.g., Hodge theory, crystalline cohomology.

 

Example 12. Let us assume $k$ is a field of characterisitc zero. It follows from Deligne's mixed Hodge theory that there is a unique motivic measure $H: \mathrm{Var}_k \longrightarrow \mathbb{Z}[u,v]$, which assigns each smooth projective variety $X$ over $k$ its Hodge polynomial:
\begin{equation*}
    H(X,u,v) = \sum_{p,q}(-1)^{p+q}h^{p,q}(X)u^p v^q
\end{equation*} where $h^{p,q}(X) = \dim H^q(X,\Omega^p)$ is the $(p,q)$-Hodge number of $X$. For instance, if $k = \mathbb{C}$ then $H(\mathbb{P}^n,u,v) = \sum_{i=0}^n (uv)^i$. The uniqueness of $H$ is highly nontrivial to be proved; it is a consequence of the fact that $K_0(\mathrm{Var}_k)$ (defined below) is generated by classes of smooth proper $k$-varieties.

 

Definition 14. In the sequel, $[-]$ always denotes the isomorphism class of some object in a appropriate category. We denote by $\mathbb{Z}[\mathrm{Var}_S]$ the free abelian group on $\mathrm{Var}_S$. The Grothendieck ring over $S$ $K_0(\mathrm{Var}_S)$ is the quotient of $\mathbb{Z}[\mathrm{Var}_S]$ by its subgroup, generated by element of the form $[X] - [Z] - [X \setminus Z]$, where $X$ is a $S$-scheme of finite presentation, and $Z$ a closed subscheme of $X$. The fiber product over $S$ induces, by linearity, a ring structure on it by setting $[X] \cdot [Y] = [X \times_S Y]$. We note $\mathbf{L} = [\mathbb{A}_S^1]$ and $\mathscr{M}_S = K_0(\mathrm{Var}_S)[\mathbf{L}^{-1}]$ the localization of $K_0(\mathrm{Var}_S)$ by the element $\mathbf{L}$. When $S = \mathrm{Spec}(k)$ with $k$ a field, we simply write $\mathscr{M}_k$ instead of $\mathscr{M}_{\mathrm{Spec}(k)}$.
    
We denote by $\mathscr{M}_k[T]_{loc}$ the subring of $\mathscr{M}_k[[T]]$ generated by $\mathscr{M}_k[T]$ and the series $(1 - \mathbb{L}^a T^b)^{-1}$ with $a \in \mathbb{Z}$ and $b \in \mathbb{N} \setminus \left \{0 \right \}$.

 

Remark.

  • It is evident from the very definition of $K_0(\mathrm{Var}_k)$ that it possesses a universal property: any motivic measure on $\mathrm{Var}_k$ factors through $K_0(\mathrm{Var}_k)$.
  • $K_0(\mathrm{Var}_{\mathbb{C}})$ is not a domain. It was showed that there exist two abelian varieties $A, B$ such that $[A] \neq [B]$ but $A \times A \simeq B \times B$, which implies $([A] - [B])([A] + [B]) = [A]^2 - [B]^2 = 0$.
  • It is proved that if $k$ has characteristic zero, then $K_0(\mathrm{Var}_k)$ can be generated by isomorphism classes of irreducible smooth projective varieties subject to the blow-up relations.

Example 15. $[\mathbb{P}_k^n] = \mathbb{L}^n + \mathbb{L}^{n-1} + \cdots + \mathbb{L} + 1$.

 

Example 16. For $k$ is algebraically closed and $X = V(x^3 - y^2)$ in $\mathbb{A}^2_k$, then $[X] = [\mathbb{A}_k^1 \setminus \left \{0\right \}] + [\left \{0 \right \}] = [\mathbb{A}^1_k] = \mathbb{L}$.

 

Example 17. For any $f: Y \longrightarrow X$ a piecewise trivial fibration with fiber $F$, i.e., $X = \coprod X_i$ locally closed and $f_{\mid f^{-1}(X_i)}: f^{-1}(X_i) \longrightarrow X_i$ is of the form (precisely, isomorphic to) $X_i \times F \longrightarrow X_i$, then:
\begin{equation*}
    [Y] = \sum [f^{-1}(X_i)] = [F] \sum [X_i] = [F][X].
\end{equation*}
For instance, for a reasonable $k$-scheme $X$, $[J_m(X)] = \mathbb{L}^{m\dim(X)}[X]$.

 

Definition 18. Let $X$ be an algebraic $k$-variety of dimension $d$ (maybe singular). A subset $C \subset J_{\infty}(X)$ is called a cylinder set if $C = \pi^{-1}_n(B_n)$ for some $n \in \mathbb{N}$ and $B_n \subset J_n(X)$ is a constructible subset, i.e., a finite disjoint union of locally closed subvarieties.

 

Remark. In a paper of Denef and Loeser, it has been proven that for any algebraic variety $X$ (not necessarily smooth), $\pi_n(J_{\infty}(X))$ is constructible for any $n \in \mathbb{N}$.

 

Definition 19. Let $X$ be a smooth $k$-variety of dimension $d$ and $C = \pi^{-1}(B_n)$ a cylinder set where $B_n \subset J_n(X)$ a constructible subset. The function:
\begin{align*}
    \widetilde{\mu}: \left \{\text{cylinder sets in} \ J_{\infty}(X) \right \} & \longrightarrow \mathscr{M}_k \\
    \pi^{-1}_n(B_n) & \longmapsto \frac{[B_n]}{\mathbb{L}^{(n+1)d}}.
\end{align*}
It is straightforward to show that $\widetilde{\mu}$ is a finitely additive measure.




#733231 Motivic integration: an introduction

Posted by bangbang1412 on 13-04-2022 - 18:10 in Toán học hiện đại

In this topic, I introduce the notion of the so-called motivic integration, which is an upgrade version of the old version, namely, the p-adic integration. The word motivic literally means the values of this integration is essentially geometric. It was introduced by M. Kontsevich in his lecture in Orsay in 1995 to solve a theorem of Bartyrev stating that two birational Calabi-Yau varieties have the same Betti numbers.

 

Let $S$ be a scheme. By a $S$-algebraic variety, we mean a $S$-scheme of finite presentation. We denote by $\mathrm{Var}_S$ the isomorphism classes of finite presentation $S$-schemes. When $S = \mathrm{Spec}(k)$ with $k$ a field, we simply write $\mathrm{Var}_k$ instead of $\mathrm{Var}_{\mathrm{Spec}(k)}$.

 

Jet scheme and arc space

 

Let $X$ be a $k$-variety.

 

Proposition 1. For $m \in \mathbb{N}$, there exists an algebraic $k$-variety $J_m(X)$ such that:
\begin{equation*}
    \mathrm{Hom}_k(Z \times \mathrm{Spec}(k[t]/(t^{m+1})), X) \simeq \mathrm{Hom}_k(Z,J_m(X))
\end{equation*} for any $k$-scheme $Z$.

 

Proof. It is sufficient to deal with the case $X, Z$ are affine, i.e., $X = \mathrm{Spec}(R)$ and $Z = \mathrm{Spec}(A)$ for some $k$-algebra $R$ and some finitely generated $k$-algebra $R = k[x_1,...,x_n]/(f_1,...,f_r)$.
\begin{equation*}
    \mathrm{Hom}_k(\mathrm{Spec}(A) \times_k \mathrm{Spec}(k[t]/(t^{m+1})), \mathrm{Spec}(R))  \simeq \mathrm{Hom}_k(\mathrm{Spec}(A \otimes k[t]/(t^{m+1})), \mathrm{Spec}(R))
\end{equation*} which is nothing but $\left \{\varphi: k[x_1,...,x_n] \longrightarrow A[t]/(t^{m+1}) \mid \varphi(f_i) = 0 \ \forall \ i = \overline{1,r} \right \}$. For such a $\varphi$, set:
\begin{equation*}
    \varphi(x_i) = a_i^0 + a_i^1 t + \cdots + a_i^m t^m \ \forall \ i =\overline{1,n}
\end{equation*} and,
\begin{equation*}
    \varphi(f_i) = F^0_i(a^u_v) + F^1_i(a^u_v)t + \cdots + F^m_i(a^u_v) t^m
\end{equation*} where $u = \overline{0,m}, v = \overline{1,n}$ and $F^t_i$'s are polynomials in $a^{u}_v$. Consequently, we see that $\varphi(f_i)=0$ if and only if all $F^t_i(a^u_v) = 0$; and hence
\begin{align*}
    \left \{\varphi: k[x_1,...,x_n] \longrightarrow A[t]/(t^{m+1}) \mid \varphi(f_i) = 0 \ \forall \ i = \overline{1,r} \right \} &  = \mathrm{Hom}(k[x_j,x^0_j,...,x^m_j]_{j=\overline{1,n}}/(F_i^l(x^u_j)), A) \\
    & = \mathrm{Hom}(\mathrm{Spec}(A),\mathrm{Spec}(R_m))
\end{align*}
where $R_m = k[x_j,x^0_j,...,x^m_j]_{j=\overline{1,n}}/(F_i^l(x^u_j))$; and finally we can define $J_m(X) = \mathrm{Spec}(R_m)$.

 

Definition 2. For $m \geq n$, the natural surjections:
\begin{equation*}
       k[t]/(t^{m+1}) \twoheadrightarrow k[t]/(t^{n+1}) \twoheadrightarrow  k
\end{equation*} induced transition morphisms $\pi_{m,n}: J_m(X) \longrightarrow J_n(X)$, make $(J_m(X), \pi_{m,n})$ a projective system. Define $J_{\infty}(X) = \underset{m \longrightarrow \infty}{\lim} J_m(X)$ and denote by $\pi_m$ the $m^{th}$-canonical projection $\pi_m:J_m(X) \longrightarrow J(X)$.

 

Remark. It is not trivial that the limit $\underset{m \longrightarrow \infty}{\lim} J_m(X)$ exists in the category of schemes. We must prove that the transition morphisms $\pi_{m,n}$'s are affine.

 

Proposition 3. For any $k$-scheme $Z$, we have:
\begin{equation*}
    \mathrm{Hom}_k(Z \hat{\times_k}  \mathrm{Spec}(k[[t]]), X) \simeq \mathrm{Hom}_k(Z,J_{\infty}(X))
\end{equation*} where $Z \hat{\times} \mathrm{Spec}(k[[t]])$ means the formal completion of $Z \hat{\times}  \mathrm{Spec}(k[[t]])$ along the subscheme $Z \times_k \left \{0 \right \}$.

 

Definition 4. For $m \in \mathbb{N}$, the scheme $J_m(X)$ is called the $m^th$ jet scheme of $X$ and $J_{\infty}(X)$ is called the arc space of $X$. For any $k$-scheme $Z$, elements in $\mathrm{Hom}_k(Z,J_m(X))$ are called $Z$-valued $m$-jets of $X$ and elements in $\mathrm{Hom}_k(Z,J_{\infty}(X))$ are called $Z$-valued arcs of $X$. If $Z = \mathrm{Spec}(k)$, we just say $m$-jets or arcs.

 

Example 5. Let $X = V(x^3 + y^2) \subset \mathbb{A}^2_k$. View $x,y$ as formal power series in $t$ and consider the equation:
    \begin{equation*}
        (a_0+a_1t+\cdots)^3 + (b_0+b_1t+\cdots)^2 = 0.
    \end{equation*} By truncating the above equation at degree $m+1$, it gives us the defining equations of $J_m(X)$. For instance, $J_0(X)$ is given by $a_0^3+b_0^2=0$; $J_1(X)$ is given by $a_0^3+b_0^2=0$ and $3a_0^2 a_1 + 2b_0 b_1=0$.

 

Proposition 6. Let $X \longrightarrow Y$ be an étale morphism of $k$-varieties, then $J_m(X) \cong J_m(Y) \times_Y X$ for any $m \in \mathbb{N} \cup \left \{\infty \right \}$.

 

Proof. We prove that equality on the level of functors of points. We have:
\begin{equation*}
   \mathrm{Hom}(-.J_m(X)) \simeq \mathrm{Hom}(- \times_k \mathrm{Spec}(k[[t]]/(t^{m+1})), X)
\end{equation*} and
\begin{equation*}
    \mathrm{Hom}(-,J_m(Y) \times_Y X) \simeq\mathrm{Hom}(-,J_m(Y)) \times\mathrm{Hom}(-,X) \simeq \mathrm{Hom}(- \times_k \mathrm{Spec}(k[[t]]/(t^{m+1})), Y) \times \mathrm{Hom}(-,X)
\end{equation*} For a $k$-scheme $Z$ we consider the diagram:

Screenshot 2022-04-13 at 18-08-09 m2thesis.png

We have to show that  for a $Z$-valued $m$-jet of $Y$ and $Z$-valued $0$-jet of $X$ induce a $Z$-valued $m$-jet of $X$ (the other direction is obvious). Since $X \longrightarrow Y$ is étale, it is formally étale so such a dashed arrow exists.

 

Corollary 7. Let $U \hookrightarrow X$ be an open immersion, then $J_m(U) \hookrightarrow J_m(X)$ is also an open immersion for any $m \in \mathbb{N} \cup \left \{\infty \right \}$.

 

By an analogous method, we deduce the following important result:

 

Proposition 8. Let $X$ be a smooth $k$-scheme of dimension $d$. Then $J_m(X)$ is locally a $\mathbb{A}^{md}$-bundle over $X$. In particular, $J_m(X)$ is smooth of dimension $(m+1)d$. In the same way, $J_{m+1}(X)$ is locally a $\mathbb{A}^d$-bundle over $J_m(X)$.




#733238 Motivic integration: an introduction

Posted by bangbang1412 on 13-04-2022 - 19:44 in Toán học hiện đại

Motivic integration: general definition (without smoothness)

 

Let $X$ be an algebraic $k$-variety of pure dimension $d$, we want to extend our measure to a broader generality (without smoothness conditions) so that we can integrate simple functions $\alpha: J_{\infty}(X) \longrightarrow \mathbb{Z}$ whose fibers are well-understood.

 

Definition 29. Let $C$ be a constructible subset of $X$ such that $\pi_n^{-1}(B_n) = C$ for some constructible subset of $J_n(X)$. If furthermore:

  • $\pi_n(C) = B_n$,
  • $\pi_m(C) \subset J_{m}(X)$ is constructible for any $m \geq n$,
  • The truncation morphisms $\pi_{m+1,m}: \pi_{m+1}(C) \longrightarrow \pi_m(C)$ is a piecewise trivial fibration with fiber $\mathbb{A}^d$,

then we say that $C$ is stable at level $n$. We say that $C$ is stable if it is stable at some level. When $X$ is smooth, all cylinders are stable.

 

Definition 30. Let $C$ be a stable cylinder at level $n$. We set $$\widetilde{\mu}(C) = \frac{[\pi_n(C)]}{\mathbb{L}^{(n+1)d}} \in \mathscr{M}_k$$ The stability condition ensures that this definition is independent of the choice of $n$. By proposition 8, when $X$ is smooth, all cylinder subsets are stable. In particular, $J_{\infty}(X)$ is a stable cylinder, and:
\begin{equation*}
    \widetilde{\mu}(J_{\infty}(X)) = \frac{[X]}{\mathbb{L}^{d}}.
\end{equation*} Theorem 31. There exists an algebra $\mathbf{B}_X$ . of subsets of $J_{\infty}(X)$, which contains all stable cylinders nd a unique map $\mu: \mathbf{B}_X \longrightarrow \hat{\mathscr{M}_k}$ satisfying the following conditions:

  • If $C$ is a stable cylinder set, then $\mu(C) = \widetilde{\mu}(C)$.
  • If $C \in \mathbf{B}_X$ is contained in $J_{\infty}(Z)$ where $Z$ is a closed subvariety of $X$ with $\dim(Z) < \dim(X)$, then $\mu(C) =0$.
  • Let $(C_i)_{i \in \mathbb{N}}$ be a sequence in $\mathbf{B}_X$ such that $C_i$'s are mutually disjoint and $C = \bigsqcup C_i$ belongs to $\mathbf{B}_X$, then $\sum \mu(C_i)$ converges to $\mu(C)$ in $\hat{\mathscr{M}}_k$.
  • If $C, D$ are in $\mathbf{B}_X$ with $C \subset D$, and if $\mu(D)$ belongs to the closure $\hat{F}_m$ of $F_m$ in $\hat{\mathscr{M}_k}$, then $\mu(C) \in \hat{F}_m$.

Remark. Elements in $\mathbf{B}_X$ are called semi-algebraic sets, but we do not stress to the precise definition here.

 

Definition 32. Let $C$ be in $\mathbf{B}_X$ and $\alpha: C \longrightarrow \mathbb{Z} \cup \left \{ \infty \right \}$ a function such that $\alpha^{-1}(s) \in \mathbf{B}_X$ for any $s \in \mathbb{Z} \cup \left \{ \infty \right \}$ and $\mu(\alpha^{-1}(\infty)) = 0$, we set:
\begin{equation*}
    \int_C \mathbb{L}^{-\alpha}d\mu  = \sum_{s\in \mathbb{Z}} \mu(C \cap \alpha^{-1}(s)) \mathbb{L}^{-s},
\end{equation*} in $\hat{\mathscr{M}}_k$, whenever the right hand side converges in $\hat{\mathscr{M}}_k$, in which case we say $\mathbb{L}^{-\alpha}$ is integrable on $C$.

 

Theorem 33. (Change of varibles) Let $X, Y$ be algebraic $k$-varieties of pure dimension $d$ and $h : Y \longrightarrow X$ be a proper birational morphism. Let assume $Y$ to be smooth. Let $C \in \mathbf{B}_X$ and $\alpha: J_{\infty}(X) \longrightarrow \mathbb{N}$ be a simple function. Then
\begin{equation*}
        \int_C \mathbb{L}^{-\alpha}d\mu = \int_{h^{-1}(C)} \mathbb{L}^{-\alpha \circ h - \mathrm{ord}  h^*(\Omega^d_X)} d\mu.
\end{equation*}
Now we can present a rough proof of Bartyrev's theorem. The following diagram illustrates the construction of $\mathscr{M}_k$:


Screenshot 2022-04-13 at 19-24-04 arXiv math_0507404v1 math.AG 20 Jul 2005 - blickle.pdf.png

and it motivates the following proof.

 

Theorem 34. (Bartyrev) Let $X_1,X_2$ be two birational smooth Calabi-Yau varieties, then they have the same Hodge numbers.

 

Proof. Let $K$ denote the canonical divisor. We resolve the birational map to a Hironaka hut:

Screenshot 2022-04-13 at 19-26-40 arXiv math_0507404v1 math.AG 20 Jul 2005 - blickle.pdf.png

In the change of variables formula, we let $\alpha: J_{\infty}(X_1) \to \mathbb{Z} \cup \left \{ \infty \right \}$ be the zero map. Then

$$\int_{J_{\infty}(X_1)} \mathbb{L}^{-\alpha} d\mu = \mu(F^{-1}(0)) = \frac{[\pi(J_{\infty}(X_1)]}{\mathbb{L}^{(0+1)n}} = \frac{[X_1]}{\mathbb{L}^n},$$ and analogously, let $\alpha': J_{\infty}(X_2) \to \mathbb{Z} \cup \left \{ \infty \right \}.$ be the zero map, we see that $\alpha \circ (\pi_1)_{\infty} = \alpha' \circ (\pi_2)_{\infty}$, both equal the zero map. Applying the change of varibles,

$$\int_{J_{\infty}(Y)} \mathbb{L}^{-\alpha \circ (\pi_1)_{\infty} - \mathrm{ord}\pi_1^*K_{Y/X_1}}d\mu = \int_{J_{\infty}(X_2)} \mathbb{L}^{-\alpha'}d\mu = \frac{[X_2]}{\mathbb{L}^n},$$ which implies that $[X_1]=[X_2]$ in $\hat{\mathscr{M}}_{\mathbb{C}}$, we apply the Hodge polynomial to deduce the theorem.

 

Example 35. Let $D = \varnothing, X' = \mathrm{Bl}_Y(X)$ be the blow of $X$ along the smooth center $Y$ of codimension $c$ in $X$. The relative canonical divisor is $K_{X'/X}=(c-1)E$ where $E$ is the exceptional divisor of the blowup. Using the previous proposition, we have:
\begin{align*}
    \int_{J_{\infty}(X')} \mathbb{L}^{-\mathrm{ord}_{K_{X'/X}}} d\mu_{X'} & = \int_{J_{\infty}(X')} \mathbb{L}^{-\mathrm{ord}_{(c-1)E}} d\mu_{X'} \\
    & = [X' \setminus E] + \frac{[E]}{[\mathbb{P}^c]} \\
    & = [X \setminus Y ] + [Y] = [X].
\end{align*}

Thanks to the change of variables formula, we deduce the rationality of the motivic zeta function and a proof of Bartyrev's theorem.

 

Theorem 36. Let $X$ be an algebraic $k$-variety and let $A$ be a semi-algebraic subset ($\in \mathbf{B}_X$) of $J_{\infty}(X)$. The power series:
\begin{equation*}
    P_C(T) =  \sum_{n=0}^{\infty}[\pi_n(A)]T^n,
\end{equation*} considered as an element of $\mathscr{M}_k[[T]]$, is rational and belongs to $\mathscr{M}_k[T]_{loc}$.

 

Motivic zeta function and motivic nearby cycles

 

Let $k$ be a field of characteristic zero. Let us assume that $X$ is a smooth quasi-projective $k$-scheme of pure dimension $d$ and $f: X \longrightarrow \mathbb{A}_k^1$ is a flat morphism of $k$-schemes. Consider the diagram

Screenshot 2022-04-13 at 19-14-08 m2thesis.png

where $i$ is the zero section of the structural morphism of the affine line and $j$ its complement. We identify $f$ with the image of $t$ under the ring morphism $k[t] \longrightarrow \Gamma(X,\mathcal{O}_X)$.

 

Definition 37. The motivic zeta function is defined as follows
\begin{equation*}
    Z_f(T) = \sum_{n \geq 1} Z^1_n T^n \in \mathscr{M}_{X_{\sigma}}[[T]],
\end{equation*} where $Z^1_n = \mathbb{L}^{-nd}[\left \{\varphi \in J_n(X) \mid f \circ \varphi = t^n + O(t^{n+1}) \right \}]  = \mathbb{L}^{-nd}[\mathscr{X}^1_n] \in \mathscr{M}_{X_{\sigma}}$.

 

Remark. The expression of $\mathscr{X}^1_n$ requires $X$ to be of pure dimension $d$. Otherwise, one has to work connected components by connected components. We also know from theorem 36 that $Z_f(T) \in \mathscr{M}_{X_{\sigma}}[T]_{loc}$.

 

Let $h: X' \longrightarrow X$ be an embedded resolution of the singularities of $(X,X_{\sigma})$. By this, we mean a proper morphism $h: Y \longrightarrow X$ with $X'$ smooth such that the restriction
\begin{equation*}
    h: X' \setminus h^{-1}(X_{\sigma}) \longrightarrow X \setminus X_{\sigma}
\end{equation*} is an isomorphism and $h^{-1}(X_{\sigma}) = \sum_{i \in I} m_i D_i$ has only simple normal crossings as a subvariety of $X'$. For $\varnothing \neq J \subset I$, we define $D_J, D_J^{\circ}$ as in the previous post. We denote by $\rho_J: \widetilde{D}^{\circ}_J \longrightarrow D_J^{\circ}$ the étale cover of $D^{\circ}_J$, locally defined as follows. For any $x \in D_J^{\circ}$, there exists an affine open neighborhood $U$ of $x$ in $X$ (for the Zariski topology), a regular sequence of elements $(t_j)_{j \in J}$ of the ring $\Gamma(U, \mathcal{O}_X)$, and a unit $u \in \Gamma(U,\mathcal{O}_X^{\times})$ such that
\begin{equation*}
    f = u \prod_{j \in J} t_j^{N_j}
\end{equation*} and such that the component $D_j \cap U$ of $D \cap U$, for any $j \in J$, can be identified with the affine closed subscheme $V(t_j)$ of $U$. The base change of $\rho_J$ along the opent immersion $U \cap D_J^{\circ} \hookrightarrow D_J^{\circ}$ is identified with the finite étale morphism of $k$-schemes
\begin{equation*}
    \mathrm{Spec}\left(\mathcal{O}_{U \cap D_J^{\circ}}[T]/(T^{N_J}- u) \right) \longrightarrow U \cap D^{\circ}_J,
\end{equation*} where the strictly positive ineger $N_J$ is the greatest common divisor of the $N_j$ for all $j \in J$.

 

Theorem 38. Let $h: X' \longrightarrow X$ be an embedded resolution of the singularities of $(X,X_{\sigma})$. Let us denote by $D = h^{-1}(X_{\sigma}) = \sum_{i\in I} m_i D_i$ the exceptional divisor of $h$, that is supposed to be a simple normal crossings divisor, with $D_i$, $i \in I$, as (reduced) irreducible components. There exist strictly positive integers $n_i$, $i \in I$, such that we have the following formulars
\begin{equation*}
    Z_f(T) = \sum_{\varnothing \neq J \subset I}(\mathbb{L}-1)^{\left|J \right|-1}[\widetilde{D}^{\circ}_J]\prod_{j \in J}\frac{1}{T^{-m_j}\mathbb{L}^{n_j}-1} \in \mathscr{M}_{X_{\sigma}}[[T]],
\end{equation*} and
\begin{equation*}
    Z_{f,x}(T)  = \sum_{\varnothing \neq J \subset I}(\mathbb{L}-1)^{\left|J \right|-1}[\widetilde{D}^{\circ}_J \cap h^{-1}(x)]\prod_{j \in J}\frac{1}{T^{-m_j}\mathbb{L}^{n_j}-1} \in \mathscr{M}_{k}[[T]],
\end{equation*} for any $x \in X_{\sigma}(k)$.

 

Definition 39. The motivic nearby cycle $\psi_f$ is defined as, thanks to the rationality of the motivic zeta function,
\begin{equation*}
    \psi_f = -\left(\underset{T \longrightarrow +\infty}{\lim} Z_f(T) \right)= \sum_{\varnothing \neq J \subset I} [\widetilde{D}^{\circ}_J](1 - \mathbb{L})^{\left|J \right|-1} \in \mathscr{M}_{X_{\sigma}}.
\end{equation*} If $x \in X_{\sigma}(k)$, we define the motivic Milnor fiber as follows.
\begin{equation*}
    \psi_{f,x} = -\left(\underset{T \longrightarrow +\infty}{\lim} Z_{f,x}(T) \right)= \sum_{\varnothing \neq J \subset I} [\widetilde{D}^{\circ}_J \cap h^{-1}(x)](1 - \mathbb{L})^{\left|J \right|-1} \in \mathscr{M}_{k}.
\end{equation*}




#733235 Motivic integration: an introduction

Posted by bangbang1412 on 13-04-2022 - 18:25 in Toán học hiện đại

Order function associated to an effective divisor

 

Let $X$ be a smooth $k$-variety of dimension $d$. Let $D$ be an effective divisor, $x$ is a point in $X$ and $g$ is a local defining equation for $D$ on a neighborhood $U$ of $x$ in $X$. For an arc $\gamma_u$ over a point $u \in U$. The intersection number $\gamma_u \cdot D$ is defined to be the order of vanishing of the formal power series $g(\gamma_u(t))=0$ at $t=0$.

 

Definition 20. Define the function $F_D$ to be:

\begin{align*}
    F_D: J_{\infty}(X) & \longrightarrow \mathbb{Z}_{\geq 0} \cup \left \{\infty \right \}\\
    \gamma_u & \longmapsto \gamma_u \cdot D.
\end{align*}

We want to integrate the function $F_D$ over $J_{\infty}(X)$ and hence we must understand the level sets $F^{-1}_D(s)$ for $s \in \mathbb{Z}_{\geq 0} \cup \left \{\infty \right \}$.

 

For an effective divisor $D = \sum_{i=1}^r a_i D_i$ ($D_i$'s are prime divisors) and $J \subset \left \{1,2,...,r \right \}$ a subset, define:
\begin{equation*}
    D_J = \begin{cases}
     \bigcap_{j \in J} D_j & \text{if} \ J \neq \varnothing, \\
     Y & \text{if} \ J = \varnothing
    \end{cases}
    \ \ \ \ \text{and} \ \ \ \
    D^{\circ}_J  = D_J \setminus \bigcup_{j \in \left \{1,2,...,r\right \}\setminus J} D_j.
\end{equation*} Recall that a divisor $D = \sum_{i=1}^r a_i D_i$ on $X$ has only simple normal crossings if at each point $x \in X$, there is a neighborhood $U$ of $x$ with coordinates $x_1,...,x_n$ for which a local defining equation for $D$ is $g = x_1^{a_{j_1}}\cdots x_{j_r}^{a_{j_x}}$ with $0 \leq j_x \leq d$.

 

Lemma 21. For $D=\sum a_i D_i$ has only simple normal crossings such that all $D_i$'s are smooth, $F^{-1}_D(s)$ is a cylinder set for $s \in \mathbb{Z}_{\geq 0}$.

 

Lemma 22. $F^{-1}_D(\infty)$ is not a cylinder set but a countable intersection of cylinder sets.

 

So far, lemma 22 tells us that $F_D$ is not $\widetilde{\mu}$-measurable ($\widetilde{\mu}$ in definition 19) because $F_D^{-1}(\infty)$ is not a cylinder set. To proceed, we have to extend $\widetilde{\mu}$ to a measure $\mu$ such that $F^{-1}_D(\infty)$ is $\mu$-measurable. We first see that $J_{\infty}(X) \setminus F_D^{-1}(\infty)$ is a countable disjoint union of cylinder sets
\begin{equation*}
    J_{\infty}(X) \setminus \pi^{-1}_0 \pi_0(F_D^{-1}(\infty)) \sqcup \bigsqcup_{n \in \mathbb{Z}_{\geq 0}} \left(\pi_n^{-1}\pi_n(F_D^{-1}(\infty)) \setminus \pi_{n+1}^{-1}\pi_{n+1}(F_D^{-1}(\infty)) \right).
\end{equation*} The above decomposition suggests us that we should extend $\widetilde{\mu}$ to a measure $\mu$ defined on the colletion of countable disjoint union of cylinder sets so that $J_{\infty}(X)\setminus F_D^{-1}(\infty)$ (and hence $F_D^{-1}(\infty)$) is $\mu$-measurable. However, countable sums are not defined in $\mathscr{M}_k = K_0(\mathrm{Var}_k)[\mathbb{L}^{-1}]$ and nothing warrants that our measure is well-defined in the sense that it is independent of the choice of the decomposition into countable disjoint union of cylinder sets. Kontsevich solved both problems at once! Follow a paper of Loeser, one should proceed by analogy with $p$-adic integration: $K_0(\mathrm{Var}_k)$ plays the role of $\mathbb{Z}$ and $K_0(\mathrm{Var}_k)[\mathbb{L}^{-1}]$ plays the role of $\mathbb{Z}[p^{-1}]$. Since in $\mathbb{R}$, $p^{-i}$ has limit $0$ as $i \longrightarrow \infty$, we should complete $K_0(\mathrm{Var}_k)[\mathbb{L}^{-1}]$ in such a way that $\mathbb{L}^{-i}$ has limit $0$ as $i \longrightarrow \infty$.

 

Definition 23. Definine $F^m$ to be the subgroup of $\mathscr{M}_k$ generated by element of the form $[S]\mathbb{L}^{-i}$ with $\dim(S)\leq i - m$. We have $F^{m+1} \subset F^m, \ \mathbb{L}^{-m} \in F^m$ and $F^m F^n \subset F^{m+n}$. We denote by $\hat{\mathscr{M}}_k$ the completion with respect to this filtration.

 

Definition 24. Let $\mathcal{C}$ denote the collection of countable disjoint unions of cylinder sets $\coprod_{i \in \mathbb{N}} C_i$ for which $\widetilde{\mu}(C_i) \longrightarrow 0$ as $i \longrightarrow \infty$, together with their complements. Extend $\widetilde{\mu}$ to a measure $\mu$ on $\mathcal{C}$ given by
\begin{equation*}
    \bigsqcup_{i \in \mathbb{N}} C_i \longmapsto \sum_{i \in \mathbb{N}}\widetilde{\mu}(C_i).
\end{equation*} It is nontrivial to show that this definition is independent of the choice of $C_i$'s.

 

Lemma 25. $\widetilde{\mu}(\pi_n^{-1}\pi_n(F_D^{-1}(\infty))) \in F^{n+1} \subset \mathscr{M}_k$.

 

Lemma 26. $F_D^{-1}(\infty)$ is $\mu$-measurable and $\mu(F_D^{-1}(\infty))= 0$.

 

Definition 27. (naive version of motivic integration). Keeping the same hypotheses on $X$ and $D$ ($D$ has only simple normal crossings), we define the motivic integral of the pair $(X,D)$ to be
\begin{equation*}
    \int_{J_{\infty}(Y)} \mathbb{L}^{-F_D}d\mu = \sum_{s \in \mathbb{Z}_{\geq 0} \cup \left \{\infty \right \} } \mu(F_D^{-1}(s))\mathbb{L}^{-s}.
\end{equation*} Since $F_D^{-1}(\infty)$ has measure zero so in fact the sum on the right is over $\mathbb{Z}_{\geq 0}$.

 

Proposition 28. With the same hypotheses as in the previous definition. Then:
\begin{equation*}
    \int_{J_{\infty}(X)} \mathbb{L}^{-F_D}d\mu = \sum_{J \subset \left \{1,2,...,r\right \}} [D^0_J]\left( \prod_{j \in J}\frac{\mathbb{L}-1}{\mathbb{L}^{a_j+1}-1} \right) \mathbb{L}^{-d}.
\end{equation*}




#734647 Ghi chú về đối đồng điều động lực

Posted by bangbang1412 on 27-08-2022 - 11:22 in Toán học hiện đại

Anh nghĩ không nên dịch motivic là động lực vì motivic là tính từ của motif. Động lực thì lại là motivation mất rồi.


Đúng là vậy, bình thường em giữ nguyên từ motivic nhưng vì muốn Việt hóa nên ghi theo cách wiki chứ từ motivic chả có nghĩa gì thực sự liên quan từ động lực.



#734641 Ghi chú về đối đồng điều động lực

Posted by bangbang1412 on 26-08-2022 - 22:24 in Toán học hiện đại

Gửi mọi người một self-study note của mình về đối đồng điều động lực (motivic cohomology) phát triển bởi Voevodsky. Đối đồng điều động lực được dự đoán tồn tại bởi Beillinson, cụ thể, ông dự đoán tồn tại một phức $\mathbb{Z}(n)$ sao cho hypercohomology trên Zariski site này cho ta một đối đồng điều $H^{*,n}(X,\mathbb{Z})=\mathbb{H}_{Zar}^*(X,\mathbb{Z}(n))$ mà khi hạn chế tại một số bậc đặc biệt ta thu được K-lý thuyết Milnor, nhóm Bloch-Chow bậc cao và đồng thời có một dãy phổ hội tụ về K-lý thuyết Quillen sao cho sau khi tensor với $\mathbb{Q}$ dãy phổ này suy biến về $\gamma$-lọc của K-lý thuyết Quillen. Nổi tiếng hơn, giả thuyết Bloch-Kato-Milnor dự đoán tồn tại một đẳng cấu $K^M_*(F)/l \simeq H^*_{et}(F,\mu_l^{\otimes *})$ trong đó $F$ là một trường, $l$ nguyên tố sao cho $1/l \in F$ được Voevodsky chứng minh tương đương với giả thuyết Beillinson-Lichtembaum $H^{p,q}(X,\mathbb{Z}/l) \simeq H^p_{et}(X,\mu_l^{\otimes q})$. Voevodsky sau đó đã được huy chương Fields vì chứng minh trọn vẹn giả thuyết Bloch-Kato bằng cách xây dựng một lớp đa tạp dựa trên công trình của Rost. Với mình đây là thành công đầu tiên hướng tới lý thuyết motive của Grothendieck vì giả thuyết Bloch-Kato đã kết nối hai loại bất biến: transcendental (nhóm Chow) và arithmetic (đối đồng điều etale).

 

Đối đồng điều động lực tới nay có rất nhiều cách xây dựng, có thể kể đến:

  1. Như hypercohomology trên Zarikis hoặc Nisnevich site.
  2. Như nhóm Bloch-Chow bậc cao.
  3. Như hom-set trong phạm trù motive hình học $\mathbf{DM}_{gm}$ hoặc phạm trù motive hình học effective $\mathbf{DM}^{eff}_{gm}$ (phạm trù này là một ứng viên khá tốt cho phạm trù mixed motives dự đoán bởi Grothendieck nhưng rất tiếc chỉ bằng một ví dụ đơn giản Voevodsky chứng minh nó không có $t$-structure nào theo nghĩa của Deligne.
  4. Biểu diễn trong phạm trù đồng luân ổn định motivic (không trong note) bằng vật biểu diễn là phổ Eilenberg-MacLane motivic.

Trong note của mình mình chọn hai cách $1$ và $3$, mình không chứng minh chúng agree với nhau mà chọn từng hình thức luận sao cho tiện việc tính toán và đi thẳng vào những chỗ cần đi. Tất cả các định nghĩa trên đều đồng nhất khi ta xét trên phạm trù các đa tạp trơn trên một trường. Trong trường hợp đặc số $0$ trường có giải kì dị ta có thể chỉ xét $k$-đa tạp (không nhất triết hơn) mà vẫn có đối đồng điều motivic.

 

Mọi người có thể thảo luận thêm về đối đồng điều động lực tại chủ đề này luôn.

Attached Files




#741176 Đánh giá tổng Kloosterman và biến đổi Fourier l-adic

Posted by bangbang1412 on 24-08-2023 - 08:14 in Toán học hiện đại

Here I'd like to talk more about $D^b_{ctf}(X)$ (coefficient in some $\mathbb{Z}/n$) and this should partially explain why Grothendieck pointed out to Illusie as "the right notion". Every object in $D^b_{ctf}(X)$ is in fact quasi-isomorphic to a bounded flat complex whose components are constructible! (note that my definition in the previous answer did not include the constructibility). Any such (bounded, flat, constructible components) is called a perfect complex.

 

In some sense, the flatness is equivalent to projectiveness, and hence when we deal with derived categories of modules, we shoud replace everywhere flatness with projectiveness. Let $R$ be a ring, a complex in $D(R)$ is called perfect if it is quasi-isomorphic to a bounded complex of finite projective module. We denote by $D_{perf}(R)$ the triangulated subcategory of $D(R)$ formed by perfect complexes. This definition is slightly different with the ones in $D^b_{ctf}(X)$ but in $D^b(R)$ they are almost equivalent, namely, a complex in $D^b(R)$ is necessarily perfect provided that its cohomology are perfect (cohomology are perfect $\simeq$ cohomology are constructible) (at this point, I haven't checked why we can remove projectiveness, this is possibly due to the boudedness that we have imposed). But more importantly,

 

Proposition. For a ring, the set of compact objects of $D(R)$ are precisely perfect complexes.

 

Remind that a compact object $K$ in a category with small direct sums is a object such that $\operatorname{Hom}(K,-)$ commutes with small direct sums.

 

Proposition. The smallest strictly full triangulated subcategory stable under direct factors of $D(R)$ generated by a single object $R$ (regarded as a complex concentrated at degree $0$) is precisely the full subcategory of $D(R)$ consisting of perfect complexes.

 

Remind that a subcategory $\mathcal{C}$ of a category $\mathcal{A}$ with finite direct sums is stable under direct factors if whenever $X \oplus Y \in \mathcal{C}$ then both $X,Y \in \mathcal{C}$. These two results are subtle, I have to say that. Let me formulate in a more formal way: suppose that $\mathcal{T}$ is a triangulated having all direct sums and $\Lambda \subset \mathcal{T}$ is a set (not a proper class) of objects

  • There exists a smallest triangulated subcategory $ \left < \left< \Lambda \right>\right >$ containing $\Lambda$ and stable under direct sums. If all objects of $\Lambda$ are compact and $ \left < \left< \Lambda \right>\right > = \mathcal{T}$ then we say that $\mathcal{T}$ is compactly generated
  • There exists a smallest triangulated subcategory $ \left< \Lambda \right>^{ct}$ containing $\Lambda$ and stable under direct factors. If $\mathcal{T}$ is compactly generated by $\Lambda$ then (by a consequence of abstract Brown representability theorem) we have $\left< \Lambda \right>^{ct}$ is exactly the triangulated category of compact objects of $\mathcal{T}$.

Moreover, the fact that $\mathcal{T}$ is compactly generated by $\Lambda$ is equivalent to

$$\operatorname{Hom}(A[n],B) = 0 \ \forall \ A \in \Lambda \Rightarrow B = 0.$$ In terms of the formulation above, we can write $D(R) = \left < \left< R \right>\right >$ and $D_{perf}(R) = \left<R \right>^{ct}$. Their proofs can be ignored at first but you should think in comparison with the topological world. You replace $D(R)$ with $\mathbf{SH}$, the stable homotopy category, whose objects are sequences $X=(X_n)$ of simplicial sets together with morphisms $S^1 \wedge X_n \longrightarrow X_{n+1}$ so that you can define stable homotopy groups $\pi_n^{st}(X)$ and say that morphism is a stable weak equivalence if it induces isomorphisms on stable homotopy groups. Then $\mathbf{SH}$ is obtained by inverting all stable weak equivalence just like you invert all quasi-isomorphisms for complexes, it is a triangulated category whose distinguished triangles are those isomorphic to a cone sequence. There is a very special spectrum called the sphere spectrum $\mathbb{S} = (S^n)$, with transition $S^1 \wedge S^n \overset{\sim}{\longrightarrow} S^{n+1}$ and stable homotopy groups are stable homotpy groups of spheres. You can show that

$$\mathbf{SH} = \left < \left< \mathbb{S} \right>\right >.$$

Hence you can view every spectrum as a module over the sphere spectrum (and this is indeed the right way in the sense that: the category $\mathbb{SH}$ is not à priori a tensor category, to get a tensor structure you have to work with symmetric spectra, i.e. spectra with action of symmetric groups, and prove that two stable categories are equivalent and symmetra have $\otimes$ and $\underline{\operatorname{Hom}}$ moviated from the way of thinking every spectrum is a module over $\mathbb{S}$) and the triangulated subcategory formed by compact objects is $\left< \mathbb{S} \right>^{ct}$. The condition

$$\operatorname{Hom}(\mathbb{S}[n],B) = 0 \Rightarrow B = 0.$$ is in some sense equivalent to saying that for a CW-complex or simplicial set $X$, if $\pi_n(X)=0$ then $X=\bullet$ (Whitehead's theorem). Here comes to another subtle point; why these two worlds are so similar? The answer lies in the Dold-Kan correspondence theorem, basically it says that the category of chain complexes of $\mathbb{Z}$-modules is equivalent to the category of simplicial abelian groups. This equivalence maps homotopy groups to homology groups so that you can say homology groups are homotopy groups. The homotopy groups of $R[n]$ behaves like the homology of $S^n$ (concentrated at degree $0$ and $n$).

 

In summary,

  • Both $D(R)$ and $\mathbf{SH}$ are stable model categories, and hence triangulated ones. Moreover, they are all tensor triangulated category.
  • Both $D(R)$ and $\mathbf{SH}$ are compactly generated with a single generator.
  • Another example that I can provide is that derived category of $\mathcal{O}_X$-module, the single generator is $\mathcal{O}_X$ itself.

It is likely to define

$$D^b(X) = \left < \left< \text{a single generator} \right> \right>,$$

where we have to specify the generator. A naive guess is the constant sheaf on $X$, but this isn't enough, as you'd like to have Poincaré duality, you have to add Tate twist into the play. Therefore, the definition should be: $D^b_{ctf}(X)$ as the smalles full triangulated subcategory stable under direct factors of $D^b(X)$ generated by objects of the form

$$f_!(\mathbb{Z}/n)(-d)[-2d]$$ where $f: Y \longrightarrow X$ is smooth of relative dimension $d$, i.e. 

$$D^b(X) = \left < \left< f_!(\mathbb{Z}/n)(-d)[-2d] \right> \right> \ \ \ \ \text{and} \ \ \ \ D^b_{ctf}(X) =  \left< f_!(\mathbb{Z}/n)(-d)[-2d] \right>^{ct}$$

Actually, this should be true though I couldn't find a reference but by this tag on StackProject, a "weaker" result holds, where we change triangulated categories to abelian categories, namely, the category of constructible sheaves of abelian groups is the smallest full subcategory of the category of sheaves of abelian groups contaning objects of the form $j_!\mathbb{Z}/n$ (with $j: U \longrightarrow X$ being étale, aka smooth of relative dimension $0$ $\Rightarrow$ no need to consider Tate twist) and closed under finite limits and colimits.

 

But this is the way people nowadays define constructible motives. For instance, in motivic homotopy theory where we offen work with the stable homotopy category $\mathbf{SH}(X)$ of Voevodsky (has small direct sums like $D^b(X)$) in which such a nice representation like a perfect complex is not available then this way of definining constuctible seems to be an appropriate way (and much easier to work with).




#741137 Đánh giá tổng Kloosterman và biến đổi Fourier l-adic

Posted by bangbang1412 on 21-08-2023 - 23:26 in Toán học hiện đại

Đây là một bài mình viết sau khi đi nghe seminar do giáo sư Ngô Bảo Châu báo cáo hôm 17/8 tại viện Toán học với tựa đề Perverse sheaves and fundamental lemmas tuy nhiên giáo sư không có đủ thời gian để đi vào cả hai chủ đề mà bài nói xoay quanh việc đánh giá tổng Kloosterman bằng cách chuyển ngôn ngữ hàm số sang ngôn ngữ đối đồng điều và áp dụng giả thuyết Weil. Do đó mình để tựa đề như trên. Để thuận tiện, mình sẽ sử dụng tiếng anh.

 

Follow Katz's lectures on Weil II, let me spend some momemt to recall the motivating problem: given a prime $p$ and an integer $a$ s.t. $(a,p)=1$, the Kloosterman sum is defined as the complex number

$$\mathrm{Kl}(a,p) = \sum_{(x,y) \in \mathbb{F}_p: xy = a} \operatorname{exp}\left(\frac{2\pi i}{p}(x+y) \right).$$ By an elementary argument, one can see that this sum is a real number and in the early time when Kloosterman studied the Hardy-Littlewood circle method, he wanted to bound this sum by a function of $p$.

 

Some motivations

 

Định lý

(Kloosterman 1926) For any $\epsilon > 0$, we have $\left |\mathrm{Kl}(a,p) \right| < Cp^{3/4+\epsilon}$. 

Kloosterman's proof was quite elementary, however, the bound can be sharpen much more as follows.

Định lý

(Weil) We have $\left |\mathrm{Kl}(a,p) \right| \leq 2\sqrt{p}$.

This estimate is a consequence of Weil's proof of the "early Riemann hypothesis". The analytic version of Kloosterman sums is

$$\int_{-\infty}^{\infty}e^{i(ax+x^{-1})}dx$$ which is clearly not convergent, but we can approximate it by $\sqrt{a}K(\sqrt{a})$ where $K$ is the Bessel function. More generally, one can consider the Kloosterman sum

$$\mathrm{Kl}(a,p) = \sum_{xy=a \in \mathbb{F}_p} \psi(x+y) = \sum_{x \in \mathbb{F}_p}\psi(ax+x^{-1})$$ for any character $\psi:\mathbb{F}_p \longrightarrow \mathbb{C}^{\times}$, i.e. $\psi(x+y)=\psi(x)\psi(y)$. Here we can also prove that $\left| \mathrm{Kl}(a,p) \right| \leq 2 \sqrt{p}$ but even more, we can prove that

$$\mathrm{Kl}(a,p) = \alpha + \overline{\alpha}$$ where $\alpha$ is a complex number with $\left| \alpha \right| = \sqrt{p}$. This remains true if we replace $p$ by some of its power. This is where algebraic geometry enters the play. The first task is to transfer functions to sheaves. At the level of sheaves, we have more operations to manipulate (at least functions do not have something like duality).  But before one can see why we have to translate everything to cohomology language, one needs to have some clues about Grothendieck's formalism of six operations in $l$-adic cohomology.

 

l-adic cohomology

 

Let's fix a finite field $k=\mathbb{F}_{q}$ (where $q = p^n$ and $p$ prime) and $X/k$ be a variety. Given an integer $n$ invertible on $k$, then we can define that derived category $D^b_c(X,\mathbb{Z}/n)$ of chain complexes (modulo quasi-isomorphisms) of etale sheaves having cohomology sheaves are constructible. If $l \neq p$ is another prime, we define

$$D^b_c(X) = D^b_c(X, \overline{\mathbb{Q}}_l) = \left( \underset{\longleftarrow}{\lim} \ D^b_c(X,\mathbb{Z}/l^n\mathbb{Z}) \right) \otimes_{\mathbb{Z}_l} \overline{\mathbb{Q}}_l.$$ This definition is subtle and technical so one might follow Bhatt and Scholze's instruction to pretend that $D^b_c(X,\overline{\mathbb{Q}}_l)$ is some full subcategory of a derived category $D^b(X,\overline{\mathbb{Q}}_l)$. This is in fact does not cause any harm because almost every result for $D^b_c(X)$ is already true at the level $D^b_c(X,\mathbb{Z}/n\mathbb{Z})$. As far as I understand, the dissatisfaction with this limit-taking step is one of the reasons why Scholze introduced the pro-etale site.

Denote by $\mathfrak{TR}$ to be $2$-category of essentially small triangulated categories, then the family 

$$D^b_c: \mathrm{Var}/k \longrightarrow \mathfrak{TR} \  \ X \longmapsto D^b_c(X)$$ defines a $2$-functor admitting a formalism of six operations $(f^*,f_*,f_!,f^!,\otimes,\underline{\mathrm{Hom}})$, e.g. proper + smooth base change theorems, purity, Poincare duality,...

Objects of $D^b_c(X)$ are called $\mathbb{Q}_l$-sheaves or $l$-adic sheaves. The tensor product admits a unit denoted $\mathbb{Q}_{l,X}$ corresponding to the "constant" $l$-adic sheaf. For a $l$-adic sheaf $\mathcal{F}$, we define the $i$-th $l$-adic cohomology by setting

$$H^i(X \otimes_k \overline{k},\mathcal{F} \otimes_k \overline{k}) = \mathrm{Hom}_{D^b_c(X)}(\mathbb{Q}_{l,X},p_*\mathcal{F}[n]).$$ if $p: X \longrightarrow \mathrm{Spec}(k)$ is the structural morphism. Similarly, 

$$H^i_c(X \otimes_k \overline{k},\mathcal{F} \otimes_k \overline{k}) = \mathrm{Hom}_{D^b_c(X)}(\mathbb{Q}_{l,X},p_!\mathcal{F}[n]).$$ There is a subcategory of this category called smooth $l$-adic sheaves. Instead of treating (smooth) $l$-adic sheaf as complexes, we follow a shorter path:

Định lý

Let $X/k$ be an algebraic variety and $\overline{x} \longrightarrow X$ be a geometric point, then there is an equivalent of categories
$$\left \{\text{etale} \ \overline{\mathbb{Q}}_l-\text{sheaves} \right \} \overset{\sim}{\longrightarrow} \left \{\text{continuous rep. of} \ \pi_1(X,\overline{x}) \ \text{of} \ \overline{\mathbb{Q}}_l-\text{vector spaces} \right \}.$$ and moreover, smooth $l$-adic sheaves correspond to those representations which are of finite dimension. The equivalence is given by sending each etale $\overline{\mathbb{Q}}_l$ to its fiber over $\overline{x}$
.

Frobenii

 

During the study of this subject, I found out that the definitions of the Frobenius morphism and their traces are ambiguous, precisely, there are several definitions of Frobenii, and the question is: which one is the right one that is used in our calculations and how are they related to others? I'll discuss few approaches to this definition, the explicit one and the abstract one. We still fix $k = \mathbb{F}_q$ and $k_n= \mathbb{F}_{q^n}$, the unique finite extension of degree $n$ of $k$.

 

Explicit definition

 

Although there are some different notions, they all arise from a single one, namely, the absolute Frobenius.

Định nghĩa

Let $A/k$ be an algebra, the Frobenius endomorphism $\mathrm{Frob}:A \longrightarrow A$ is simply the ring homomorphism $a \longmapsto a^p$. This construction is carried to schemes as it should be: if $X/k$ is a scheme, then the absolute Frobenius endomorphism $\mathrm{Frob}: X \longrightarrow X$ is a homeomorphism at the level of underlying topological spaces but on the structure sheaf is $f \longmapsto f^p$. Alternatively, it is defined locally by the Frobeninus endomorphism of affine pieces.

Caution. The Frobenius endomorphism is not an isomorphism in general. 

Bổ đề

The Frobeinus endomorphism $\mathrm{Frob}: X \longrightarrow X$ is finite of degree $q^{\dim(X)}$.

Proof. I strongly recommend you to prove this result with $X = \mathrm{Spec}(k[x_1,...,x_n])$ and move to the general case. Otherwise you can look at Milne's note.

Bổ đề

If $f: X \longrightarrow Y$ is a morphism of $k$-schemes, then $\mathrm{Frob}_Y \circ f = f  \circ \mathrm{Frob}_X$. In other words, the Frobenius construction is natural

Proof. Obvious.

 

Much much more stronger is the following.

Định lý

If $f: U \longrightarrow X$ is an etale morphism of $k$-varieties, then the diagarm \begin{xy}
\xymatrix {
 U \ar[r]^{\mathrm{Frob}} \ar[d]_{f} & U \ar[d]_f \\
                             X \ar[r]_{\mathrm{Frob}}  &  X
}
\end{xy}

is cartesian.

Proof. By the previous lemma, there exists a canonical morphism, which is called the relative Frobenius morphism $\mathrm{Frob}_{X/U}: U \longrightarrow X \times_X U$. Note that since $f$ is etale, its base change, the projection onto the first factor $pr_X: X \times_X U \longrightarrow X$ is also etale. But $pr_X \circ \mathrm{Frob}_{X/U} = f$ so that $\mathrm{Frob}_{X/U}$ is etale. The absolute Frobenii are universally bijective (as noted in the definition), this forces $\mathrm{Frob}_{X/U}$ to be universally bijective. A morphism which is universally bijective and etale must be an isomorphism due to StackProject.

We can consider others Frobenii

  • The relative Frobenius $\mathrm{Frob}_r = \mathrm{Frob}_X \times \mathrm{id}_{\overline{k}}: X \otimes_k \overline{k} \longrightarrow X \otimes_k \overline{k}$. This one is a special case of the one in the proof above.
  • The arithmetic Frobenius $\mathrm{Frob}_a = \mathrm{id}_X \times \mathrm{Frob}_{\overline{k}}:X \otimes_k \overline{k} \longrightarrow X \otimes_k \overline{k}$.
  • The geometric Frobenius $\mathrm{Frob}_g = \mathrm{id}_X \times \mathrm{Frob}_{\overline{k}}^{-1}:X \otimes_k \overline{k} \longrightarrow X \otimes_k \overline{k}$.

The relative and arithmetic are automorphisms while the geometric and the absolute are not.

Bổ đề

Given a variety $X/k$, then we have $X(k_r)  = \overline{X}^{\mathrm{Frob}_r^n}$ where the relative Frobenius acts on $\overline{X}$ on the first factor. In other words, the set of $k_n$-points of $X$ is the set of closed points of $\overline{X}$ which is fixed under the $r$-iteration of the Frobenius.

Proof. Check on affine pieces.  

 

The next point is to formulate the Grothendieck trace formula, which (I think people may not drop this point at the first reading) is our main tool of computation. We have to find a natural way to define an endormophism, denoted $\mathrm{Frob}^*$

$$\mathrm{Frob}^*: H^i_c(X \otimes_k \overline{k}, \mathcal{F} \otimes_k \overline{k}) \longrightarrow H^i_c(X \otimes_k \overline{k}, \mathcal{F} \otimes_k \overline{k})$$ for every $l$-adic sheaf $\mathcal{F}$ and its pullback $\mathcal{F} \otimes_k \overline{k}$ to $X \otimes_k \overline{k}$.

 

Think topologically and remember how people thought about sheaves in the beginning days. Well, sheaves are actually sheaves of sections of etale spaces (by this, I really mean we have some equivalence of categories), the same thing happens here: for every $l$-adic sheaf $\mathcal{F}$ on $X$, there exists an algebraic space (which plays the role of an etale space in the topological world) $[\mathcal{F}]$ together with an etale morphism $f: [\mathcal{F}] \longrightarrow X$ such that $\mathcal{F}$ becomes the sheaf of sections of this morphism. As a consequence, we may identify $\mathcal{F}$ with $[\mathcal{F}]$. By base change, we obtain an etale morphism $f \otimes_k \overline{k}: [\mathcal{F}] \otimes_k \overline{k} \longrightarrow X \otimes_k \overline{k}$ and in a similar to the theorem above, the diagram

\begin{xy}
\xymatrix {
\overline{\mathcal{F}} = \mathcal{F} \otimes_k \overline{k} \ar[r]^{\mathrm{Frob}} \ar[d]_{f} & \overline{\mathcal{F}} \ar[d]_f \\
                             X \ar[r]_{\mathrm{Frob}}  &  X
}
\end{xy}

is cartesian. That being said, $\overline{\mathcal{F}} \simeq  \mathrm{Frob}^*\overline{\mathcal{F}}$ where by $\mathrm{Frob}^*$ I really mean pullback of a sheaf. This isomorphism yields two important facts:

  • The composition $$\mathrm{Frob}^*: H_c^i(X \otimes_k \overline{k}, \overline{\mathcal{F}}) \longrightarrow H_c^i(X \otimes_k \overline{k},\mathrm{Frob}^*\overline{\mathcal{F}}) \simeq H_c^i(X \otimes_k \overline{k}, \overline{\mathcal{F}})$$ is the one that we are seeking, where the first morphism is the natural morphism. 
  • If $x \in X \otimes_k \overline{k}$ is fixed by the $n$-iteration of the absolute Frobenius, then taking stalks induces an isomorphism $\mathrm{Frob}_x^{*n}: \mathcal{F}_x \overset{\sim}{\longrightarrow} \mathcal{F}_x$.

Định lý

(Grothendieck-Lefschetz trace formula). With these data, we have

$$\sum_{x \in X(k_n)}\mathrm{Trace}(\mathrm{Frob}_x^{*n},\mathcal{F}_x) = \sum_i (-1)^i\mathrm{Trace}(\mathrm{Frob}^{*n},H^i_c(X \otimes_k \overline{k},\overline{\mathcal{F}})).$$ In particular, 

$$\sum_{x \in X(k)}\mathrm{Trace}(\mathrm{Frob}_x^{*},\mathcal{F}_x) = \sum_i (-1)^i\mathrm{Trace}(\mathrm{Frob}^{*},H^i_c(X \otimes_k \overline{k},\overline{\mathcal{F}})).$$

If we set

$$\mathrm{Trace}_{\mathcal{F}}(x) =  \mathrm{Trace}(\mathrm{Frob}_x^{*},\mathcal{F}_x)$$ for each $x \in X(k)$, then this constitues a function

$$\mathrm{Trace}: X(k) \longrightarrow \overline{\mathbb{Q}}_l = \mathbb{C}$$ with the following properties

  • For any $x \in X(k)$ and $\mathcal{F},\mathcal{G} \in D^b_c(X)$ $$\mathrm{Trace}_{\mathcal{F}}(x)\mathrm{Trace}_{\mathcal{G}}(x) = \mathrm{Trace}_{\mathcal{F} \otimes \mathcal{G}}(x).$$
  • For any morphism of $k$-varieties $f: X \longrightarrow Y$ $$\mathrm{Trace}_{f^*\mathcal{F}}(x)  = \mathrm{Trace}_{\mathcal{F}}(f(x)).$$
  • For any $y \in Y(k)$ then $$\sum_{x \in X_y(k)} \mathrm{Trace}_{\mathcal{F}}(x) = \mathrm{Trace}_{f_!\mathcal{F}}(y).$$

Katz's point of view

 

Given a connected variety $X/k$ and for any point $x: k_r \longrightarrow X$, we get an induced group homomorphism

$$x_*: \pi_1(k_r,\overline{k}) \longrightarrow \pi_1(X,\overline{k})$$ by the functoriality of the etale fundamental group functor. Since $\pi_1(k_r)$ contains the Frobenius automorphism $\mathrm{Frob}_{k_r}: \overline{k} \longrightarrow \overline{k}, a \mapsto a^{\left| k_r \right|}$, we can consider its image via $x_*$ and set

$$x_*(\mathrm{Frob}_{k_r}) = \mathrm{Frob}_{k_r,x}.$$ Now given a smooth $l$-adic sheaf, i.e. a finitely dimensional representation 

$$\mathcal{F}: \pi_1(X) \longrightarrow \mathrm{GL}(r,\overline{\mathbb{Q}}_l),$$ and a $k$-point $x: k \longrightarrow X$ then it makes sense to consider the trace of the automorphism $\mathrm{Trace}(\mathcal{F}(\mathrm{Frob}_{k,x}))$ which is nothing but $\mathrm{Trace}_{\mathcal{F}}(x)$ considered before. However, I do not have any reference for this.

 

Artin-Schreier theory

 

Now with the formalism of $l$-adic cohomology in hands, we are ready to translate functions to cohomology. We introduce things called Artin-Schreier sheaf on $\mathbb{A}^1$. Here again, $k = \mathbb{F}_q, q = p^m$.

 

The Artin-Schreier sheaf is the morphism 

$$\begin{align*} L: \mathbb{A}^1_k &  \longrightarrow \mathbb{A}^1_k \\ t & \longmapsto t - t^q  \end{align*}$$ (here $t$ denotes the canonical coordinate on $\mathbb{A}^1$) is an etale covering whose whose Galois group is $\mathbb{F}_q$, i.e. $\mathrm{Aut}_{\mathbb{A}^1}(\mathbb{A}^1) = k$ and generated by $x \longmapsto x+1$.  Note that the fundamental group $\pi_1(\mathbb{A}^1)$ contains $\mathrm{Aut}_{\mathbb{A}^1}(\mathbb{A}^1)$ as an element of the projective system, so there is a canonical projection 

$$\pi_1(\mathbb{A}_k^1) \longrightarrow k.$$ Given any additive character $\psi: k  \longrightarrow \overline{\mathbb{Q}}_l^{\times}$, one then has a local system of rank $1$ from the composition 

$$\mathcal{L}_{\psi}: \pi_1(\mathbb{A}_k^1) \longrightarrow k \overset{\psi}{\longrightarrow} \overline{\mathbb{Q}}_l^{\times}$$ denoted $\mathcal{L}_{\psi}$, called the Artin-Schreier sheaf of $\psi$. The important fact is that

Bổ đề

$\mathrm{Trace}_{\mathcal{L}_{\psi}}(x)  = \psi(x)$ for any $x \in k = \mathbb{A}^1_k(k)$.

Proof. Since $\mathrm{Trace}_{\mathcal{L}_{\psi}}(x) = \mathrm{Trace}(\psi(\mathcal{L}_{\psi}(\mathrm{Frob}_{k,x})))$, we need to know what is $\mathcal{L}_{\psi}(\mathrm{Frob}_{k,x})$; in other words, where the Frobenius goes. We are done if we can prove that $ \mathcal{L}_{\psi}(\mathrm{Frob}_{k,x})=x$. To be continued.

 

Now we come to the main point, namely, the cohomological expression of Kloosterman sums. For any value $a$, we consider the hyperbol

$$X_a = \left \{(x,y) \in \mathbb{A}^2_k \mid xy = a \right \}$$

and consider the morphism $h_a: X_a \longrightarrow \mathbb{A}^1, (x,y) \mapsto x+y$. By theorem 5 and lemma 4, we have

$$\mathrm{Kl}(a,\psi) = \sum_{i=0}^2 (-1)^i \mathrm{Trace}(\mathrm{Frob}^*, H^i_c(X_a \otimes_k \overline{k}, h_a^*\mathcal{L}_{\psi} \otimes_k \overline{k})).$$ Note that $X_a$ is non-compact curve, so $H^0(X_a) = 0$ and by Poincare duality $H^2(X_a)=0$, therefore 

$$\mathrm{Kl}(a,\psi) = - \mathrm{Trace}(\mathrm{Frob}^*, H^1_c(X_a \otimes_k \overline{k}, h_a^*\mathcal{L}_{\psi} \otimes_k \overline{k})).$$ Note that, $$\dim \ H^1_c(X_a \otimes_k \overline{k}, h_a^*\mathcal{L}_{\psi} \otimes_k \overline{k}) = -\chi_c(X_a, h^*\mathcal{L}_{\psi})$$ the Euler characteristic with compact support. We'd like to compute this dimension first. Thanks to the Grothendieck-Ogg-Shafarevich theorem, we can compute this characteristic as follows.

Định lý

(Grothendieck-Ogg-Shafarevich). Let $\overline{X}$ be a proper smooth curve over $k$ and $X$ an open subset of $\overline{X}$ and $\mathcal{F}$ a local system on $X$. Then

$$\chi_c(X \otimes_k \overline{k},\mathcal{F}) = \chi_c(X \otimes_k \overline{k})\mathrm{rank}(\mathcal{F})  - \sum_{x \in \overline{X}\setminus X} \mathrm{Sw}_x(\mathcal{F})$$ where $\mathrm{Sw}$ are Swan conductors.

The Swan conductors are hard to be defined but in practice, one just needs to know its formal properties:

  • $\mathrm{Sw}_x(\mathcal{F})$ depends only on its restriction to the punctured formal disc $\hat{X}_x^{\bullet}$. 
  • $\mathrm{Sw}_x(\mathcal{F})=0$ when the restriction of $\mathcal{F}$ to $\hat{X}_x^{\bullet}$ is tame.
  • If $\mathcal{G}$ is a tame local system at $\hat{X}_x^{\times}$, then $\mathrm{Sw}_x(\mathcal{F} \otimes \mathcal{G}) = \mathrm{Sw}_x(\mathcal{F})\mathrm{rank}(\mathcal{G}).$

Here are some computations.

Ví dụ

If $X = \mathbb{A}^1$ and $\mathcal{F} = \mathcal{L}_{\psi}$, then by an elementary argument, we see that

$$\mathrm{Trace}(\mathrm{Frob}^*,H_c^1(\mathbb{A}^1,\mathcal{L}_{\psi})) = \sum_{x \in k} \psi(x) = 0$$ and hence $\chi_c(\mathbb{A}^1,\mathcal{L}_{\psi})=0$. By Grothendieck-Ogg-Shafarevich formula, we see that

$$0 = \chi_c(\mathbb{A}^1)\mathrm{rank}(\mathcal{L}_{\psi}) - \mathrm{Sw}_{\infty}(\mathcal{L}_{\psi})$$ and from this we deduce that $\mathrm{Sw}_{\infty}(\mathcal{L}_{\psi}) = 1$.

Ví dụ

For each $a \neq 0$, we see that $X_a \simeq \mathbb{G}_m$ so by Grothendieck-Ogg-Shafarevich formula,

$$\chi_c(X_a,h_a^*\mathcal{L}_{\psi}) = \chi_c(X_a) - \mathrm{Sw}_0(h_a^*\mathcal{L}_{\psi}) - \mathrm{Sw}_{\infty}(h_a^*\mathcal{L}_{\psi}) = - \mathrm{Sw}_0(h_a^*\mathcal{L}_{\psi}) - \mathrm{Sw}_{\infty}(h_a^*\mathcal{L}_{\psi}).$$ By the properties of Swan conductors

$$\begin{align*} \mathrm{Sw}_0(h_a^*\mathcal{L}_{\psi}) & = \mathrm{Sw}_0(x^*\mathcal{L}_{\psi} \otimes y^*\mathcal{L}_{\psi}) \\ & = \mathrm{Sw}_0(x^*\mathcal{L}_{\psi})\mathrm{rank}(y^*\mathcal{L}_{\psi})  = 1 \end{align*}$$ since $y^*\mathcal{L}_{\psi}$ is even unramified (not just tame) and by the previous example. By symmetry, $\mathrm{Sw}_{\infty}(h_a^*\mathcal{L}_{\psi}) = 1$ and finally this all implies that $\chi_c(X_a,h_a^*\mathcal{L}_{\psi})=2$.

 

Weight theory of Deligne

 

We fix once for all an identification $\iota: \overline{\mathbb{Q}}_l  \overset{\sim}{\longrightarrow} \mathbb{C}$ so that we can speak of an absolute on $\overline{\mathbb{Q}}_l$. Given a smooth $\mathbb{Q}_l$-sheaf $\mathcal{F}$ on an algebraic variety $X/k$, $k_n/k$ a finite extension of $k$.

$$\mathcal{F}: \pi_1(X) \longrightarrow \mathrm{GL}(r,\mathbb{C})$$ and a point $x \in X(k_n)$, then we say that

  • $\mathcal{F}$ is pure of weight $w$ if for each $n$, every eigenvalue of $\mathrm{Frob}^{*n}_x$ has eigenvalues with absolute values $\left|k \right|^{w/2}$.
  • $\mathcal{F}$ is mixed of weight $\geq w$ if if for each $n$, every eigenvalue of $\mathrm{Frob}^{*n}_x$ has eigenvalues with absolute values $\geq \left|k \right|^{w/2}$.
  • $\mathcal{F}$ is mixed of weight $\leq w$ if if for each $n$, every eigenvalue of $\mathrm{Frob}^{*n}_x$ has eigenvalues with absolute values $\leq \left|k \right|^{w/2}$.

We call the celebrated theorem due to Deligne, originally known as Weil conjectures.

Định lý

(Deligne) Let $X/k$ be a variety and $\mathcal{F}$ is a $l$-adic sheaf mixed of weight $\leq 0$, then every eigenvalue of 

$$\mathrm{Frob}^*:H_c^i(X \otimes_k \overline{k},  \mathcal{F} \otimes_k \overline{k}) \longrightarrow H_c^i(X \otimes_k \overline{k},  \mathcal{F} \otimes_k \overline{k})$$ has absolute values $\leq \left |k \right|^{i/2}$

In Weil II, Deligne proved something much stronger where one replaces $U \longrightarrow \mathrm{Spec}(k)$ by a morphism $f: X \longrightarrow Y$, then $R^if_!\mathcal{F}$ is mixed of weight $\leq w +i$  whenever $\mathcal{F}$ is mixed of weight $\leq w$. However, the Target theorem is enough to deduce the last part of the Weil conjectures and estimates of Kloosterman sums. 

 

From Deligne's weight theorems, the computation $\dim \ H^1_c(X_a \otimes_k \overline{k} ,h_a^*\mathcal{L}_{\psi} \otimes_k \overline{k}) = 2$, and 

$$\mathrm{Kl}(a,\psi) = - \mathrm{Trace}(\mathrm{Frob}^*, H^1_c(X_a \otimes_k \overline{k}, h_a^*\mathcal{L}_{\psi} \otimes_k \overline{k})).$$ we see that

$$\left | \mathrm{Kl}(a,\psi) \right| \leq 2p^{1/2}.$$

More generally, if we define a generalized Kloosterman sum as 

$$\mathrm{Kl}_m(a,\psi) = \sum_{x_1\cdots x_m = a, x_i \in k}\psi(x_1 + \cdots + x_m)$$ then we have an estimate $\left |\mathrm{Kl}_m(a,\psi) \right | \leq mp^{(m-1)/2}$.  




#741146 Đánh giá tổng Kloosterman và biến đổi Fourier l-adic

Posted by bangbang1412 on 22-08-2023 - 16:32 in Toán học hiện đại

In this post, I'd like to give a rapid introduction to the theory of $l$-adic Fourier transform developed by Laumon-Deligne-... My goal is not to how can we apply $l$-adic Fourier transform to prove the Weil conjectures but rather to see why their definitions are natural in comparison with the classical theory. My feeling is that it is easier to present Fourier transforms on finite fields than on measurable spaces (which require a lot of work and details) and the $l$-adic one is formally adapted from the one for finite fields.
 

Fourier analysis on finite abelian groups

Given a finite abelian group $G$, written additively, what we want to do here is to define a space $L^2(G)$ similar to the Hilbert space $L^2(X)$ of square integrable functions $X \longrightarrow \mathbb{C}$ (modulo equal almost everywhere relation) for $X$ being a measurable space. Then it is possible to develop a Fourier transform on $L^2(G)$. The finiteness seems to be a technical condition that you can see to be useful in every step. At least with this hypothesis, we do not to worry about the convergence of sums. We do not go into the "Hilbert theory" of $L^2(G)$ deeply but rather go straight to the Plancherel formula and Fourier inverse formula and see how it can be generalized to $l$-adic cohomology.

 

We define a character of $G$ to be a group homomorphism $\psi: G \longrightarrow (\mathbb{C}^{\times},\times)$. We call it trivial if $\psi(x) = 1$ for each $x \in G$. Since $G$ is finite, every $\psi(x)$ ($x \in G$) is a root of unity. In particular, every character takes values in the circle $S^1$.

 

Ví dụ

If $G = \mathbb{F}_{p}$, a field with $p$ elements, then $\psi(x) = e^{2\pi i x/p}$ is a character.

Ví dụ

If $\psi$ is a character, then $\overline{\psi}$. They are different if $\psi$ is not identical to $1$. Note that 

$$\psi(-x) = \psi(x)^{-1} = \overline{\psi(x)}$$ since it lies on $S^1$.

Ví dụ

If $\psi,\varphi$ are characters, then $\psi\varphi$ is a character as well.

Mệnh đề

If $\psi$ is a non-trivial character of $G$, then $\sum_{x \in G}\psi(x)=0$.

Proof. Since $\psi$ is non-trivial, there exists $y \in G$ with $\psi(y) \neq 1$. We have

$$\psi(y)\sum_{x \in G}\psi(x) = \sum_{x \in G}\psi(x+y) = \sum_{x \in G}\psi(x)$$ and hence the sum itself is zero because $\psi(y) \neq 1$.

Hệ quả

Let $\psi$ and $\varphi$ be characters of $G$. Then 

$$\sum_{x \in G}\overline{\psi(x)}\varphi(x) = \begin{cases} \left|k \right| & \psi = \varphi \\ 0 & \psi \neq \varphi \end{cases}$$

Proof. If $\psi = \varphi$, then it is the consequence of the fact $\psi(x)^{-1} = \overline{\psi(x)}$ while if $\psi \neq \varphi$ then $\overline{\psi}\varphi$ is a non-trivial character, hence it follows from proposition 4.

 

Now we work in the case where $G = k$ is a finite field, then we have a, being motivated from the classical case

$$\hat{f}(x) = \int_{-\infty}^{\infty} f(y)e^{-2\pi i xy} dy$$

we define the Fourier transform

$$\begin{align*} T_{\psi}f: k & \longrightarrow \mathbb{C}^{\times} \\ x & \longmapsto  \sum_{y \in k}f(y)\psi(-xy).\end{align*}$$ The Fourier transform is clearly linear, i.e. $T_{\psi}(f+g) = T_{\psi}(f) + T_{\psi}(g)$ and $T_{\psi}(af) = aT_{\psi}(f)$. The Fourier inversion formula in this case becomes almost trivial.

 

Mệnh đề
(Fourier inverse). We have

$$T_{1/\psi}(T_{\psi}f) = \left|k \right|f$$

Proof. We compute the LHS explicitly

$$\begin{align*}T_{1/\psi}(T_{\psi}f)(x) & =  \sum_{y \in k}T_{\psi}(f)(y)\overline{\psi}(-xy) \\ & = \sum_{y \in k}\left(\sum_{z \in k}f(z)\psi(-yz) \right)\overline{\psi}(-xy)  \\ & = \sum_{y,z \in k} f(z)\psi(y(x-z)) \\ & = \sum_{z \in k}f(z)\left(\sum_{y \in k}\psi(y(x-z))  \right) \\ & = \left|k \right|f(x) \end{align*}$$ thanks to corollary 5.

 

We can endorse the vector space $\mathbb{C}^k$ of functions $k \longrightarrow \mathbb{C}$ with an inner product

$$\left< f, g \right>  = \sum_{x \in k}\overline{f(x)}g(x)$$ then we have an analogue of the usual Plancherel formula.

 

Mệnh đề

(Plancherel formula). For functions $f,g: k \longrightarrow \mathbb{C}^{\times}$ and a character $\psi:k \longrightarrow \mathbb{C}^{\times}$

$$\left<T_{\psi}f, T_{\psi}g\right> = \left|k \right|\left <f,g \right >$$

Proof. We expand everything $$\begin{align*}\left<T_{\psi}f, T_{\psi}g\right> & = \sum_{x \in k}\overline{T_{\psi}f(x)}T_{\psi}g(x) \\ & =  \sum_{x \in k}\left( \sum_{y \in k}\overline{f(y)}\psi(xy) \right)\left( \sum_{z \in k}g(z)\psi(-xz) \right) \\ & = \sum_{x,y,z \in k}\overline{f(y)}g(z)\psi(x(y-z)) \\ & = \sum_{y,z \in k}\overline{f(y)}g(z)\left(\sum_{x \in k}\psi(x(y-z)) \right). \end{align*}$$ We analyze the sum $\sum_{x \in k}\psi(x(y-z))$.

  • If $y = z$ then this sum is $\left|k \right|$.
  • If $y \neq z$ then this sum is zero by proposition 4.

Hence  

$$ \sum_{y,z \in k}\overline{f(y)}g(z)\left(\sum_{x \in k}\psi(x(y-z)) \right) = \sum_{y \in k }\left|k \right|\overline{f(y)}g(y) = \left|k \right| \left<f,g \right>.$$ We are now able to motivate the definitions in the $l$-adic cohomology.

 

l-adic Fourier transform

 

We restrict ourself to the definition of something called $l$-adic Fourier transform on the affine line $\mathbb{A}^1_k$ where $k$ is a finite field. More precisely, we want to define some operator

$$T_{\psi}: D_c^b(\mathbb{A}^1,\overline{\mathbb{Q}}_l) \longrightarrow  D_c^b(\mathbb{A}^1,\overline{\mathbb{Q}}_l)$$ associated to any character $\psi: k\longrightarrow \mathbb{C}^{\times}$ and is forced to satisfy the Fourier inverse formula and the Plancherel formula. To do this, we have to have some sheaf-to-functions correspondence, for each variety $X/k$, naturally, we have the following: for any $K \in D_c^b(X,\overline{\mathbb{Q}}_l)$

$$\begin{align*} f^K: k & \longrightarrow \mathbb{C} \\ x & \longmapsto \mathrm{Trace}(\mathrm{Frob}^*_{\overline{x}},K_{\overline{x}}) = \sum_i (-1)^i \mathrm{Trace}(\mathrm{Frob}^*_{\overline{x}},\mathscr{H}^i(K)_{\overline{x}}) \end{align*}$$ where $\mathscr{H}^i$ denote cohomological sheaves, which are assumed to be constructible.

 

Given any character $\psi: k  \longrightarrow \overline{\mathbb{Q}}_l^{\times}$, one then has a local system of rank $1$ from the composition 
$$\mathcal{L}_{\psi}: \pi_1(\mathbb{A}_k^1) \longrightarrow k \overset{\psi}{\longrightarrow} \overline{\mathbb{Q}}_l^{\times}$$ denoted $\mathcal{L}_{\psi}$, called the Artin-Schreier sheaf of $\psi$. We claim that

 

Bổ đề

$f^{\mathcal{L}_{\psi}}(x) = \psi(-x)$. 

The next thing is how can we translate operations of functions to operations of sheaves:

  • (Product formula) The product of functions should correspond to tensor product of sheaves: $f^{K \otimes L}(x) = f^K(x)f^L(x)$ for every $x \in X(k)$
  • (Pullback formula) Pullback of functions should correspond to pullback of functions, which is just composition $f^{f^*K}(x) = f^K(f(x))$ for every morphism $f: X \longrightarrow Y$ of $k$-varieties and $x \in X(k)$.
  • (Sum formula) Proper pushforwards should correspond to taking sums or integrals: this is a consequence of Grothendieck-Lefschetz trace formula and proper base change theorem. For every morphism $f: X \longrightarrow Y$ of $k$-varieties and $K \in D^b_c(X,\overline{\mathbb{Q}}_l)$, we have $$f^{f_!K}(y) = \sum_{x \in X_y(k)}f^K(x)$$ for any $y \in Y(k)$.

Consider the diagram

\begin{xy}

\xymatrix{

& \mathbb{A}^1 \times \mathbb{A}^1 \ar[rr]^m \ar[dr]^{\pi^2} \ar[dl]_{\pi^1} & & \mathbb{A}^1 \\

 \mathbb{A}^1 & & \mathbb{A}^1 &

}

\end{xy}

where $\pi^1$ are projections and $m$ the multplication $(x,y) \longmapsto xy$. The $l$-adic Fourier transform is defined to be

$$\begin{align*} T_{\psi}: D^b_c(\mathbb{A}^1,\overline{\mathbb{Q}}_l) & \longrightarrow D^b_c(\mathbb{A}^1,\overline{\mathbb{Q}}_l) \\ K & \longmapsto \pi_!^1(\pi^{2*}K \otimes m^*\mathcal{L}_{\psi})[1] \end{align*}$$ where the shift $[1]$ is put in order to preserve the perversity, which is not of our interest here. The sheaf $m^*\mathcal{L}_{\psi}$ plays the role of the character $\psi(-xy)$ in the formula

$$T_{\psi}f(x) =  \sum_{y \in k}f(y)\psi(-xy).$$ We prove that our definition is really a sheaf-theoretic version of the discrete Fourier transform (up to a sign).

 

Bổ đề
$f^{T_{\psi}(K)}(x) = - \sum_{y \in k} f^K(y)\psi(-xy)$ for any $x \in k$ and $K \in D^b_c(\mathbb{A}^1,\overline{\mathbb{Q}}_l)$.

 

Proof. By the last part of our remark above, we have (we delete the shift since it is irrelevant here)

$$\begin{align*} f^{T_{\psi}(K)}(x) & =  \sum_i (-1)^i \mathrm{Trace}\bigg(\mathrm{Frob}^*_{\overline{x}}, \mathscr{H}^i\big( \pi_!^1(\pi^{2*}K \otimes m^*\mathcal{L}_{\psi})_{\overline{x}} \big) \bigg) \\ & =  - \sum_i (-1)^i \sum_{y \in k} \mathrm{Trace}(\mathrm{Frob}^*_{(\overline{x},\overline{y})}, \mathscr{H}^i(\pi^{2*}K \otimes m^*\mathcal{L}_{\psi})) \\ &  =- \sum_i (-1)^i \sum_{y \in k} \mathrm{Trace}(\mathrm{Frob}^*_{(\overline{x},\overline{y})}, \mathscr{H}^i(\pi^{2*}K) \otimes \mathscr{H}^i( m^*\mathcal{L}_{\psi})) \\ & = - \sum_i (-1)^i \sum_{y \in k} \mathrm{Trace}(\mathrm{Frob}^*_{(\overline{x},\overline{y})}, \mathscr{H}^i(\pi^{2*}K)) \mathrm{Trace}(\mathrm{Frob}^*_{(\overline{x},\overline{y})}, \mathscr{H}^i( m^*\mathcal{L}_{\psi}))  \\& =    - \sum_i (-1)^i \sum_{y \in k} \mathrm{Trace}(\mathrm{Frob}^*_{(\overline{x},\overline{y})}, \mathscr{H}^i(\pi^{2*}K)) \psi(-xy) \\ & = - \sum_{y \in k} \sum_i (-1)^i \mathrm{Trace}(\mathrm{Frob}^*_{(\overline{x},\overline{y})}, \mathscr{H}^i(\pi^{2*}K)) \psi(-xy) \\ & = - \sum_{y \in k} f^K(y)\psi(-xy) \end{align*}$$ where we have used:

  • The sum formula for $\pi^1: \mathbb{A}^1 \times \mathbb{A}^1 \longrightarrow \mathbb{A}^1$ in the first equality.
  • In the second one, homology commutes with tensor product.
  • In the third one, we applied the product formula. 
  • In the rest, we applied the pullback formula and lemma 8

Let us now verify the Plancherel formula before the Fourier inverse formula (which is more complicated). 

 

 

Định lý
(Plancherel). We have

$$\left<f^{T_{\psi}(K)},f^{T_{\psi}(L)}\right> = \left|k \right|\left<f^K,f^L\right> \ \forall \ K,L \in D^b_c(\mathbb{A}^1,\overline{\mathbb{Q}}_l)$$

Proof. This is just a formal manipulation based on lemma 6 and proposition 3.

 

Định lý
(Fourier inverse). We have

$$T_{\psi^{-1}}T_{\psi}K = K(-1)$$ where $(-1)$ denotes the Tate twist.

 

For this result, we need an auxiliary lemma, omitted proof, but can be understood heuristically as the Fourier transform of the canonical character equals the dirac delta function.

 

Bổ đề
Let $i: 0 \hookrightarrow \mathbb{A}^1$ be the canonical closed immersion, then $i_*\overline{\mathbb{Q}}_l = \delta$ is the skyscraper sheaf at the origin, we then have

$$T_{\psi}(\overline{\mathbb{Q}}_l[1]) = \delta(-1).$$

Nhận xét
. The occurence of Tate twist here is understandable because it corresponds to multiplying $1/2\pi$ in the formula

$$\delta(x) = \frac{1}{2\pi}\int_{-\infty}^{\infty}e^{ixy}dy$$

Proof of Fourier inverse. Let's consider the diagram, in which the square is cartesian

 

\begin{xy}
\xymatrix {
&  & \mathbb{A}^1 \times \mathbb{A}^1 \times \mathbb{A}^1 \ar[dr]^{\pi^{23}} \ar[dl]_{\pi^{13}} &  & \\
                       & \mathbb{A}^1 \times \mathbb{A}^1  \ar[dl]_{\pi^1} \ar[dr]^{\pi^2} & & \mathbb{A}^1 \times \mathbb{A}^1 \ar[dl]_{\pi^1} \ar[dr]^{\pi^2}& \\

\mathbb{A}^1 & & \mathbb{A}^1 & & \mathbb{A}^1
}
\end{xy}

 

and define $\alpha: \mathbb{A}^3 \longrightarrow \mathbb{A}^2$ by $(x,y,z) \longmapsto (x,y-z)$, then

$$\begin{align*} T_{\psi^{-1}}T_{\psi}K & = \pi^1_!\bigg(\pi^{2*}\big(\pi_!^1(\pi^{2*}K \otimes m^*\mathcal{L}_{\psi}\big) \otimes m^*\mathcal{L}_{\psi^{-1}} \bigg)[2] \\ & = \pi^1_! \bigg( \pi_!^{12}\pi^{23*}\big( \pi^{2*}K \otimes m^*\mathcal{L}_{\psi} \big) \otimes m^*\mathcal{L}_{\psi^{-1}} \bigg)[2] &  \text{proper base change} \\ & =\pi^1_! \pi_!^{12}\big( \pi^{23*}\pi^{2*}K \otimes \pi^{23*}m^*\mathcal{L}_{\psi} \otimes \pi^{12*}m^*\mathcal{L}_{\psi^{-1}} \big)[2] &  \text{projection formula} \\ & =  \pi^1_! \pi_!^{12}(\pi^{23*}\pi^{2*}K \otimes \alpha^*m^*\mathcal{L}_{\psi})[2] & \text{by the last lemma below} \\ & = \pi^1_! \pi_!^{13}(\pi^{13*}\pi^{2*}K \otimes \alpha^*m^*\mathcal{L}_{\psi})[2] &  \pi^1 \pi^{12} = \pi^1\pi^{13} \ \text{and} \ \pi^2 \pi^{23} = \pi^2\pi^{13} \\ & = \pi^1_! (\pi^{2*}K \otimes \pi_!^{13}\alpha^*m^*\mathcal{L}_{\psi})[2] & \text{projection formula}\end{align*}$$  Consider the cartesian diagram

\begin{xy}
\xymatrix {
\mathbb{A}^3 \ar[r] \ar[d]_{\pi^{13}} \ar[r]^{\alpha} & \mathbb{A}^2 \ar[d]_{\pi^2} \\
                             \mathbb{A}^2 \ar[r]_{\beta}  &  \mathbb{A}^1
}
\end{xy}

where $\beta(x,z) = z-x$, then by base change we get

$$\begin{align*} T_{\psi^{-1}}T_{\psi}K &  = \pi^1_!(\pi^{2*}K \otimes \beta^*\pi^2_!m^*\mathcal{L}_{\psi})[2] & \\ & = \pi^1_!(\pi^{2*}K \otimes \beta^*T_{\psi}\overline{\mathbb{Q}}_l[-1])[2] & \\ & = \pi^1_!(\pi^{2*}K \otimes \beta^*i_*\overline{\mathbb{Q}}_l(-1)[-2])[2] & \text{previous lemma} \\ & = \pi^1_!(\pi^{2*}K \otimes \beta^*i_*\overline{\mathbb{Q}}_l)(-1) \end{align*}$$ But the square

 

\begin{xy}
\xymatrix {
\mathbb{A}^1 \ar[r] \ar[d]_{\Delta} & 0 \ar[d]_i \\
                             \mathbb{A}^2 \ar[r]_{\beta}  &  \mathbb{A}^1
}
\end{xy}

 

is cartesian, where $\Delta$ is the diagonal, note that $i$ is proper so by base change again, we have

$$\begin{align*} T_{\psi^{-1}}T_{\psi}K & = \pi^1_!(\pi^{2*}K\otimes \Delta_!\overline{\mathbb{Q}}_l(-1)) & \\ & = \pi_!^!\Delta_!(\Delta^*\pi^{2*}K\otimes \overline{\mathbb{Q}}_l(-1)) & \text{projection formula} \\ & = K(-1) & \pi^1\Delta = \mathrm{id} \ \text{and} \ \pi^2 \Delta = \mathrm{id} \end{align*}$$ as desired.

 

We finish the proof by proving the following.

 

Bổ đề

The following holds

$$\pi^{12*}(m^*\mathcal{L}_{\psi^{-1}}) \otimes \pi^{23*}(m^*\mathcal{L}_{\psi}) = \alpha^*m^*\mathcal{L}_{\psi}.$$

Proof. Set $\mathbb{A}^1 = \mathrm{Spec}(k[t])$ and $\mathbb{A}^3 = \mathrm{Spec}(k[x,y,z])$ and considre

$$X = \mathrm{Spec}(k[x,y,z,u,v]/(u^{\left|k \right|}-u-xy,v^{\left|k \right|}-v-yz)) \longrightarrow \mathbb{A}^3$$ which is a Galois covering whose group of Deck transformations is isomorphic to $k \times k$. There are three projections of $X$ onto $\mathbb{A}^1$ given by

$$t \longmapsto u, t \longmapsto v-u, t \longmapsto v.$$ We view $\mathbb{A}^1$ as a scheme over itself by Artin-Schreier morphism, then the three morphisms above induce homomorphisms of groups of deck transformations

$$k \times k \longrightarrow k$$ given respectively by

$$(a,b) \longmapsto a, (a,b) \longmapsto b - a, (a,b) \longmapsto b.$$ We compose the original character $\psi$ with these three morphisms to get three new characters

$$\pi_1(X) \longrightarrow k \times k \longrightarrow k \overset{\psi}{\longrightarrow} \mathbb{C}^{\times}.$$ The three new characters correspond to the sheaves involved in the equation

$$\begin{align*} \pi^{12*}m^*\mathcal{L}_{\psi^{-1}} &\longrightarrow   \psi_1(a,b) = 1/\psi(a) \\ \alpha^*m^*\mathcal{L}_{\psi} & \longrightarrow \psi_2(a,b) = \psi(b)/\psi(a) \\ \pi^{23*}m^*\mathcal{L}_{\psi} & \longrightarrow \psi_3(a,b) = \psi(b) \end{align*}$$ then the question boils down to the trivial fact that $\psi_1\psi_3=\psi_2$.